Community Health Final

Réussis tes devoirs et examens dès maintenant avec Quizwiz!

12. A school nurse is administering medications at the school. Which of the following guidelines should be followed? a. A current drug reference should be available in case information is needed b. The nurse should administer medications brought in from home by the child c. Medications cannot be administered without a physician order d. Narcotics and controlled substances should be kept in a locked cabinet

ANS: A A current drug reference should always be available so that it can be consulted for information.

Change for the community as client must often occur at several levels because: a. Health problems caused by lifestyle are multidimensional. b. Most individuals can change their habits alone. c. Aggregates are responsible for social change. d. Geographic areas often have health risks that the nurse must identify.

ANS: A Because health problems caused by lifestyle cannot be solved simply by asking individuals to choose healthy habits, change for the community client must often take place at several levels.

The levels of practice encompassed by the Intervention Wheel are: a. Communities, individuals and families, and systems b. Assessment, diagnosis, and evaluation c. Primary, secondary, and tertiary d. Communities, populations, and aggregates

ANS: A Communities, individuals and families, and systems are the three levels of public health nursing practice.

A nurse is working with a group of clients with diabetes and is teaching a class about avoiding the long-term effects of diabetes. The nurse begins the class by reviewing the basic physiology of diabetes, which was taught the week before. This nurse is providing effective education by: a. Stimulating recall of prior learning b. Gaining attention c. Presenting the material d. Providing learning guidance

ANS: A The educator should have the learners recall previous knowledge related to the topic of interest.

An example of a point source of air pollution is: a. A smoke stack b. The number of cars and trucks c. How much fossil fuel is consumed in a community d. Ground ozone levels

ANS: A Point sources of pollution are identifiable sources of air pollution, such as a smoke stack.

Examples of modifiable behaviors include: a. Smoking, poor diet, and alcohol consumption b. Cancer, emphysema, and cardiovascular disease c. Walking, running, and aerobic exercise d. Genetic abnormalities

ANS: A The leading causes of death in 2000 were tobacco use, poor diet, physical inactivity, and alcohol consumption. These are modifiable behaviors.

The main characteristics of partnership are: a. Awareness, flexibility, and distribution of power b. Rights, responsibilities, and consensus c. Commitment, participation, and articulation d. Collaboration, advocacy, and utility

ANS: A The main characteristics of partnership are being informed, flexibility, and negotiated distribution of power.

The Guide to Clinical Preventive Services provides: a. The most recent recommendations for preventive interventions b. Assistance in interpreting Healthy People 2020 c. A basis for public health nursing practice d. Health risk appraisal instruments

ANS: A The Guide to Clinical Preventive Services provides recommendations for preventive interventions including screening tests, counseling, immunizations, and chemoprophylaxis.

A nurse who uses such data as minutes from a community meeting is using: a. Secondary analysis b. Informant interviews c. A survey d. A windshield survey

ANS: A The data is secondary because it is collected by someone else.

4. Which of the following supplies or equipment should a nurse have available in the school health office? (Select all that apply.) a. Cervical spine collars b. Complete emergency kit that fulfills American Hospital Association requirements c. Epinephrine autoinjector kit d. Material for splints

ANS: A, C, D The school nurse needs much equipment to deal with emergencies in the school. Basic necessary equipment includes full oxygen tanks with oxygen masks of different kinds, splints, cervical spine collars, sterile dressings, and an epinephrine autoinjector kit in case a child goes into anaphylactic shock after exposure to an allergen. A hospital-oriented emergency kit would become quickly outdated (medications) and extremely expensive.

A nurse may define a community as a(n) (select all that apply): a. Social group determined by geographic boundaries b. Group of people who share common values and interests c. Group of people defined by their interactions d. Individual with a specific health concern

ANS: A, B, C Community can be defined as many things, including a social group determined by geographical boundaries, a group of people who share common values and interests, and a group of people defined by their interactions.

A public health nurse uses collaboration, coalition building, and community organizing when: a. Providing case management, referral, and follow-up services with individuals b. Carrying out collective action at the systems or community levels of practice c. Conducting a community assessment d. Implementing primary and secondary prevention strategies

ANS: B Collaboration, coalition building, and community organizing are the interventions often carried out at the systems and community levels of practice. These interventions can be used at all levels of prevention.

Which characteristics observed in a teenage boy should always alert the CHN to the possibility of suicide? a. Age between 15 and 19 years b. Questioning sexual orientation and history of depression c. Threatening to cause harm to peers d. A history of torturing and abusing animals

ANS: B Depression, homosexuality, and questioning gender identity are among the leading risk factors for adolescent suicide. Those who threaten peers or torture and abuse animals are more likely to cause harm to others than to themselves.

21. Which of the following is most important for school nurses to master in order to prepare for health care delivery in the future? a. Complementary and alternative therapies such as acupuncture b. Computer and technology use c. Psychoanalytical techniques d. Self-defense techniques

ANS: B In the future, school nursing will use telehealth and telecounseling to teach health education. School nurses will use the Internet to work with children and parents.

When conducting informant interviews in a small community, the nurse would most likely contact: a. The state department of health for death records b. A local priest for congregation information c. Surrounding communities for crime comparison d. The Centers for Disease Control and Prevention (CDC) for illnesses in the area

ANS: B Informant interviews are methods of directly collecting data. County health department nurses and church officials are often good key informants.

A nurse promotes alliances among organizations for a common purpose. Which public health intervention is being implemented? a. Health teaching b. Coalition building c. Surveillance d. Referral and follow-up

ANS: B One example of coalition building is promoting alliances among organizations for a common purpose.

Evaluation of individual and group progress toward health goals is important. Which component should be included in the evaluative process? a. Type of teaching strategy used b. Recognition of accomplishments in the group c. Conflict that occurred in the group d. The type of leadership in the group

ANS: B Recognition of accomplishments in the group and of the group is built into the evaluative process.

A CHN is asked by a parent group to explain the risk factors for alcoholism. What statement should the CHN include in the explanation? a. Alcoholism is determined solely by environment. b. Alcoholism is determined partly by genes. c. Alcoholism is higher in women. d. Persons born with fetal alcohol syndrome are alcoholics from birth.

ANS: B Research has shown conclusively that alcoholism is, at least in part, genetic and not just the result of family environment.

A health educator trying to change a clients attitudes about smoking would be using which of the following domains? a. Cognitive b. Affective c. Psychomotor d. Developmental

ANS: B The affective domain is used to attempt to influence what individuals, families, communities, and populations feel, think, and value.

The biomedical model defines health as the: a. Avoidance of illness b. Absence of disease c. Promotion of healthy behaviors d. Protection from illness and disease states

ANS: B The biomedical model defines health as the absence of disease. It does not explain why populations remain healthy or how health is enhanced.

A nurse cares for the community as the client when focusing on: a. Providing care for aggregates living in the community b. The collective good of the population c. The provision of care for families in the home setting d. Providing health education in the community

ANS: B The community is considered the client when the nursing focus is on the collective good of the population. Providing education is one way the nurse can care for the community as the client

A 5-feet, 6-inch, 25-year-old female who weighs 120 pounds walks 5 miles a day because she has a long family history of early death from heart attacks and is refraining from a sedentary lifestyle. Which term best describes this example? a. Health promotion b. Illness prevention c. Health maintenance d. Health protection

ANS: B The woman is exercising to prevent an illness that runs in her family. Illness prevention is a behavior directed toward reducing the threat of illness.

Which nursing diagnosis is written at the community-level? a. Risk of hypertension related to poor diet and sedentary lifestyle b. Risk of obesity among school-age children related to lack of opportunities to engage in physical activity c. Risk of ineffective health maintenance among individuals who do not have access to a primary care provider d. Ineffective coping related to multiple stressors as evidenced by client crying and stating she has no support system

ANS: B There are three parts: risk of, among, and related to. The risk of identifies a specific problem or health risk faced by the community. Among identifies the specific community client with whom the nurse will be working in relation to the identified problem or risk. Related to describes characteristics of the community.

Why would a nurse refer to the Code of Ethics for Nurses or the Public Health Code of Ethics? a. To provide answers for ethical dilemmas b. To guide professional practice related to ethics c. To increase moral leadership in ethics d. To find a framework for ethical decision making

ANS: B These codes provide general ethical principles and guide personnel in thinking about the underlying ethics of the profession.

When would a nurse use The Guide to Clinical Preventive Services? a. Conducting group risk appraisal and risk reduction b. Providing recommendations for preventive interventions c. Completing a community wellness inventory d. Improving environmental living conditions and sanitation

ANS: B These include screening tests, counseling, immunizations, and chemoprophylaxis regimens for more than 80 conditions.

4. At the annual community health fair, the school health nurse displays a science booth that examines the hazards of ineffective hand washing. Which of the following best describes the nurse's role in this scenario? a. Consultant b. Community outreach c. Counselor d. Researcher

ANS: B When participating in community outreach, nurses reach out to residents in the community. One common way this occurs is when nurses are involved in activities such as community health fairs or festivals in the schools.

A public health nurse implements the public health intervention of health teaching at the systems-level of practice by: a. Participating in the Great American Smokeout b. Working with a local employer to provide smoking cessation education c. Providing one-on-one counseling to smokers d. Advocating for increased taxes on tobacco products

ANS: B Working with a local employer to provide smoking cessation education is the only example of health teaching at the systems-level of practice. Participating in the Great American Smokeout occurs at the community level and is not health teaching. Providing one-on-one counseling to smokers is health teaching at the individual-level of practice. Advocating for increased taxes on tobacco products occurs at the systems-level of practice, but it is not health teaching.

2. In an effort to decrease health disparities and improve life expectancy, the Social Security Act was amended in 1998 to provide federal funding to: a. assure access to health care for elderly Americans. b. build hospitals to care for the medically indigent. c. insure children without health insurance. d. provide supplementary income for citizens with disabilities.

ANS: C Title XXI of the Social Security Act, passed in 1998, established the State Children's Health Insurance Program to provide funds to insure currently uninsured children. Legislation enacted subsequently provided for new outreach and case-finding efforts to enroll eligible children in Medicaid.

When using the principles of virtue ethics in decision making, a nurse would: a. Provide efficient and effective nursing care. b. Identify the meaningful facts in the situation. c. Seek ethical community support to enhance character development. d. Plan ways to restructure the social practices that oppress women.

ANS: C According to Aristotle, virtues are acquired and include interest in the concept of the good, including benevolence, compassion, trustworthiness, and integrity. One part of the process is seeking ethical community support to enhance character development.

A client eats a nutritious, balanced diet on a daily basis to keep the current state of health. This is described as _____ behavior. a. Illness prevention b. Health promotion c. Health maintenance d. Health protective

ANS: C Health maintenance behavior is directed toward keeping a current state of health.

18. A school nurse is demonstrating the use of a peak flow meter to help children with chronic asthma recognize when they need to use a rescue inhaler. Which of the following levels of prevention is being used by the nurse? a. Primary b. Secondary c. Tertiary d. Both primary and secondary

ANS: C Tertiary prevention includes caring for children with long-term health needs, including asthma and disabling conditions. The nurse is teaching disease management (i.e., when to use an inhaler).

A community that demonstrates commitment would most likely support the development of a: a. Community club to facilitate community involvement b. Childrens recreation program c. Curb-side recycling program and community-based education about recycling d. Cooperative agreement with a neighboring city to share needed services

ANS: C The commitment condition of community competence defined by Cottrell refers to the affective and cognitive attachment to a community, that is worthy of substantial effort to sustain and enhance. Creating a recycling program as well as associated education enhances both the cognitive and affective domains of the community.

The greatest single source of air pollution in the United States is from: a. Waste incineration b. Power plants c. Motor vehicles d. Molds

ANS: C Waste incineration and power plants are major contributors after motor vehicles. Molds contribute to poor indoor air quality.

The growing multiculturalism of American society can contribute to ethnicity conflicts when: a. Cultural standards are congruent with professional standards. b. Cultural traditions within an ethnic group align with those of the community. c. Ethnic groups overburden the health care system. d. The greater communitys values are jeopardized by specific ethnic values.

ANS: D Callahan offered perspectives on judging diversity and suggests a thoughtful tolerance and some degree of moral persuasion (not coercion) for ethnic groups to alter values so that they are more in keeping with what is normative in American culture.

How have nurses historically learned to identify a possible relationship between environmental chemical exposures and their potential harm? a. Extrapolation by toxicologists b. Biomonitoring c. Completing chemistry courses d. Observing signs and symptoms in clients

ANS: D Nurses have historically made discoveries related to chemical exposure when people presented with signs and symptoms related to known chemical toxicity. The first two options are modern methods.

An established group requests a teaching and learning session on hypertension. What can the nurse expect with this type of group? a. The group membership will change from week to week. b. The members all have the same interests. c. They prefer lectures rather than demonstrations. d. The group already has operating methods that have been successful.

ANS: D Nurses working with established groups should know that this type of group has membership ties and an existing structure that has proven to be successful.

18. Which of the following best describes when the home health nurse must document required Outcome and Assessment Information Set (OASIS-B1) data? a. Before any episode of hospitalization b. After each home health visit c. For all incidences of error or mistake in care d. On first admission to home health care

ANS: D OASIS-B1 data are measured and reported to CMS (1) on admission to home health care, (2) after an episode of hospitalization, (3) at the time of recertification, and (4) on discharge from care or death at home. Data are submitted by each agency to a national databank, and agencies receive both results and comparisons with similar agencies to determine areas needing improvement. The data reported from OASIS determine the payment received by the home health agency for the client's total episode of care.

A public health nurse is organizing a multidisciplinary team to address the issue of water pollution in the community. The most likely members that would be invited to address this issue would be: a. Physicians, water sanitation workers, and occupational therapists b. Pharmacologists, radiologists, and epidemiologists c. Nurse practitioners, pharmacologists, and environmentalists d. Geologists, meteorologists, and chemists

ANS: D Scientists who study how pollutants travel in air, water, and soil are geologists, meteorologists, and chemists.

Which community-based risk-reduction intervention resulted in equally favorable health risk changes for both control and treatment groups? a. Framingham Heart Study b. Stanford Heart Disease Prevention Program c. North Karelia Study d. Minnesota Heart Health Program

ANS: D The Minnesota Heart Health Program resulted in equally favorable health risk changes for both control and treatment groups.

When completing a community assets assessment, the community health nurse would gather information about which of the following? A) Key informants B) Strengths of the community C) Specific problem of the community D) Overall life of the community

B Feedback: An assets assessment focuses on the strengths and capacities of a community rather than its problems. Key informants are important sources of information with a comprehensive assessment. Information about a specific health problem is gathered during a problem-oriented assessment. Information about an overall view of the community and its life is gathered with a familiarization or windshield survey.

A nurse is using the technique of motivational interviewing when working with a client. Which of the following statements by the client indicates the client is ready to make the change? a. "I should change." b. "I am willing to change." c. "It's important to change." d. "I want to change."

B

Which of the following would a community health nurse use when conducting an outcome evaluation? A) Organization B) Family progress C) Timing D) Performance

B Feedback: Outcome evaluation involves determining the change in the family's health status or progress. Organization and timing are components of the structure-process evaluation. Performance is part of self- evaluation.

A community health nurse is compiling a list of organizations that serve the needs of persons with disabilities. Which of the following would the nurse categorize as a federal agency? A)National Organization on Disability B)National Council on Disability C)National Association of the Deaf D)American Foundation for the Blind

B Feedback: The National Council on Disability is an independent federal agency. The National Organization on Disability, National Association of the Deaf, and American Foundation for the Blind are private organizations.

When developing programs to assist individuals with disability and chronic illness, which of the following would be most important? A)Narrow focus B)Holistic practice C)Orientation to the present D)Episodic care

B Feedback: The role of the community health nurse with respect to disabilities and chronic illness requires a holistic practice and broad focus. Strong and sustained efforts are needed to achieve results. The community health nurse addresses current needs and plans for the future.

A client who is gay tells the nurse that he has disclosed his sexual preference to a few close friends, but has not come out. How does the nurse interpret this statement? a. This client is confused as to the appropriate terms used for telling others. b. Once you have disclosed, coming out is inevitable. c. The majority of the client's social contacts and family are unaware that he is gay. d. Only the client's friends know he is gay; his family does not.

C Correct: Coming out involves openly living as a gay, lesbian, bisexual, or transgender individual.

A patient who is gay reports that he does not use a condom with his main partner but always uses one when he "goes clubbing." How does the population health nurse document this information? a. Syndemia b. Cisgendering c. Serosorting d. Siliconizing

C Correct: Serosorting is the practice of limiting sexual partners to persons of the same HIV status or use of condoms with HIV discordant or unknown partners.

A community health nurse is collecting data about the family's demographics. Which of the following would the nurse include? Select all that apply. A) Dietary patterns B) Housing C) Climate D) Socioeconomic status E) Education of members F) Ethnicity

DEF Family demographics refer to such descriptive variables as a family's composition, its socioeconomic status, and the ages, education, occupation, ethnicity, and religious affiliations of members. Dietary patterns, housing, and climate are variables assessed with the physical environment.

Which of the following things does a nurse who is traveling by automobile to make a home visit need? Select all that apply. A) A full gas tank and well-operating vehicle B) A map that includes the geographic location where the home visit will be made C) A cellular telephone D) A bus schedule E) Exact change Ans:

A, B, C Feedback: A nurse who is traveling by automobile to make a home visit needs the following: a full gas tank and well-operating vehicle, a map that includes the geographic location where the home visit will be made, and a cellular telephone. If the nurse is not using public transportation, the nurse does not need exact change for each bus trip or a bus schedule.

A 66-year-old woman is retired and no longer has health insurance through her place of employment. Which of the following programs would be appropriate for her health insurance needs? 1. Medicare 2. Medicaid 3. Social Security 4. Economic Opportunity Act

ANS: 1 The Social Security Act was amended to include health insurance benefits for the elderly, which is addressed through Medicare.

Which statement about education is true? a. It emphasizes the provider of knowledge and skills. b. It emphasizes the recipient of knowledge and skills. c. It is a process of gaining knowledge and expertise. d. It results in behavioral change.

ANS: A Education is the establishment and arrangement of events to facilitate learning.

Which core function supports the belief that all Americans should receive basic health care services? a. Assessment b. Assurance c. Policy development d. Advocacy

ANS: B Assurance purports that all persons should receive essential personal health services.

Screening school-age children for hearing deficits is an example of: a. Primary prevention b. Secondary prevention c. Tertiary prevention d. Health promotion

ANS: B Secondary prevention focuses on early detection and prompt treatment of disease, injury, or disability.

A nurse who reads the local community newspaper is using which method to gather data about the community? a. Informant interview b. Survey c. Participant observation d. Windshield survey

ANS: C The nurse is deliberately sharing in the life of the community.

Which Healthy People 2020 goal that has been established has been met? A)The number of infants being put to sleep on their backs and the rate for sudden infant death syndrome B)Improving the proportion of infants who are breast-fed C)Reduce the rate of fetal and infant deaths D)Reduce the occurrence of fetal alcohol syndrome

Ans: A Feedback: After years of working toward improving maternal-child health, the United States has made limited progress. One objective, however, has been met

The nurse gives a very informative and engaging presentation and then gives everyone in the audience a handout that outlines the presentation. Later, the nurse discovers that many of the handouts were thrown away before the audience left the building. Which of the following educational principles has the nurse forgotten? a. Audiences expect PowerPoint or video presentations, not lectures. b. Many Americans do not have a high reading level. c. People want photographs and images, not wordy outlines. d. The nurse gave them too much information too fast for them to want to cope with it all.

B

A community health nurse is involved in a screening program for lead exposure. The nurse would target this program primarily to which age group? A) Birth to -year-olds B) - to 3-year-olds C) 3- to 5-year-olds D) 5- to 7-year-olds

B Feedback: The critical age of lead exposure (or peak level) is thought to be between 18 and 36 months. Levels generally begin to decline after age 3.

When building a coalition, which of the following would the community health nurse do first? A) Conduct a community assessment B) Identify key players C) Identify potential members D) Define goals and objectives

D Feedback: Steps to coalition building include defining goals and objectives, conducting a community assessment, identifying key players or leaders, and identifying potential coalition members.

A home health nurse visits a client who was referred following hospitalization for severe malnutrition related to gastroesophageal reflux disease (GERD) and a large hiatal hernia. Resolution measure? The client: a. Related the importance of taking a daily vitamin. b. Gained two pounds in the past month. c. Will relate the importance of maintaining diet high in nutrient quality. d. Can state the side effects of her medications.

Gained two pounds in the past month. Correct: A resolution measure focuses on the degree to which an existing problem has been resolved.

A home health nurse is working with a client who has several open wounds. Soiled bandages from dressing changes, and equipment used for this client should be: a. Disposed of in the same manner as the client's garbage. b. Brought to the local landfill so as not to mix with the client's regular waste. c. Handled in the same manner as if the client were in a clinical setting. d. Handled as regular garbage, but marked as "Hazardous Waste."

Handled in the same manner as if the client were in a clinical setting. Correct: Rationale: The nurse should adhere to the agency's standards of practice, incorporate universal precautions for preventing the spread of blood borne diseases, and educate clients and family members in infection control measures, including universal precautions.

A referral to the home health agency has been made. As the nurse makes plans to see the client, several steps need to be accomplished. Choose the nurse's initial step in this process. a. Prioritize client needs b. Implement the visit c. Make a preliminary assessment d. Make nursing hypotheses

Make a preliminary assessment Correct: Before the home visit, the nurse conducts a preliminary assessment to review existing information about the client and his or her situation.

A patient has been hospitalized with a femur fracture that has not healed as expected. As discharge comes closer the family inquires about the possibility of home health care visits. What information should the nurse provide? (Select all that apply.) a. The client must first pass the OASIS screen before home health visits are certified. b. The client must first pass HHRG screening before home health visits are certified. c. Qualification requires that the client is homebound. d. Since the client requires physical therapy placement in a long term care facility is necessary. e. The client must be eligible for Social Security benefits to be eligible for Medicare reimbursement of services.

Qualification requires that the client is homebound. Correct: One of the requirements of home health care is that the client is homebound. The client must be eligible for Social Security benefits to be eligible for Medicare reimbursement of services. Correct: Social Security benefits are required in order for the client to be eligible for Medicare.

A home health nurse is completing an initial visit to a client. Which information is the most important to document? a. The objectives developed by the nurse prior to the visit b. The client's physical assessment c. Referrals made to outside agencies d. Discharge summary

The client's physical assessment Correct: The nurse should document the actual (not preliminary) assessment of client health status and the health needs identified as well as the interventions employed to address these needs.

A home health nurse is evaluating the effectiveness of the visits to a particular client. Which statement reflects evaluation of a health promotion/illness prevention measure? a. The client's mobility and range of motion measurements after physical therapy sessions. b. The number of times the client attended health education classes. c. The client's blood pressure measurements after initiating treatment for hypertension. d. The client's weight after nutrition therapy was instituted.

The number of times the client attended health education classes. Correct: Evaluative criteria for health promotion/illness prevention measures reflect health promotion or the absence of specific health problems. Monitoring the number of times the client attended health education classes would indicate that the client's interest in activities that promote health or healthy lifestyles.

Immunization for measles is an example of: a. Primary prevention b. Secondary prevention c. Tertiary prevention d. Health promotion

a. Primary prevention

The concept of "health for all by the year 2000" was initially introduced by which international organization? a. World Health Organization (WHO) b. Centers for Disease Control (CDC) c. United Nations International Children Emergency Fund (UNICEF) d. National Institutes of Health (NIH) Healthy People 2000

a. World Health Organization (WHO) The WHO introduced the goal of "health for all." The CDC strives to prevent and control infectious and chronic diseases, injuries, workplace hazards, disabilities, and environmental health threats. UNICEF focuses on child and women's health.

2. During a home visit, a case manager for a community health center notes marked pitting edema, shortness of breath, and increased fatigue in a 52-year-old male client who lives alone. The client is admitted to the hospital, where he is diagnosed with congestive heart failure. The case manager works with the hospital's utilization manager to devise a discharge plan. The case manager's most logical next step would be to: a. assess the client, obtain information on the scope of services covered by the benefit plan for the client if needed services are not covered, seek to identify and arrange for the resources to provide these services. b. call the client, reintroduce himself or herself, and explain his or her role as a case manager for homebound clients. c. discuss with the family their schedule of availability to offer care in the client's home ensure that the client has daily visits by family members. d. investigate the availability of local support and rehabilitation services for clients with congestive heart failure contact the client's family.

a. assess the client, obtain information on the scope of services covered by the benefit plan for the client if needed services are not covered, seek to identify and arrange for the resources to provide these services.

8. Congress passed the Balanced Budget Act of 1997 with provisions intended to ensure the appropriateness of home health services for those who received them; however, the act may have increased health disparities for vulnerable populations such as: a. frail older adults. b. low-income families with newborns. c. poor clients discharged from acute care. d. clients requiring intravenous antibiotics.

a. frail older adults.

A CHN presents a proposal for a program for preventing teen pregnancy to a group of parents. In the discussion that follows the presentation, which statement by a parent indicates the need for additional teaching by the CHN? a. "I do not know if my son is sexually active however, I have decided that I am going to talk to him about birth control, just in case." b. "I have found that being very strict and checking on my daughter whenever she is out are the best ways to prevent trouble." c. "I plan to sit down with my daughter and have an honest talk about sexuality and potential risks." d. "I will start spending more time with my teens when I get home from work."

b. "I have found that being very strict and checking on my daughter whenever she is out are the best ways to prevent trouble."

Which is an example of an epidemic? a. "Bird" flu in China b. Adult obesity in the United States c. An isolated case of smallpox in Africa d. The nursing shortage in the United States

b. Adult obesity in the United States

Which is an example of a social determinant of health? a. Ethnicity b. Income c. Gender d. Marital status

b. Income

Screening for hearing defects is an example of: a. Primary prevention b. Secondary prevention c. Tertiary prevention d. Health promotion

b. Secondary prevention

If a bioterrorism attack was to occur in the United States, which program would be used to provide large quantities of medication to the American public? a. Cities Readiness Initiative b. Strategic National Stockpile (SNS) c. Public Health Information Network (PHIN) d. Project BioShield

b. Strategic National Stockpile (SNS)

Upholding a client's right to make a choice and to act on the choice involves: a. Affirming b. Supporting c. Informing d. Decision making

b. Supporting

A nurse is told that a screening test has high specificity. This means that the test: a. Provides precise and consistent readings b. Accurately identifies those with the condition or trait c. Accurately identifies those without the trait d. Has a high level of false positives

c. Accurately identifies those without the trait

A population is best defined as a: a.High-risk group b.School or institutional setting c.Collection of individuals who share at least one common characteristic d.Geographical location within a community

c. Collection of individuals who share at least one common characteristic

What is the primary cause of vulnerability? a. Race b. Age c. Poverty d. Illness

c. Poverty

Which of following factors is not necessary to consider when determining priorities for health problem interventions? a. The aggregate's preferences b. The number of individuals affected by the health problem c. The nurse's preferences d. The availability resources

c. The nurse's preferences Factors that assist the nurse in determining priorities for aggregate health care include the aggregate's preferences

When working with a client, when would it be most appropriate for a nurse to use life care planning? a. When organizing a timeline of life events b. When documenting client information and requests c. When assessing present and future client needs d. When estimating future costs for medical care

c. When assessing present and future client needs

A 17-year-old pregnant cocaine addict who is homeless is considered: a. At risk b. A special population c. A Healthy People 2020 target group d. A vulnerable individual

d. A vulnerable individual

Which statement about community health nursing practice is correct? a.It focuses on the delivery of personal health services to individuals and families. b.It provides care to protect the health of the community as a whole. c.It emphasizes the setting where care is provided for clients and families. d.It requires a baccalaureate preparation for practice.

a. It focuses on the delivery of personal health services to individuals and families.

Public health nursing is a specialty because: a.It has a distinct focus and scope of practice. b.It must be done by a registered nurse with a master's degree. c.It is focused on disadvantaged citizens. d.It performs interventions at the acute care level.

a. It has a distinct focus and scope of practice.

A community health nurse is providing a class for pregnant women about the dangers of alcohol consumption during pregnancy. Which of the following would the nurse include as an effect on the newborn? Select all that apply. A)Intellectual impairment B)Low-birth-weight C)Leukemia D)Respiratory distress E)Altered growth and development F)Childhood cancers

ABE Feedback: The health of infants can be dramatically affected by maternal consumption of alcohol. The pregnancy itself can be threatened. However, the most devastating consequence of alcohol consumption during pregnancy is fetal alcohol spectrum disorders (FASD) and fetal alcohol syndrome (FAS), both of which compromise the intellectual functioning, birth weight, and altered growth and development of the infant or child. Respiratory distress, leukemia, and other childhood cancers are not associated with maternal alcohol consumption.

Which of the following trends in health issues in the United States between 1900 and 1955 is accurate? 1. There was a rise in chronic disease such as heart disease and cancer. 2. There was a rise in communicable disease. 3. The crude mortality rate increased dramatically. 4. The life span after diagnosis remained the same.

ANS: 1 Leading causes of death in 1955 were heart disease and cancer, while in 1900 they were pneumonia and tuberculosis. All other answers are false.

Which of the following programs provided funds for neighborhood health centers, Head Start, and other community action programs? 1. Medicare 2. Medicaid 3. Social Security 4. Economic Opportunity Act

ANS: 4 Funding for neighborhood health centers, Head Start, and other community action programs began in 1964 with the Economic Opportunity Act.

7. A nurse planning a smoking cessation clinic for adolescents in the local middle schools and high schools is providing: a. community-oriented care. b. community-based care. c. secondary care. d. tertiary care.

ANS: A Community-oriented nurses emphasize health promotion, health maintenance, and disease prevention, as well as self-reliance on the part of clients. Regardless of whether the client is a person, family, or group, the goal is to promote health through education about prevailing health problems, proper nutrition, beneficial forms of exercise, and environmental factors such as the safety of food, water, air, and buildings.

A nurse established an ongoing group meeting of teenagers with diabetes. In the early stages, the nurse was very directive in arranging location, providing low-carbohydrate drinks and snacks, steering the discussion, and trying to meet all the teenagers' needs. After the group had been meeting for about 3 months, the nurse noticed that the group members no longer simply accepted everything the nurse suggested. Instead, the teenagers began making decisions themselves, and eventually, the nurse no longer controlled the group. Which of the following most likely happened to cause this shift? a. The group became cohesive enough to share leadership tasks. b. Teenagers don't like feeling dependent on adults with power. c. Teenagers often rebel against adult authority. d. The nurse was overwhelmed and lost control of task process.

A

3. A community-oriented nurse is writing a grant application for funding for a nurse-run clinic serving clients with chronic illnesses. The grant application asks for information regarding program benefits, effectiveness, and efficiency. The most effective tool to obtain this information would be: a. cost studies. b. Mobilizing for Action through Planning and Partnerships (MAPP) model. c. Planning Approach to Community Health (PATCH) method. d. Tracer method.

ANS: A Cost studies are essential to show the value of nursing in the marketplace now and in the future. All cost studies involve three major tasks: financial, research, and statistical. The financial tasks include identifying total program costs. The statistical tasks include identifying appropriate, quantifiable measures for analyzing data. The research tasks include setting up an appropriate study design to answer questions about benefit, efficiency, or effectiveness. Types of cost studies include cost-accounting, cost-benefit, cost-effectiveness, and cost-efficiency studies.

The community health nurse is determining the health of a community by examining status. Which of the following would the nurse examine? A) Leading causes of death and illness B) Adequacy of health services C) Socioeconomic distributions D) Strengths of the community

A Feedback: Status typically comprises morbidity and mortality data identifying the physical, emotional, and social determinants of health. Physical and social indices include vital statistics, leading causes of death and illness, suicide rates, and rates of drug and alcohol addiction. Social determinants can be identified by crime rates and functional ability level, or by high school dropout rates or average income levels. Adequacy of health services and socioeconomic distributions reflect structure. Strengths of the community involve process.

A community health nurse is engaged in evaluation of a health plan using a formative evaluation. The nurse would focus on which of the following? A) Process during the actual intervention B) Outcomes of the interventions C) Development of performance standards D) Impact on the clients' health

A Feedback: The focus of formative evaluation is on process during the actual interventions. It uses performance standards that are developed to determine what is working and not working. Summative evaluation focuses on the outcome of the interventions and examines the programs' impact on clients' health.

Which of the following are the three areas has Healthy People identified for public health action for the year 2020 using the International Classification of Functioning, Disability, and Health? Select all that apply. A)Improve the conditions of life for persons with disabilities B)Address the inequitable distribution of resources among people with disabilities and those without disabilities C)Expand the knowledge base and raise awareness about determinants of health for people with disabilities D)Ensure the health of people with disabilities by influencing many social and physical factors E)Public health activities need to focus only on preventing disability

A, B, C Feedback: The three areas that Healthy People has identified for public health action for the year 2020 using the International Classification of Functioning, Disability, and Health include improve the conditions of life for persons with disabilities

A client is self-employed as a mechanic and has limited health insurance coverage. This client is considered: a. Poor b. Near poor c. Medically indigent d. Uninsured

c. Medically indigent

A nurse operates a school-based clinic in a local school, making it easier for children to access health care. This is an example of: a. Case finding b. Wrap-around services c. Outreach d. Comprehensive services

c. Outreach

A screening for diabetes revealed 20 previously diagnosed diabetics and 10 probable new cases, which were later confirmed, for a total of 30 cases. This is called: a. Prevalence b. Incidence c. Attack d. Morbidity

a. Prevalence

As a result of an outbreak of influenza in a community, a nurse encourages members of the community to receive the influenza vaccine. Which level of prevention is being used? a. Primary prevention b. Secondary prevention c. Tertiary prevention d. Multifactorial prevention

a. Primary prevention

What action should a nurse take in order to reduce the risk of liability? Select all that apply. a. Provide accurate documentation of client visits. b. Inform clients of their rights of appeal. c. Refer clients to the providers of their choice. d. Utilize care planning whenever possible.

a. Provide accurate documentation of client visits. b. Inform clients of their rights of appeal.

An example of tertiary prevention is: a. Rehabilitative job training b. Parenting education c. Testicular self-examination d. Family counseling

a. Rehabilitative job training

What is implied by the web of causation model? a. Variables interact resulting in higher probability of illness. b. One disease causes another, especially in vulnerable populations. c. The greater the poverty, the more likely people are to have diseases. d. Immunizations are necessary because vulnerable populations spread disease.

a. Variables interact resulting in higher probability of illness.

The rationale behind the Affordable Care Act was to a. put individuals, families, and small business owners in control of their health care. b. increase federal aid to states for hospital facilities. c. improve the quality of care in rural areas and introduce systematic statewide health care planning. d. force the implementation of health maintenance organizations.

a. put individuals, families, and small business owners in control of their health care. The act put individuals, families, and small business owners in control of their health care. The Hill-Burton Act provided federal aid to states for hospital facilities, resulting in improved quality of care in rural areas and systematic statewide planning. Implementation of health maintenance organizations was not required by the Affordable Care Act.

By the middle of the 21st century, the world population is expected to a. reach almost 10 billion. b. reach 8 billion. c. be largely unchanged from the 20th century. d. show a slight decline.

a. reach almost 10 billion. The world's population soared to 4 billion between 1960 and 1974 and then to 5 billion between 1974 and 1987. In 1999, the world population was 6 billion

The nurse is involved in a conflict resolution situation with the parents of a 2-year-old boy. The parents are deciding if "spanking" the child is a disciplinary method that they will employ. The mother says, "I do not believe in spanking. I see it as abusive and demeaning." The mother's statement is an example of: a. Cooperation b. Assertiveness c. Bargaining d. Collaboration

b. Assertiveness

When public health nurses conduct an assessment of a community's health, they: a. Define one problem that will be the focus for a year. b. Assess a social network of interacting individuals usually in a defined territory. c. Minimize the effects of health risks and hazards. d. Intervene at the population-level by changing laws and regulations.

b. Assess a social network of interacting individuals usually in a defined territory.

Public health practitioners prepared at the graduate level should be able to: a.Teach public and community health nursing b.Assess and intervene successfully at the aggregate level c.Diagnose and treat disease and have prescriptive authority d.Run for political office as experts in public health policy

b. Assess and intervene successfully at the aggregate level

Which activity would a nurse engage in when using population management? a. Census taking to determine the total number of people in the population b. Assessing the needs of the client population through compilation of health histories c. Providing case management services for every citizen in the community d. Selecting programs for wellness that are repeated annually

b. Assessing the needs of the client population through compilation of health histories

The nurse who compares the rate of teenage pregnancy in various areas of the city is practicing the public health core function of: a.Assurance b.Assessment c.Prevention d.Policy development

b. Assessment

Which task would a nurse be most likely to perform while volunteering on a disaster medical assistant team? a. Set up immunization clinics. b. Assist in triaging disaster victims. c. Provide all the medical care for disaster victims. d. Complete a needs assessment of the community.

b. Assist in triaging disaster victims.

The necessary basic preparation for public health nursing is a(n) _____ in nursing. a.Associate's degree b.Baccalaureate degree c.Master's degree d.PhD

b. Baccalaureate degree

An enduring process in which a manager establishes systems and monitors the health status, resources, and outcomes for a targeted aggregate of the population is called: a. Case management b. Care management c. Advocacy d. Continuity of care

b. Care management

The nurse and client, a 20-year-old expectant mother, are discussing the advantages and disadvantages of breastfeeding. To enhance understanding of the implications of breastfeeding, the nurse says, "Tell me more about how you will work full time and breastfeed." Asking this question is an example of which aspect of the advocacy process? a. Verification b. Clarification c. Amplification d. Affirmation

b. Clarification

To better address emerging public health issues, a public health nurse enrolls in a course addressing which content area? a.Leadership b.Ethics c.Communication d.Finance

b. Ethics

The differential vulnerability hypothesis refers to the: a. Resistance of certain groups to risk factors b. Increased susceptibility to cumulative risk factors among vulnerable groups c. Variability in the effects of stressors according to socioeconomic status d. Increased sensitivity of the very young and the very old to risk factors

b. Increased susceptibility to cumulative risk factors among vulnerable groups

What is anticipated to happen in the future with disaster management? a. Vague and unorganized future because constant emergence of new disasters b. Increasing sophistication in technology and surveillance c. Lack of involvement by national and state officials d. Decreasing need for public health workers to provide care

b. Increasing sophistication in technology and surveillance

When case managers provide a formal communication link among all parties concerning the plan of care management, they are assuming the role of: a. Facilitator b. Liaison c. Coordinator d. Negotiator

b. Liaison

Which are examples of knowledge domains used in case management? a. Legal issues, malpractice recognition, and community involvement b. Teaching, counseling, and education skills c. Advocacy, political campaigning and legislative change d. Grant application, bargaining contracts, and securing funding

b. Teaching, counseling, and education skills

The use of weapons of mass destruction or natural disasters, such as earthquakes, will often have more casualties because: a. They cause the most widespread destruction. b. Victims have little time to make evacuation preparations. c. Those with chronic conditions cannot escape in time. d. The early warning systems are not effective.

b. Victims have little time to make evacuation preparations.

A public health nurse implements the public health intervention of health teaching at the systems-level of practice by: a. Participating in the "Great American Smokeout" b. Working with a local employer to provide smoking cessation education c. Providing one-on-one counseling to smokers d. Advocating for increased taxes on tobacco products

b. Working with a local employer to provide smoking cessation education

What is the first action a nurse should take when assessing persons arriving at a shelter following a disaster? a. Limit the amount of equipment and medications brought into the shelter. b. Determine if the person has a psychological condition requiring special attention. c. Assess whether this type of facility is appropriate for the person. d. Provide medical care for persons as if they were in a hospital.

c. Assess whether this type of facility is appropriate for the person.

Making sure that essential community-oriented health services are available defines which of the core public health functions? a.Policy development b.Assessment c.Assurance d.Scientific knowledge-based care

c. Assurance

Providing for the availability of essential personal health services for people who would otherwise not receive health care defines which public health core function? a.Assessment b.Prevention c.Assurance d.Policy development

c. Assurance

An individual who pursues neither his or her concerns nor another's concerns is using which conflict management behavior? a. Accommodating b. Collaborating c. Avoiding d. Compromising

c. Avoiding

The type of epidemiologic study that is used to describe a group of persons enrolled in a study who share some characteristic of interest and who are followed over a period of time to observe some health outcome is a(n): a. Case control study b. Cross-sectional study c. Cohort study d. Experimental study

c. Cohort study

Which is an example of a natural disaster? a. Transportation accident b. Pollution c. Communicable disease epidemic d. Fire

c. Communicable disease epidemic

Voters have recently decided to have fluoride added to the city water system. Epidemiologists wanting to study the effect of fluoride on dental caries would be conducting a(n): a. Ecological study b. Double-blind study c. Community trial d. Screening

c. Community trial

Mutual benefit with limited loss for everyone is a goal of: a. Negotiating b. Assertiveness c. Conflict management d. Cooperation

c. Conflict management

The public health nurse must participate in the essential services of public health. These include: a.Monitoring health status by completing a community assessment b.Diagnosing and investigating health problems in the world c.Informing, educating, and empowering people about health issues d.Working in law enforcement to regulate health and ensure safety

c. Informing, educating, and empowering people about health issues

The first nurse to establish international links and networks was a. Clara Barton. b. Mary Breckenridge. c. Dorothea Dix. d. Florence Nightingale.

d. Florence Nightingale. Florence Nightingale's legacy serves as the foundation for community health nursing in the global health care arena. She channeled her energy into all aspects of health from the care of wounded soldiers at Scutari to the broad public policies that affected health in her time. The other three women were instrumental primarily for their contributions in the United States.

Which would be considered a serious epidemic of influenza? a. 50 cases b. 100 cases c. 500 cases d. Unable to determine

d. Unable to determine

In which situation would a nurse experience vicarious traumatization? a. When sustaining an injury during the disaster b. When exhausted and unable to adjust to the pace at home c. When frustrated about the response time during the disaster d. When listening to survivors' stories about traumatic events

d. When listening to survivors' stories about traumatic events

Public health nurses who develop and implement local public health policies through partnerships with agencies, organizations, and consumers within the community are using which core public health function? a.Assessment b.Prevention c.Assurance d.Policy development

d.Policy development

One of the primary focuses of improving the health of the American people in the twenty-first century is to address: a.Bioterrorism and global health threats b.Delivery of individual care and hygiene c.The need for increased hospital and acute care d.Chronic disease and disability management

a. Bioterrorism and global health threats

A nurse applies the ethical principle of non-maleficence when: a. Administering medications using the five rights b. Allowing clients to be active participants in their care c. Providing patient privacy when delivering care d. Referring a client to a physical therapist

ANS: A Non-maleficence requires that one do no harm. It requires that health care professionals act according to the standards of due care, always seeking to produce the least amount of harm possible.

A public health nurse uses Assumption 2, Public health nursing practice focuses on populations, to guide practice. Which would be considered a population of interest? a. Healthy school children b. Homeless individuals c. A person recently diagnosed with diabetes d. Teenage parents

ANS: A A population is a collection of individuals who have one or more personal or environmental characteristics in common. Populations of interest are populations that are essentially healthy but could improve factors that promote or protect health

A nurse demonstrates the leadership behavior of reflecting when: a. Providing feedback on how behavior appears to others b. Introducing new topics to the group c. Verifying information through questions and restatement d. Highlighting important points from the discussion

ANS: A Reflecting involves giving feedback on how behavior appears to others. Advising introduces new topics. Clarifying verifies new information. Summarizing highlights the important points.

Which core competency of communication is used by nurses engaged in group work? a. Soliciting input from individuals and organizations b. Using simple language when presenting information c. Asking the group to develop the program of interest d. Presenting material to lay audiences only

ANS: A Soliciting input from individuals and organizations is a communication competency.

In the past, community-oriented nurses have been called: a. District nurses b. Almshouse nurses c. Soldier nurses d.Sisters

ANS: A In the past, community-oriented nurses have been called public health nurses, district nurses, visiting nurses, school nurses, occupational health nurses, and home health nurses.

Which intervention is being performed on the individual level? Select all that apply. a. Teaching well-balanced nutrition to a mother with a malnourished child b. Providing a parent age-specific information for immunizations c. Assessing the community for other cases of malnutrition d. Assessing other families on the same street for similar problems

ANS: A, B Teaching well-balanced nutrition and age-specific information about immunizations are the only interventions that focus on individuals. The other examples relate to families and the community.

When conducting an effective educational program, a nurse would use which basic principle? Select all that apply. a. Use a clear, succinct style. b. Use an active voice. c. Refer to Internet sources. d. Use aids to highlight key points.

ANS: A, B, D Reliable sources must be used, and information found on the Internet is not always reliable.

15. Which of the following should be the minimum requirement for a nurse to be prepared for home health nursing? a. An RN license and a baccalaureate degree in a health-related field b. A baccalaureate degree in nursing and RN licensure c. An associate's degree in nursing and RN licensure d. Eligibility for certification as a home health nurse

ANS: B A baccalaureate degree in nursing should be the minimum requirement for entry into professional practice in any community health setting.

According to Leavell and Clark, primary prevention consists of: a. Health promotion and rehabilitation b. Health promotion and specific protection c. Early diagnosis and prompt treatment d. Health maintenance and early diagnosis

ANS: B Leavell and Clark stated that primary prevention includes health promotion and specific protection from disease.

Which of the following is true about African American nurses in public health? 1.Segregation existed until the 1960s, which made certificate and graduate education more difficult to obtain. 2.Cities were the major areas where they practiced 3. They often belonged to the Frontier Nursing Service. 4.They were recruited heavily as military nurses.

ANS:1 The other statements listed are false.

Which one of the following statements about low-birth-weight and very-low-birth-weight babies is most accurate? A) Low-birth-weight is more common in Whites and Hispanics than African Americans. B) Birth weight is one of the most important predictors of infant mortality. C) The incidence of low-birth-weight in the United States is decreasing in relation to the increase in multiple births. D) Recent trends in very-low-birth-weight babies show that they continue to have morbidity with significant increases in survival.

Ans: B Feedback: Birth weight is one of the most important predictors of infant mortality. Low birth weight is more common in African Americans than in Whites and Hispanics. The overall incidence of low-birth-weight is increasing slightly, which can be explained by increased multiple births.

The nurse manager who makes sure that the staff members who work in a local clinic are competent is demonstrating the public health core function of: a.Assurance b.Assessment c.Prevention d.Policy development

a. Assurance

Which population is most likely to be affected by poverty? a. Women b. Men c. Two-parent families d. Adoptive families

a. Women

Collecting data and monitoring the health status of the population defines which of the core public health functions? a.Assessment b.Prevention c.Assurance d.Policy development

a.Assessment

A set of actions one undertakes on behalf of another is: a. Social justice b. Advocacy c. Linguistically appropriate health care d. A waiver

b. Advocacy

The most important predictor of overall mortality is: a. Race b. Age c. Gender d. Income

b. Age

Population-focused practice focuses on defining the problems or needs of and implementing solutions for: a.Individuals b.Aggregates c.Communities d.Geographical regions

b. Aggregates

The life-threatening parasitic disease that causes nearly 500,000 deaths annually is a. AIDS. b. malaria. c. tuberculosis. d. hepatitis.

b. malaria. Malaria is a life-threatening parasitic disease transmitted by mosquitoes. Malaria causes more than 212 million acute illnesses and 429,000 deaths annually. AIDS is a disease caused by the HIV virus. Tuberculosis is an infectious disease caused by the tubercle bacillus. Hepatitis is inflammation of the liver.

Six characteristics that can help identify a student who may be thinking about drastic violence are:

(1) Venting: having mood swings (2) Vocalizing: threatening others (3) Vandalizing: damaging property (4) Victimizing: seeing himself or herself as a victim (5) Vying: belonging to gangs (6) Viewing: witnessing the abuse of others. By helping to identify students who might be considering school violence, help may be obtained and violent actions may be prevented.

An outbreak of a gastrointestinal illness from a food-borne pathogen is an example of a(n): a. Attack rate b. Point epidemic c. Secular trend d. Event-related cluster

b. Point epidemic

A public health nurse provides a clinic for HIV-positive citizens in the community. This is an example of: a.Primary prevention b.Secondary prevention c.Tertiary prevention d.Policy making

c.Tertiary prevention

A nurse in community health is planning to begin a class to help mothers returning to work better cope with the stresses of multiple roles. The nurse would most likely use: a. an andragogical approach. b. a behavioral approach. c. an operational approach. d. a pedagogical approach.

A

A group of teenagers with diabetes become upset and angry because they cannot agree on whether or not to have meetings during summer vacation. Which of the following should the nurse do to be effective in helping the teenagers resolve the conflict? a. Admit that it is difficult for everyone to agree on everything; then ask whether the group can try to decide how to agree on the issue. b. Assume adult authority and announce that meetings will be suspended until fall. c. Recognize that most of the teenagers want to have the meetings, but two are being stubborn; ask the two deviant members to leave the group. d. Suggest that the group avoid discussing it further but rather think about it over the next week and try to discuss the situation more rationally next week.

A

A neighborhood association group has asked the local nurse in community health for a class on environmental hazards. The nurse in community health has seen good information in the community health text and thinks about getting permission to copy some of the information. The most important thing the nurse should do before using copies of this material is to: a. assess the literacy level of the group. b. see how many plan to attend. c. see whether the group is ready to learn. d. secure a good space for a group meeting.

A

A nurse in community health integrates new slides into a presentation that will be given to a local elementary school group regarding the techniques of proper hand washing. The new slides will repeat essential points during the presentation. This demonstrates the nurse's understanding of what principle? a. Repetition b. Integration c. Participation d. Sequencing

A

When evaluating an educational program, the nurse discovers that only 25% of community members met the learning objectives. Which of the following parts of the program should be improved? a. Educator b. Content c. Learners d. Objectives

A

Which educational method has been shown to be most effective in fostering treatment adherence? a. Internet based education b. In-person counseling c. Telephone counseling d. Self-directed learning

A

Which of the following statements regarding the Health Belief Model is accurate? a. Cues to action are an important component of the model. b. Multiple methods of education should be used when implementing this model. c. The first stage experienced in this model is the pre-contemplation stage. d. To successfully implement this model, ongoing maintenance of the behavior must be considered.

A

A community health nurse visits a local women's health center to provide teaching about the effects of smoking during pregnancy. After teaching the class, the nurse determines that the women have understood the teaching when they identify which of the following as a possible maternal or infant effect? A) Low-birth-weight newborns B) Newborn feeding problems C) Genetic malformations D) Developmental delays

A Feedback: One of the strongest predictors of both LBW and preterm birth is maternal smoking. Other studies have shown that there is a greater incidence of stillbirths, spontaneous abortions, and perinatal mortality in pregnancies in which women smoke. Smoking during pregnancy is not associated with genetic malformations, developmental delays, or feeding problems.

The population nurse is working with an individual who is taking hormone therapy as part of the gender recognition surgery. The nurse would teach the client to increase monitoring for which health problems? (Select all that apply.) a. Heart disease b. Urinary tract infection c. Stroke d. Sexually transmitted infection e. Breast cancer

A & C: increased risk of heart disease and stroke w/ hormone therapy. E: increased risk of BrCA w/ hormone therapy

A patient tells a nurse, "I know all about this already. I read about it on the Internet." The patient then summarizes what he has learned. Because some of this information seems incorrect, what questions would be effective for the nurse to ask? (Select all that apply.) a. Were you able find what you wanted easily on the website? b. Did the website say when the information was updated? c. Did the website state who was responsible for the information? d. Did you enjoy reading the material on the website?

A B C

A nurse is conducting an in-service education session on the Centers for Disease Control and Prevention's (CDC's) updates for preventive services for a group of nurses in community health. The nurse would demonstrate the best understanding of the educational process by integrating which planning strategies? Select all that apply. a. Use of films b. Small group interaction c. Use of games d. Session timing e. Session space

A B C D E

A nurse has been making ongoing visits to a preschool to help the staff teach the children important skills, such as brushing their teeth after meals. Now the nurse is scheduled to teach carbohydrate counting to adults newly diagnosed with diabetes. Which of the following actions will the nurse need to take when teaching this population? (Select all that apply.) a. Appeal to the need for autonomy and choice. b. Emphasize that anyone with diabetes must know this information. c. Explain how to cope with being a guest at a dinner. d. Recognize that this audience will depend on the instructor to set goals for learning.

A C

Which of the following characteristics must a learner possess to successfully master psychomotor learning? (Select all that apply.) a. The necessary ability to perform the skill b. An open mind to learning the new skill c. A sensory image of how to perform the skill d. Opportunities to practice the skill

A C D

Which of the following barriers are specific to the educator? Select all that apply. a. Fear of public speaking b. Low literacy c. Limited experience with the topic d. Lack of motivation e. Lack of experience with gaining participation

A C E

Which of the following are overarching goals for the health of the nation presented in Healthy People 2020? Select all that apply. A) To attain high-quality, longer lives free of preventable disease, disability, injury, and premature death B) To achieve health equity, eliminate disparities, and improve the health of all groups C) To create social and physical environments that promote good health for all D) To promote quality of life, healthy development, and healthy behaviors across all life stages E) Healthy People 2020 does not address the health of the nation but does address the health of individuals.

A, B, C, D Feedback: The four overarching goals for the health of the nation are the following: To attain high-quality, longer lives free of preventable disease, disability, injury, and premature death To achieve health equity, eliminate disparities, and improve the health of all groups To create social and physical environments that promote good health for all To promote quality of life, healthy development, and healthy behaviors across all life stages

Which of the following are specific safety measures the nurse should apply when making a home visit? Select all that apply. A) Plan to reschedule the visit if you find a large group of people assembled between you and the client's door. B) Immediately leave the home when family members begin to physically fight with one another. C) Enter the residence before you determine that the family you are intending to visit does live there and is home. D) Travel only in pairs for all home visits. E) If someone approaches you and indicates that they want your nursing bag, throw your bag away from where you are and run in the opposite direction. F) Leave an itinerary of your planned travels, the telephone numbers of families you will attempt to visit and your cellular phone number at your base of operation.

A, B, E, F Feedback: It is most important to leave an itinerary of your planned travels, the telephone numbers of families you will attempt to visit, and your cellular phone number at your base of operation. Specific safety measures the nurse should apply when making a home visit include planning to reschedule the visit if you find a large group of people assembled between you and the client's door; immediately leaving the home when family members begin to physically fight with one another; and if someone approaches you and indicates that they want your nursing bag, throw your bag away from where you are and run in the opposite direction. It is not appropriate for the nurse to enter the residence before determining that the family that is to be visited does live there and is home. It is not necessary to travel only in pairs for all home visits.

The physician orders trazadone (Desyrel) for Mrs. W, a 78-year-old widow with depression, 150 mg to take at bedtime. Which of the following statements about this medication would be appropriate for the home health nurse to make in teaching Mrs. W about trazadone? A. "You may feel dizzy when you stand up, so go slowly when you get up from sitting or lying down." B. "You must be sure and not eat any chocolate while you are taking this medicine." C. "We will need to draw a sample of blood to send to the lab every month while you are on this medication." D. "If you don't feel better right away with this medicine, the doctor can order a different kind for you."

A. "You may feel dizzy when you stand up, so go slowly when you get up from sitting or lying down."

Which of the following represents a nursing intervention at the tertiary level of prevention? A. Serving as case manager for a mentally ill homeless client B. Leading a support group for newly retired men C. Teaching prepared childbirth classes D. Caring for a depressed widow in the hospital

A. Serving as case manager for a mentally ill homeless client

Which of the following represents a nursing intervention at the primary level of prevention? A. Teaching a class in parent effectiveness training B. Leading a group of adolescents in drug rehabilitation C, Referring a married couple for sex therapy D. Leading a support group for battered women

A. Teaching a class in parent effectiveness training

The steps of the ethical decision making process are similar to the steps of: a. Healthy People 2010 b. Deontology c. The nursing process d. Advocacy

ANS: C The nursing process involves the same basic steps: assessment, diagnosis, planning, implementation, and evaluation.

The American Red Cross, through its Rural Nursing Service, improved living conditions in villages and isolated farms. Which of the following is an example of the resourcefulness of a nurse of this era? 1.Using hot bricks, salt, or sandbags to substitute for hot water bottles 2.Testing well water for pollutants 3.Teaching school and developing curricula for rural nursing programs 4.Providing post-surgical care

ANS:1 In providing medical care, rural nurses were resourceful in finding alternatives when they did not have medical products that were available in urban areas.

The Frontier Nursing Service was established by Mary Breckinridge to emulate systems of care used in the Highlands of Scotland. Her biggest contribution was: 1.Establishment of the Henry Street Settlement 2.Development of health programs geared toward improving the health care of the rural and often inaccessible populations 3.Blazing a nursing trail through the Rockies, providing nursing care to miners and their families 4.Teaching birth control measures to large numbers of women in the South

ANS:2 Mary Breckenridge developed health programs geared toward improving the health care of the rural and often inaccessible populations in the Appalachian regions of southern Kentucky. Lillian Wald established the Henry Street Settlement.

Early colonial health efforts in the United States included: 1.Establishment of schools of nursing 2.Development of vaccines given to large numbers of people 3.Collection of vital statistics, improved sanitation, and control of communicable diseases introduced through seaports 4.Development of public housing and almshouses

ANS:3 The other choices are events that happened after the colonial period.

Occupational health nursing began as industrial nursing. What was the purpose of this type of nursing? 1.Inventing new machines to streamline production of medical goods 2.Investigating industrial injuries to improve work conditions 3.Working at industrial sites treating work related-injuries 4.Providing care for factory workers and their families

ANS:4 Early occupational health nursing did not provide care for work-related injuries, but instead focused on the care of employees and their families in the home.

Lillian Wald was the first public health nurse in the United States. Which of the following is her major contribution to public health nursing? 1. Founding the American Nurses Association 2.Establishment of the New York Training Hospital for Nurses 3.Establishment of the Public Health Service 4.Establishment of the Henry Street Settlement that later became the Visiting Nurse Service of New York

ANS:4 Lillian Wald established the Henry Street Settlement.

A nursing student during World War II would likely join which group? 1.The Public Health Service of New York City 2.The Marine Nurse Corps 3.The Frontier Nursing Service 4.The Cadet Nurse Corps

ANS:4 The Bolton Act of 1943 established the Cadet Nurse Corps during World War II, which increased enrollment in schools of nursing at undergraduate and graduate levels.

A community health nurse is integrating the Healthy People 2010 goals into suggested programs for a local community. Which of the following goals would the nurse identify as having been met? A)Reduction in low-birth-weight infants B)Reduction in total preterm births C)Increase in the number of infants put to sleep on their backs D)Increased proportion of mothers breast- feeding at 6 months

Ans: C Feedback: After years of working toward improving maternal-child health, the United States has made limited progress. One objective, however, has been met: 70% of infants are now being put to sleep on their backs (up from 35% baseline), largely due to public health educational efforts and reminders. Objectives for LBW, VLBW, and preterm births all moved away from their Healthy People 2010 targets. Breast-feeding is beneficial to both mother and infant, and in 2005, almost 73% of infants were breast-fed for some period of time. However, the goal has not been met.

A community health nurse is developing a program that would encourage mothers to use breast-feeding exclusively for their babies' nutrition for the first 6 months. The nurse bases this program on the understanding about which of the following? A)Exclusive breast-feeding promotes better growth and development. B)Its lower cost provides more money for other needed items. C)Infant mortality would significantly decrease. D) It would aid in decreasing maternal mortality rates.

Ans: C Feedback: Approximately 1.3 million infants would survive annually if mothers were able to exclusively breast-feed their infants for 6 months. The lack of breast-feeding is related to decreased survival and increased morbidity from infections, lower intelligence test scores, increased cardiac risk factors, and inadequate nutrition. The superiority of breast milk and lower costs would not be factors for developing a community health program for breast-feeding. Exclusive breast-feeding is not associated with decreasing maternal mortality rates.

Critical population health indicators that affect childbearing women, infant, toddler, and preschool populations globally and in the United States include which of the following? Select all that apply. A)Maternal mortality rate B)Infant mortality rate C)HIV/AIDS D)Crude mortality rate E)Fetal alcohol syndrome

Ans:A, B, C, E Feedback: Critical population health indicators that affect childbearing women, infant, toddler, and preschool populations globally and in the United States include maternal mortality rate, infant mortality rate, HIV/AIDS, and fetal alcohol syndrome. Crude mortality rate does not apply specifically to childbearing women, infant, toddler, and preschool populations.

A nurse planning a smoking cessation clinic for adolescents in the local middle schools and high schools is providing: A. Community-oriented care B. Community-based care C. Secondary care D. Tertiary care

Answer: A - Community-oriented nurses emphasize health protection, maintenance, and promotion and disease prevention, as well as self-reliance among clients. Regardless of whether the client is a person, a family, or a group, the goal is to promote health through education about prevailing health problems, proper nutrition, beneficial forms of exercise, and environmental factors such as safe food, water, air, and buildings.

A clinic treating a child for otitis media is an example of: A. Community-based care B. Public health care C. Tertiary health care D. Community-oriented care

Answer: A - In community-based nursing, the nurse focuses on "illness care" of individuals and families across the life span. The aim is to manage acute and chronic health conditions in the community, and the practice is family-centered illness care. Community-based nursing is not a specialty in nursing but, rather, a philosophy that guides care in all nursing specialties.

A state agency has received multiple complaints regarding the availability of elder transportation services to a specific county senior center. The state agency assigns a public health nurse to work with the community to evaluate its program for elder transportation services to publicly sponsored elder care programs. The public health core function applied is: A. Assurance B. Primary prevention C. Policy development D. Public transportation

Answer: A - The public health core function of assurance focuses on the responsibility of public health agencies to be sure that activities are appropriately carried out to meet public health goals and plans. This role requires skill in assessment, investigative functions, collaboration, consultation, and cooperation. Assurance also includes assisting communities to implement and evaluate plans and projects.

A community-oriented nurse convenes a support group for teenage mothers. The nurse understands that this strategy fosters cohesiveness among members and allows the members to learn from one another. What other benefit specific to group teaching will be achieved? a. Cultural sensitivity b. Efficiency in client service c. Learning of new skills d. Distraction-free surroundings

B

Public health interventions are implemented with: a. Legislators, policy makers, and community leaders b. Individuals and families, communities, and systems c. Children, adolescents, and adults d. Health departments, public health agencies, and visiting nurses associations

b. Individuals and families, communities, and systems

A nursing student prepares a treatment plan for a client that draws upon the student's understanding of pathophysiology and nursing practice theory learned in the student's coursework. This application of new information used in a different way demonstrates the educational principle of: a. affective domain. b. cognitive domain. c. events of instruction. d. principles of effective education.

B

During which stage of the health planning process would the community health nurse create a plan? A) Assessment B) Analysis and design C) Implementation D) Evaluation

B Feedback: During the health planning process, a community health nurse creates a plan during the analysis and design stage, which correlates to the planning phase of the nursing process. Assessment involves data collection, interpretation, and goal setting. Implementation involves how to operationalize the plan and a design for monitoring progress. Evaluation involves examining the costs and benefits, judging the potential outcomes, modifying the plan, presenting the plan to the sponsoring group or agency, and obtaining acceptance.

A community health nurse arrives at a family's home. Which of the following would behaviors by the nurse would be nontherapeutic? A) Maintaining eye contact with the family members B) Paging through paperwork for information C) Sitting on the furniture near the family members D) Placing car keys in the nurse's carryall bag

B Feedback: Community health nurses, like all nurses, need to be aware of their own body language. Fidgeting with car keys during the entire visit, noisily chewing gum, giving minimal eye contact, continuously looking at or paging through paperwork, appearing rushed, and refusing to sit on any of the furniture are behaviors that tell the family a great deal about the nurse, including how he or she feels about being in their home. These actions are nontherapeutic. Sitting on the furniture near family members demonstrates interest in the family. Placing car keys in the carryall bags prevents inadvertent distractions.

A public health nurse is surveying males in a community to get a sense of the overall health risks present. After reviewing the surveys, approximately 12% identified themselves as gay. The nurse should conclude that this information: a. This coincides with the national averages. b. May have mixed meaning. c. Should not be taken seriously as the nurse really does not know what the respondents meant by their answer. d. Means these men are at much greater risk for psychological and social problems.

B Correct: Homosexuality is culturally defined and what is considered homosexual in one culture may not be in another, even if it involves same-sex sexual activity.

Which of the following strategies should be used when providing educational programs for children? (Select all that apply.) a. Emphasize how to build learning from previous experiences. b. Use simple words to enhance understanding. c. Use objects to help increase their attention. d. Emphasize the importance of the long-term consequences.

B C

Which of the following issues motivated the increased doses of certain vaccines and reformulated vaccines at the start of the 21st century? Select all that apply. A) Increased virulence of the vaccine- preventable disease causative organisms B) Pertussis immunity was found to wane with age so the vaccine was reformulated to cocoon infants by immunizing the adults around them to prevent exposure of infants. C) Varicella vaccine given in two doses has been found to increase immunity. D) Measles outbreaks were identified in populations where parents had refused MMR vaccination for their children and received personal belief exemptions.

B, C, D Feedback: The following issues motivated the increased doses of certain vaccines and reformulated vaccines at the start of the 21st century. Pertussis immunity was found to wane with age so the vaccine was reformulated to cocoon infants by immunizing the adults around them to prevent exposure of infants

The community health nurse is invited to speak to a group of students about clients with disabilities. As part of the class, the nurse intends to describe The International Classification of Functioning, Disabilities, and Health (ICF) and its attempts at standardized language. Which of the following would the nurse include as terminology that was standardized? Select all that apply. A)Disability impairment issues B)Body functions C)Handicapping conditions D)Activities E)Environmental factors F)Personal Factors

B, D, E, F Feedback: The ICF included definitions for body functions and structures, activities, and environmental and personal factors. Terms such as disability, impairment, and handicap have been removed from the definition and replaced with functioning, which is more holistic and positive.

Ann is a psychiatric home health nurse. She has just received an order to begin regular visits to Mrs. W, a 78-year-old widow who lives alone. Mrs. W's primary care physician has diagnosed her as depressed. Which of the following criteria would qualify Mrs. W for home health visits? a. Mrs. W never learned to drive and has to depend on others for her transportation. b. Mrs. W is physically too weak to travel without risk of injury. c. Mrs. W refuses to seek assistance as suggested by her physician because "I don't have a psychiatric problem." d. Mrs. W says she would prefer to have home visits than go to the physician's office.

B. Mrs. W is physically too weak to travel without risk of injury.

Which of the following represents a nursing intervention at the secondary level of prevention? A. Teaching a class about menopause to middle-aged women B. Providing support in the emergency room to a rape victim C. Leading a support group for women in transition D. Making monthly visits to the home of a client with schizophrenia to ensure medication compliance

B. Providing support in the emergency room to a rape victim

When the nursing student identifies the concept of community as client as which of the following the instructor knows that the student has grasped this concept? A) The community is comprised of individual clients. B) The community is comprised of families. C) The community is a group or population of people as the focus of nursing service. D) The community cannot be a client. Persons who are hospitalized are patients.

C Feedback: The concept of community as client refers to a group or population of people as the focus of nursing service.

A home health nurse is visiting an established client who was referred following hospitalization for a myocardial infarction. The client lives with a spouse and the nurse is assessing the client's knowledge regarding resuming normal activities. The nurse has been trying unsuccessfully to gain information about the client's resumption of sexual activity, but the client consistently changes the subject to questions about medications. This is an example of a(an): a. Nurse-oriented distraction b. Behavioral distraction c. Environmental distraction d. Physical distraction

Behavioral distraction Correct: Rationale: Behavioral distractions consist of behaviors employed by the client to distract the nurse from the purpose of the visit.

A man says, "I just can't get myself to leave the house and go for a 30-minute walk each day. I wish I could think of some way to motivate myself." Which of the following would be the best action for the nurse to take? a. Establish a written contract between him and his employer that states walking is required each day. b. Recognize the reasons why 30 minutes of walking each day is one of the best health promotion activities he can choose. c. Join a group that walks early each morning. d. Set up rewards for himself, such as a nice snack after he gets back from walking.

C

A nurse is providing an educational program about testicular self-examination (TSE). Which of the following would be the best learning objective for this program? a. Each participant will state why TSE is important and explain how to do it. b. Each participant will describe how to do TSE and discuss the dangers of testicular cancer. c. 90% of the men will correctly demonstrate testicular self-examination. d. 100% of the men will do a testicular examination correctly on a model.

C

A nurse uses lecturing as the primary method of educating clients. Which part of the education process is missing? a. Evaluation b. Experience c. Participation d. Understanding

C

A community health nurse is attempting to empower a family to become independent. Which of the following would be most appropriate? A) Focus attention on the problem areas B) Do for the family what they cannot C) Emphasize the family's strengths D) Reinforce positive traits

C To help empower families, the community health nurse needs to emphasize the family's strengths. It is the nurse's job to recognize the strengths in families and to help families recognize them as well. Focusing on problem areas portrays a negative view and can undermine the nurse's efforts. Doing for the family what they cannot fosters dependency. Reinforcing positive traits may be appropriate if those positive traits are strengths. Not all positive traits are strengths.

A member of a community weight-loss group has maintained a healthy weight for 2 years through healthy eating and daily exercise. Which of the following steps of the affective domain is this participant demonstrating? a. Analysis b. Application c. Evaluation d. Knowledge

C (Steps in the affective domain have the learner doing the following in this sequence: (1) knowledge: receives the information; (2) comprehension: responds to what is being taught; (3) application: values the information; (4) analysis: makes sense of the information; (5) synthesis: organizes the information; (6) evaluation: adopts behaviors consistent with the new values. In this example, the individual has adopted the behavior and this has resulted in the ability to maintain the value of a healthy weight.)

A key opportunity for public health nurses to improve population-focused care is by: a.Assuming traditional nursing roles b.Influencing public health policy c.Conducting community assessments d.Specializing in community-based nursing practice

b. Influencing public health policy

John, a homeless person, has just come to live in the shelter. The shelter nurse is assigned to his care. Which of the following is a priority intervention on the part of the nurse? a. referring John to a social worker b. Developing a plan of care for John c. conducting a behavioral and needs assessment on John d. helping John apply for social security benefits

C. conducting a behavioral and needs assessment on John

A home health nurse receives a referral to educate an older adult client with diabetes in proper foot care. The nurse's assessment of the client determines that the client has poor eyesight that may affect the client's ability to learn and perform certain skills. Which educational principle is being examined? a. Cognitive domain b. Events of instruction c. Principles of effective instruction d. Psychomotor domain

D

A nurse invited all the teenagers who were newly diagnosed with diabetes to a group meeting to discuss issues they all had in common. One teenager replied, "I don't know if I want to share all the problems I'm having with strangers." Which of the following is the best nurse response? a. "Don't share anything with anyone until you're comfortable doing so." b. "I can understand what you are saying." c. "No one will require you to do anything you don't want to do." d. "The purpose of the group is to have a safe place to share problems with others who may be having similar problems."

D

A nurse is meeting to discuss problems and solutions with a group of teenagers who have been newly diagnosed with diabetes. One teenager states, "My mom found this particular brand of popcorn that has only 15 carbohydrates in the whole giant bag." Which of the following best describes the group purpose that is being served by the teenager's statement? a. Maintenance function of encouraging everyone to continue the discussion b. Maintenance function of helping everyone feel comfortable talking about food c. Task function of resolving problems about what to nibble during movies d. Task function of sharing information and resources

D

A nurse is planning an educational program about cardiovascular disease. Which of the following would be the optimal time for the nurse to elicit feedback from program participants? a. At the beginning of the program b. At the program midpoint c. Immediately following program completion d. Throughout the program

D

When developing community diagnoses, which of the following would the community health nurse do? A) Focus primarily on deficits of the community B) Use broad statements to guide a range of interventions C) Employ a revised nursing diagnosis format D) Substitute the terms client, family, group, or aggregate for the word patient

D Feedback: Community diagnoses should include statements about the strengths of the community and possible sources for community solutions, as well as the community's weaknesses or problem areas. In addition, they should portray a community focus, include the community response, and identify any related factors that have potential for change through community health nursing. These may also include wellness diagnoses, which indicate maintenance or potential change responses (due to growth and development), when no deficit is present. Diagnoses statements should be narrow enough to guide interventions and use the standard nursing diagnosis format.

A school nurse is working with a family whose son recently announced that he is gay. The parents were initially angry and now, several weeks later, state to the nurse that this is a fad their son is just "going through." The nurse recognizes this as: a. Acceptance that he is gay. b. A call for help on the parent's behalf. c. A highly irregular occurrence. d. A normal phase of acceptance.

D Correct: Families go through stages similar to the stages of grief. In the denial stage the parents may see their child's professed sexual identity as a "phase" that he or she will grow out of.

The community health nurse staffs a clinic, which serves a large number of homosexual clients. When assessing young lesbian clients, the nurse would want to pay keen attention to physiological changes of which system? a. musculoskeletal b. integumentary c. genitourinary d. respiratory

D Correct: Studies show that smoking is about twice as prevalent in the GLBT population as in the straight population.

Which of the following would be appropriate as secondary prevention level activities focusing on child abuse and neglect? A) Prenatal parent preparation classes B) Public service announcements for positive parenting C) Education to improve family functioning skills D) Respite care for families with special needs children

D Feedback: Providing respite care for families with special needs children is an example of a secondary prevention action. Prenatal parent preparation classes, public service announcements, and education to improve family functioning skills are examples of primary prevention level activities.

Which of the following would be most appropriate for the community health nurse to do first when planning an initial home visit to a family? A) Obtain the basic supplies that will be needed B) Gather appropriate educational materials C) Contact the family via telephone D) Review the referral information

D Feedback: The first step is to obtain the referral and review it to gather information about the possible needs of the family and the reason for the visit. Once this is done, then the nurse would obtain the basic supplies and education materials that might be needed and contact the family to arrange the visit.

Mrs. W, a 78-year-old depressed widow, says to her home health nurse, "What's the use? I don't have anything to live for anymore." Which is the best response on the part of the nurse? a. "Of course you do, Mrs. W. Why would you say such a thing?" b. "You seem so sad. I'm going to do my best to cheer you up." c. "Let's talk about why you are feeling this way." d. "Have you been thinking about harming yourself in any way?"

D. "Have you been thinking about harming yourself in any way?"

John, a homeless person, has a history of schizophrenia and noncompliance with medications. Which of the following medications might be the best choice for John? a. Haldol b. Navane c. Lithium carbonate d. Prolixin decanoate

D. Prolixin decanoate

A hospice client has decided to remain in his own home until death. The client requires 24/7 nursing care and during the first overnight care by hospice nursing staff, they realize that the client's home is infested with cockroaches. The hospice nurse should: a. Help the family make arrangements for pest control measures immediately. b. Refuse to visit the home until the situation is taken care of. c. Tell the client's family that the client does not qualify for hospice services because of the condition in the home. d. Make an immediate referral for social services and move the client to a long-term care facility.

Help the family make arrangements for pest control measures immediately. Correct: The need for balance in the area of risk and safety affects both nurse and client. The nurse must decide what level of risk is acceptable without unduly jeopardizing the safety of the client or the nurse's own safety. The nurse might need to weigh the relative risks of changing a potentially hazardous environment versus the disruption to the client's life that will result from the change. This is a situation that requires action on the nurse's part, but not a situation that is without a solution.

A home health nurse is making a visit to a client whose dwelling does not have running water. The best way for the nurse to practice standard precautions in this case would be to: a. Have the client meet the nurse at the closest health agency. b. Use waterless hand cleaner before, during, and after the visit. c. Make a referral to social services, as this client should not be living in a home without running water. d. Only perform non-invasive procedures or assessments.

Use waterless hand cleaner before, during, and after the visit. Correct: The primary infection control measure in any setting is adequate hand washing before and after giving any direct care to clients. Hands should be thoroughly washed with soap and running water, or a waterless cleanser used if no water is available.

A home health nurse is making an initial visit to a new client. The nurse had developed a preliminary assessment prior to the visit to the client, who was referred after hospitalization for a total hip replacement. The client lives alone. What is the next step in the home visit? The nurse will: a. Evaluate the home for any mobility concerns (i.e., stairs, carpets, uneven sidewalk, etc.). b. Validate the information collected prior to the visit. c. Refer the client to physical therapy. d. Monitor the client's progress in mobility.

Validate the information collected prior to the visit. Correct: The first task in implementing the home visit is to validate the accuracy of the preliminary assessment.

Which diseases contribute to higher mortality rates in developing countries? (Select all that apply.) a. AIDS b. Hepatitis C c. Cancer d. Parasitic infection e. Dengue fever

a, d, e, Among the infectious diseases that contribute to higher mortality rates in developing countries are AIDS, tuberculosis (TB), malaria, hepatitis B, rheumatic heart disease, parasitic infection, and dengue fever. Although these diseases claim the lives of millions, it is estimated they could be reduced by up to 50% through effective public health interventions.

Which organizations offer volunteer opportunities in disaster work for nurses? Select all that apply. a. American Red Cross b. Citizens Corps c. American Nurses Association d. National Public Health Training Centers

a. American Red Cross b. Citizens Corps

A patient with a long history of asthma with many hospital admissions is referred to a case manager to assist in the management of her disease. Disease management targets: a. Chronic and costly disease conditions that require long-term care interventions b. Patients who cannot handle their disease c. Those who seek to control use by providing clients with correct information d. Patients who will need an advanced practice nurse instead of physician for monitoring

a. Chronic and costly disease conditions that require long-term care interventions

The public health nurse working with the chronically ill and families with young children needing age-specific health maintenance is using which model of case management? a. Client-focused b. System-focused c. Social service d. Long-term care

a. Client-focused

Which environmental stressor directly assaults human health? a. Global warming b. Air pollution c. Noise d. Litter

b. Air pollution Air pollution and lead poisoning directly assault human health. Noise and litter affect quality of life. Global warming affects the ecologic balance.

An epidemiologist wanting to know what caused severe diarrhea and vomiting in several people at a local banquet would be using: a. Descriptive epidemiology b. Analytic epidemiology c. Distribution d. Determinants

b. Analytic epidemiology

Which principle of justice is influenced when excessive attention to cost containment impairs the nurse's duty to provide measures to improve health? a. Autonomy b. Beneficence c. Confidentiality d. Justice

b. Beneficence

A public health nurse uses collaboration, coalition building, and community organizing when: a. Providing case management, referral, and follow-up services with individuals b. Carrying out collective action at the systems or community levels of practice c. Conducting a community assessment d. Implementing primary and secondary prevention strategies

b. Carrying out collective action at the systems or community levels of practice

A nurse promotes alliances among organizations for a common purpose. Which public health intervention is being implemented? a. Health teaching b. Coalition building c. Surveillance d. Referral and follow-up

b. Coalition building

In which situation does the nurse need to be aware of the potential for selection bias? a. Determining the population to be studied b. Considering how the participants will enter the study c. Studying cause and effect relationships d. Documenting results of the study

b. Considering how the participants will enter the study

Which vulnerable group would be most sensitive to adverse effects? a. Pregnant teenager living with her parents for financial support b. Poor, older woman with no means of transportation c. Two-year-old boy of underinsured parents d. Recently unemployed father of five

b. Poor, older woman with no means of transportation

The term community-based nursing care is applied when implementing which nursing intervention? a. Assessing the health needs of a defined community b. Providing care to families in a community c.Promoting the health of an entire community d.Investigating environmental health problems in a community

b. Providing care to families in a community

The probability an event will occur within a specified period of time is called: a. Rate b. Risk c. Epidemiology d. Epidemic

b. Risk

Which is an example of an agent in the epidemiologic triangle? a. Human population distribution b. Salmonella c. Genetic susceptibility d. Climate

b. Salmonella

The first stage of collaboration is: a. Trust building b. Consensus c. Collegiality d. Awareness

d. Awareness

An example of an attack rate is the: a. Number of cases of cancer recorded at a medical center b. Number of people who died of cholera in India in a given year c. Number of beef cattle inoculated against mad-cow disease on a farm d. Proportion of people becoming ill after eating at a fast-food restaurant

d. Proportion of people becoming ill after eating at a fast-food restaurant

Which demonstrates the removal of a barrier to health care? a. Discrimination against certain groups b. Treatment of pets at the same facility c. Provision of free food at a food bank d. Providing services for a rural population by using a mobile clinic

d. Providing services for a rural population by using a mobile clinic

Following a disaster, what is the priority of the disaster response? a. Clean up the environment. b. Handle the stress reaction of the victims. c. Bring in as many aid workers and nurses as possible. d. Reestablish sanitary barriers and focus on basic needs.

d. Reestablish sanitary barriers and focus on basic needs.

Which is the most appropriate action for a nurse when planning and implementing care for vulnerable populations? a. Setting up multiple clinics in a wide geographic area b. Advising legal consultants on a variety of issues c. Making laws to protect the homeless d. Teaching vulnerable individuals strategies to prevent illness and promote health

d. Teaching vulnerable individuals strategies to prevent illness and promote health

The interaction between an agent, a host, and the environment is called: a. Natural history of disease b. Risk c. Web of causality d. The epidemiologic triangle

d. The epidemiologic triangle

When assessing a community's needs, which of the following would the nurse most likely complete first? A) Familiarization assessment B) Problem-oriented assessment C) Community subsystem assessment D) Comprehensive assessment

A Feedback: A familiarization assessment is a common starting place in evaluation of a community. It involves studying data already available on a community and then gathering a certain amount of firsthand data in order to gain a working knowledge of the community. A problem-oriented assessment begins with a single problem and assesses the community in terms of that problem. It is commonly used when familiarization is not sufficient and a comprehensive assessment is too expensive. A community subsystem assessment focuses on one dimension of community life. It is a useful way for a team to conduct a more thorough community assessment. A comprehensive assessment seeks to discover all relevant community health information.

When the nursing student identifies the three dimensions of the health of a community, the nurse educator is aware that the student has grasped which of the following concepts? A) Status/people, structure, and process B) Assessment, policy development, and assurance C) Primary, secondary, and tertiary D) Function, structure, and process

A Feedback: The three dimensions of the health of the community are status/people, structure, and process. Assessment, policy development, and assurance are the three core public health functions. Primary, secondary, and tertiary are the levels of prevention. Function, structure, and process are the descriptions of Human Anatomy and Physiology.

As a community health nurse working with lesbian clients, it is important for the nurse to understand that these clients: a. Are at slightly greater risk for mental illness. b. Will always be at risk for depression. c. Have no problems with psychological disorders. d. Are at no greater risk for psychiatric disorders.

A Correct: Although GLBT individuals do not appear to be any more intrinsically susceptible to mental illness than their straight counterparts, there is some evidence to suggest that they have a slightly higher prevalence of mental illness, possible as a result of "minority stress."

What are the roles that public and community health nurses need to enact in order to maximize the outcome for pregnant adolescents? Select all that apply. A) Clinician B) Educator C) Counselor D) Provider of referrals E) Friend

A, B, C, D Feedback: Roles that public and community health nurses need to enact in order to maximize the outcome for pregnant adolescents include clinician, educator, counselor, and provider of referrals. It is not appropriate for the nurse to enact the role of friend with pregnant adolescent clients or any other client.

A community health nurse is applying an interactional framework to assess a family's health. Which of the following would the nurse do? A) View them in terms of the family's internal relationships B) Look at them from a life-cycle perspective C) Evaluate the members' changing roles and tasks D) Assess them as a social system relating to other social systems

A Feedback: An interactional framework describes the family as a unit of interacting personalities and emphasizes communication, roles, conflict, coping patterns, and decision-making processes. A developmental framework studies the family from a life-cycle perspective by examining the members' changing roles and tasks in each progressive stage. A structural-functional framework describes the family as a social system relating to other social systems in the environment.

A community health nurse is working with a group of parents who have infants and toddlers. Which of the following would the nurse include in a teaching plan for this group about reducing dental caries? A) Serving sugary foods at mealtimes and not snacks B) Providing juice in a sippy cup between meals C) Allowing the older infant to fall asleep with a bottle D) Permitting snacking throughout the day

A Feedback: It is recommended that sugary foods be eaten at mealtimes and not as snacks, and that regular snack times be established. Between- meal drinks should consist of water or milk. Juice should be given only with meals. Allowing infants to fall asleep with a bottle can lead to baby bottle tooth decay or nursing caries.

When explaining the concept of disability to a local community group, which of the following descriptions would be most appropriate for the nurse to include? A)A disability is a condition someone has, not something one is. B)The plan of care for two persons with the same disability would be the same. C)A disability refers to a person's inability to do something. D)Handicap and disability are two terms that go hand in hand.

A Feedback: Long-held negative views of disabilities are being replaced with new and more positive approaches that view individuals and their challenges from a more holistic standpoint. Thus, a disability is something that a person has, not something one is. Even persons with the same disability but their plan of care can be quite different. Earlier terminology equated disability with an inability or incapacity or handicap that presents a more negative view of the condition.

When describing the importance of the World Health Report to a group of students, the instructor would emphasize which of the following? A) It challenged the world community to focus more attention on unhealthy behaviors that lead ultimately to chronic disease, disability, and early mortality. B) Lifestyle choices are not one of the key contributors to morbidity and mortality levels in either affluent or poor countries. C)Infectious disease is the major threat. D)Malnutrition is the only threat.

A Feedback: The World Health Report challenged the world community to focus more attention on unhealthy behaviors that lead ultimately to chronic disease, disability, and early mortality. It recognized that although infectious disease and malnutrition require ongoing vigilance because they continue to plague many parts of the world, they are not the only threat. Lifestyle choices are also one of the key contributors to morbidity and mortality levels in both affluent and poor countries.

In Healthy People 2020, what is the emphasis of the categories of objectives that relate to the goal of "Disability and Health to promote the health and well-being of people with disabilities?" A)A growing emphasis on a holistic approach that recognizes life satisfaction is just as important to health and well-being as preventative services B)A growing realization that healthy-life years for persons with disabilities do not equate to decreased health costs at local, state, and national levels, unlike they do for persons without disabilities C)Focus on personal responsibility for healthy lifestyle by persons with disabilities D)Concept that persons with disabilities are always dependent on others

A Feedback: In Healthy People 2020, the emphasis of the categories of objectives that relate to the goal of "Disability and Health to promote the health and well-being of people with disabilities" is a growing emphasis on a holistic approach that recognizes life satisfaction is just as important to health and well-being as preventative services. It also indicates a growing realization that healthy life years for persons with disabilities equate to decreased costs at local, state, and national levels, just as they do for persons without disabilities. There is not a focus on personal responsibility for healthy lifestyle by persons with disabilities or a concept that persons with disabilities are always dependent on others.

A community health nurse is about to make the first home visit to a family based on a referral from the hospital where the high-risk infant was born to an adolescent mother about 6 days ago. The nurse plans to assess the family and home environment and provide anticipatory guidance. On arrival, the mother sleepily answers the door after the nurse rings the bell and knocks several times. The mother lets the nurse in but is not happy to see the nurse. She gets the infant and places him in the nurse's arms, then sits across the room and turns on the TV. Which response by the nurse would be most appropriate? A) "You must be very tired caring for a newborn baby." B) "I wasn't planning to hold the baby. You should hold him, you are the mother." C) "I need to talk to you, so please turn off the TV." D) "How do you feel being a mother at 17?"

A It is always best to acknowledge a client's feelings, and from the scenario, observations and referral information, being fatigued 6 days after having a baby is very possible. The option about holding the baby is not the best way to respond to the mother or the first comment to make, but holding the baby gives the nurse an opportunity to role model infant interaction and caring and assess his or her cleanliness and clothing appropriateness. However, the nurse may want to say that he or she would like to wash his or her hands before holding the baby. This gives the nurse an additional opportunity to teach and assess another room when washing hands. Asking to turn off the TV might be appropriate after a few moments of introductory conversation. It would be best to ask the mother to turn the TV down because of difficulty hearing her. The nurse will win the mother's favor with this approach better than by asking her to turn off the TV. Asking how she feels being a mother at 17 might be information to gather, but asking it directly as a first comment sounds like a put-down. There are better ways to obtain this information a bit later in the visit.

A community health nurse, visits an 81-year- old newly diagnosed insulin-dependent diabetic who lives alone. The nurse has visited the family three previous times and is visiting early in the morning to observe his client's insulin administration technique. The nurse finds the client depressed over his or her situation one morning. The nurse suggests that together they list positive points about his or her situation and discuss his or her feelings. This is an example of which of the following? A) Strengthening B) Data collection C) Goal-directed questioning D) Measuring family functioning

A The nurse is demonstrating a communication technique called strengthening which is used to assist the family in becoming independent of the nurse's services. Data collection, goal- directed questioning, and measurement of family function are methods used for assessment.

A pregnant woman with hypertensive disease comes to the community health center for follow up. Which of the following would be appropriate to include when teaching the woman about controlling her blood pressure at this time? A) Ensuring rest periods with legs elevated B) Limiting her daily intake of fluid C) Restricting intake of fresh fruits D) Encouraging additional weight gain

A Various methods are employed to attempt to prevent and control hypertension during pregnancy, namely, a diet rich in fresh fruits and vegetables, adequate fluid intake, weight gain limitations, rest, and regular exercise. These remain the most common preventive suggestions that community health nurses, in collaboration with the clients' primary health care providers, can give to their pregnant clients. A calm environment, along with periods of rest with the pregnant woman either elevating her feet or reclining in a left side-lying position, is also recommended.

Which of the following issues have been identified as contributing to the rise in the population of those who are homeless? Select all that apply. A. Poverty B. Lack of affordable health care C. Substance abuse D. Severe and persistent mental illness E. Growth in the number of family members living together

A, B, C, D A. Poverty B. Lack of affordable health care C. Substance abuse D. Severe and persistent mental illness

Which of the following statements about the structural dimension of the community as client are accurate? Select all that apply. A) Structure of a community refers to its services and resources. B) Community associations, groups, and organizations provide a means for accessing needed services. C) Adequacy and appropriateness of health services can be determined by examining patterns of use, number and types of health and social services, and quality measures. D) Demographic data, such as socioeconomic and racial distribution, age, gender, and educational level, are important indicators of community structure. E) Structure is the most common measure of the health of a community.

A, B, C, D Feedback: Structure of a community refers to its services and resources. Community associations, groups, and organizations provide a means for accessing needed services. Adequacy and appropriateness of health services can be determined by examining patterns of use, number and types of health and social services, and quality measures. Demographic data, such as socioeconomic and racial distribution, age, gender, and educational level, are important indicators of community structure. Status/people is the most common measure of the health of a community

Which of the following statements about competent communities are accurate? Select all that apply. A) A competent community is able to collaborate effectively identifying community needs and problems. B) A competent community is able to achieve a working consensus on goals and priorities. C) A competent community is able to agree on ways and means to implement the agreed-upon goals. D) A competent community has no problems. E) A competent community is able to collaborate effectively to take the required actions.

A, B, C, E Feedback: A competent community is able to collaborate effectively identifying community needs and problems. A competent community is able to achieve a working consensus on goals and priorities. A competent community is able to agree on ways and means to implement the agreed-upon goals. A competent community is able to collaborate effectively to take the required actions. All communities have problems.

A community health nurse is preparing for a presentation on disabilities and is planning to include information about Healthy People 2020 goals. When addressing these goals, the nurse identifies differences from the goals addressed by Healthy People 2000. Which of the following would the nurse identify as different? Select all that apply. A)Rather than individuals with disabilities and/ or chronic illnesses being defined by their limiting conditions, they are viewed as having the potential to meet and exceed health promotion and disease prevention goals set for the nation's population as a whole. B)The approach to disability now emphasizes the principles of health promotion and disease prevention for those currently experiencing disabilities and/or chronic illness. C)The goals for Healthy People 2000 have an entire section on disability and secondary conditions. A growing realization that healthy life-years for persons with disabilities equate to D) decreased health costs at local, state, and national levels, just as they do for persons without disabilities.

A, B, D Feedback: A comparison among Healthy People plans over the last decades underscores the emergence of new approaches to identifying priority areas and planning to improve the health of individuals with disabilities and chronic illness. In Healthy People 2000, only one priority was devoted to disability and chronic illness. Over the past two decades, interest in disability has accelerated. Areas that are focused on with Healthy People 2020 include rather than individuals with disabilities and/or chronic illnesses being defined by their limiting conditions, they are viewed as having the potential to meet and exceed health promotion and disease prevention goals set for the nation's population as a whole

A community health nurse is working as part of a health planning team to determine the needs of pregnant teenagers in the community. Which of the following population variables would be important to assess? Select all that apply. A) Rate of growth or decline among those in need B) Health needs and practices of subculture groups C) Geographic placement of proposed health services D) Functions of community organizations E) Social class of the community F) Level of agreement on community goals

A, B, E Feedback: Population variables include size, density, composition, rate of growth or decline, cultural characteristics, social class structure, and mobility. Pregnant teens are a subculture of teenagers. Cultural differences are population variables along with the numbers, social class, and mobility. Health needs vary among subculture and ethnic populations. Geographic placement reflects location. Functions of community organizations and social class reflect the social system.

Between 1900 and 1955, the leading causes of mortality were pneumonia, tuberculosis, and diarrhea/enteritis. By mid-century, nurses faced new challenges as the leading causes of death became: 1. Heart disease, cancer, and cerebrovascular disease 2. Influenza 3. Tropical diseases from increased travel abroad 4. Diseases from overcrowded conditions in large cities

ANS: 1 Chronic illness care, long-term illness and disability, and disease prevention became the new challenges for nurses.

Neighborhood centers that provided health care and social welfare programs were called: 1. Settlement houses 2. Nursing care centers 3. Nurse-managed clinics 4. Public health services

ANS: 1 Correct by definition. Nursing centers, nurse-managed clinics, and public health services are not necessarily in neighborhoods.

A school nurse provides education on drug abuse to a high school class. This nursing action is an example of which level of preventive care? 1. Primary prevention 2. Secondary prevention 3. Tertiary prevention 4. Primary intervention

ANS: 1 Page: 726 Feedback 1 Providing nursing education on drug abuse to a high school class is an example of primary prevention. Primary prevention services are aimed at reducing the incidence of mental health disorders within the population. 2 Secondary prevention is aimed at early detection and prompt intervention. 3 Tertiary prevention is aimed at reduction of symptoms. 4 Primary intervention is not a term associated with the public health model.

Which of the following are characteristics of a Program of Assertive Community Treatment (PACT), as described by the National Alliance on Mental Illness (NAMI)? (Select all that apply.) 1. PACT offers nationally based treatment to people with serious and persistent mental illnesses. 2. PACT is a type of case-management program. 3. The PACT team provides services 24 hours a day, 7 days a week, 365 days a year. 4. The PACT team provides highly individualized services directly to consumers. 5. PACT is a multidisciplinary team approach.

ANS: 2, 3, 4, 5 Page: 739-740 Feedback 1. NAMI defines PACT as a service-delivery model that provides comprehensive, locally, not nationally, based treatment to people with serious and persistent mental illnesses. 2. PACT is a type of case-management program. 3. The PACT team provides these services 24 hours a day, 7 days a week, 365 days a year. 4. PACT is a type of case-management program that provides highly individualized services directly to consumers. 5. It is a team approach and includes members from psychiatry, social work, nursing, substance abuse, and vocational rehabilitation.

A client at the mental health clinic tells the case manager, "I can't think about living another day, but don't tell anyone about the way I feel. I know you are obligated to protect my confidentiality." Which case manager response is most appropriate? 1. "The treatment team is composed of many specialists who are working to improve your ability to function. Sharing this information with the team is critical to your care." 2. "Let's discuss steps that will resolve negative lifestyle choices that may have increased your suicidal risk." 3. "You seem to be preoccupied with self. You should concentrate on hope for the future." 4. "This information is secure with me because of client confidentiality."

ANS: 1 Page: 739 Feedback 1 The most appropriate response by the case manager is to explain that sharing the information with the treatment team is critical to the client's care. This case manager's priority is to ensure client safety and to inform others on the treatment team of the client's suicidal ideation. 2 Stating "Let's discuss steps that will resolve negative lifestyle choices that may have increased your suicidal risk" does not protect the client's safety, which is the priority. 3 Stating "You seem to be preoccupied with self. You should concentrate on hope for the future" does not protect the client's safety, which is the priority. 4 Stating "This information is secure with me because of client confidentiality" does not protect the client's safety, which is the priority.

Which of the following have been assessed as the most common types of mental illness identified among homeless individuals? (Select all that apply.) 1. Schizophrenia 2. Body dysmorphic disorder 3. Antisocial personality disorder 4. Neurocognitive disorder 5. Conversion disorder

ANS: 1, 3, 4 Page: 745-750 Feedback 1. A number of studies have been conducted, primarily in large, urban areas, which have addressed the most common types of mental illness identified among homeless individuals. Schizophrenia is frequently described as the most common diagnosis. 2. Body dysmorphic disorder is not among the most common types of mental illnesses among homeless individuals. 3. Other prevalent disorders include personality disorders, such as antisocial personality disorder. 4. Other prevalent disorders include neurocognitive disorders. 5. Conversion disorder is not among the most common types of mental illnesses among homeless individuals.

Nurses who provided care to people in their homes and provided that care to several people at a time were called: 1. Private duty nurses 2. Visiting nurses 3. Public health nurses 4. Community staff nurses

ANS: 2 Correct by definition

A client diagnosed with schizophrenia is hospitalized owing to an exacerbation of psychosis related to nonadherence with antipsychotic medications. Which level of care does the client's hospitalization reflect? 1. Primary prevention level of care 2. Secondary prevention level of care 3. Tertiary prevention level of care 4. Case management level of care

ANS: 2 Page: 731 Feedback 1 Primary prevention aims are preventing the need of services. 2 The client's hospitalization reflects the secondary prevention level of care. Secondary prevention aims at minimizing symptoms and is accomplished through early identification of problems and prompt initiation of effective treatment. 3 Tertiary prevention aims at reducing the symptoms of a disease or illness. 4 Case management level of care is not a term associated with the public health model.

A nursing instructor is teaching about case management. What student statement indicates that learning has occurred? 1. "Case management is a method used to achieve independent client care." 2. "Case management provides coordination of services required to meet client needs." 3. "Case management exists mainly to facilitate client admission to needed inpatient services." 4. "Case management is a method to facilitate physician reimbursement."

ANS: 2 Page: 739 Feedback 1 This statement indicates that further education is needed. 2 The instructor evaluates that learning has occurred when a student defines case management as providing coordination of services required to meet client needs. Case management strives to organize client care so that specific outcomes are achieved within allotted time frames. 3 This statement indicates that learning has not occurred. 4 This statement indicates that further teaching is required.

A homeless client comes to an emergency department reporting cough, night sweats, weight loss, and blood-tinged sputum. Which disease, which has recently become more prevalent among the homeless community, should a nurse suspect? 1. Meningitis 2. Tuberculosis 3. Encephalopathy 4. Mononucleosis

ANS: 2 Page: 745-750 Feedback 1 Meningitis has not recently become more prevalent. 2 The nurse should suspect that the homeless client has contracted tuberculosis. Tuberculosis is a growing problem among individuals who are homeless, owing to being in crowded shelters, which are ideal conditions for the spread of respiratory tuberculosis. Prevalence of alcoholism, drug addiction, HIV infection, and poor nutrition also impact the increase of contracted cases of tuberculosis. 3 Encephalopathy has not recently become more prevalent. 4 Mononucleosis has not recently become more prevalent.

A major provision of the Social Security Act of 1935 was the establishment of: 1. The Frontier Nursing Service to provide nursing service to rural communities 2. State and local community health services and training of personnel 3. District nursing to provide home health care to sick people 4. Community-based settlement houses

ANS: 2 Title VI of the Social Security Act established state and local community health services and training of personnel.

DELETE

ANS: 2, 3, 1. Page: 724-725 Feedback The premise of the model of public health is based largely on the concepts set forth by Gerald Caplan (1964) during the initial community mental health movement. They include primary prevention, secondary prevention, and tertiary prevention. 1. Primary prevention is aimed at reducing the incidence of mental disorders within the population. 2. Secondary prevention is aimed at minimizing early symptoms of psychiatric illness and directed toward reducing the prevalence and duration of the illness. 3. Tertiary prevention is aimed at providing services that reduce the residual defects that are associated with severe and persistent mental illness.

The impact of World War I on public health nursing included which of the following? Select all that apply: 1. Many communicable diseases were eradicated. 2. The depletion of the ranks of public health nurses to the war. 3. The feeling that the greatest patriotic duty was to stay at home. 4. Inadequate funding was the major obstacle to extending nursing services in the community.

ANS: 2, 3, 4 Both World Wars I and II depleted the public health nurse population as nurses went off to war. The feeling of patriotism extended to the idea that patriotic duty could also be served on the home front.

Which of the following clients should a nurse recommend for a structured day program? (Select all that apply.) 1. An acutely suicidal teenager 2. A chronically mentally ill woman who has a history of medication nonadherence 3. A socially isolated older individual 4. A depressed individual who is able to contract for safety 5. A client who is hearing voices that tell the client to harm s

ANS: 2, 4 Page: 740-741 Feedback 1. A suicidal teenager is not an appropriate candidate for a structured day program. 2. The nurse should recommend a structured day program for a chronically mental ill woman who has a history of medication non-adherence. 3. A socially isolated older adult is not an appropriate candidate for a structured day program. 4. The nurse should recommend a structured day program for a depressed individual who is able to contract for safety. 5. A client hearing voices is not an appropriate candidate for a structured day program.

A nursing instructor is teaching about the Community Health Centers Act of 1963. What was a deterring factor to the proper implementation of this act? 1. Many prospective clients did not meet criteria for mental illness diagnostic-related groups. 2. Zoning laws discouraged the development of community mental health centers. 3. States could not match federal funds to establish community mental health centers. 4. There was not a sufficient employment pool to staff community mental health centers.

ANS: 3 Page: 723 Feedback 1 A client who did not meet criteria for mental illness was not a deterring factor. 2 Zoning laws were not a deterring factor. 3 A deterring factor to the proper implementation of the Community Mental Health Centers Act of 1963 was that states could not match federal funds to establish community mental health centers. This act called for the construction of comprehensive community mental health centers to offset the effect of deinstitutionalization, the closing of state mental health hospitals. 4 Insufficient staffing was not a deterring factor.

A nurse is implementing care within the parameters of tertiary prevention. Which nursing action is an example of this type of care? 1. Teaching an adolescent about pregnancy prevention 2. Teaching a client the reportable side effects of a newly prescribed neuroleptic medication 3. Teaching a client to cook meals, make a grocery list, and establish a budget 4. Teaching a client about his or her new diagnosis of bipolar disorder

ANS: 3 Page: 725 Feedback 1 Teaching about pregnancy prevention is primary prevention. 2 Teaching about side effects of a new medication and bipolar disorder is secondary prevention. 3 The nurse who teaches a client to cook meals, make a grocery list, and establish a budget is implementing care within the parameters of tertiary prevention. Tertiary prevention consists of services aimed at reducing the residual effects that are associated with severe and persistent mental illness. It is accomplished by preventing complications of the illness and promoting rehabilitation that is directed toward achievement of maximum functioning. 4 Teaching about bipolar disorder to a newly diagnosed client is secondary prevention.

When a home health nurse administers an outpatient's injection of haloperidol decanoate (Haldol decanoate), which level of care is the nurse providing? 1. Primary prevention level of care 2. Secondary prevention level of care 3. Tertiary prevention level of care 4. Case management level of care

ANS: 3 Page: 739 Feedback 1 Primary prevention is aimed at preventing services before they are needed. 2 Secondary prevention is aimed at early detection and fast intervention. 3 When administering medication in an outpatient setting, the nurse is providing a tertiary prevention level of care. Tertiary prevention services are aimed at reducing the residual effects that are associated with severe and persistent mental illness. It is accomplished by preventing complications of the illness and promoting rehabilitation that is directed toward achievement of maximum functioning. 4 Case management level of care is not a term associated with the public health model.

however, the resources were not available to make the transition a success." 4. "Your discharge from the state hospital was based on presumed family support, and this was not forthcoming."

ANS: 3 Page: 745-750 Feedback 1 This is inaccurate because the client was not discharged prematurely. 2 This is inaccurate because the client was not discharged prematurely due to schizophrenia. 3 The most accurate nursing response is to explain to the client that the resources were not available to make transitioning out of a state hospital a success. There are several factors that are thought to contribute to homelessness among the mentally ill: deinstitutionalization, poverty, lack of affordable housing, lack of affordable health care, domestic violence, and addiction disorders. 4 This statement is not accurate based on the client's situation.

Ruth Freeman was a leading public health educator, administrator, consultant, author, and leader of the National Health Organization of the twentieth century. Which of the following characterize her philosophy of nursing? 1. Nursing should move from acute care to community-based care. 2. Nurse practitioners should control public health. 3. Nursing is about caring for people, and is also intellectually challenging and offers many professional opportunities. 4. All nurses should seek graduate education to increase their credibility.

ANS: 3 Ruth Freeman felt that nursing was about caring for people and was also intellectually challenging and offered many professional opportunities. She did not specifically espouse the other statements.

An accomplishment for which the Frontier Nursing Service was noted is: 1. Improvement of the care of sick and injured soldiers 2. Establishment of a fee-for-service program for workers at Metropolitan Life Insurance Company 3. Reduction of infant and maternal mortality regardless of environmental conditions 4. Increasing funding for communicable disease treatment

ANS: 3 The Frontier Nursing Service nurses were trained in nursing public health and midwifery and provided care to rural and inaccessible areas, which led to reduced mortality.

The Elizabeth Poor Law of 1601 is similar to which of the following current laws? 1. Welfare 2. Food Stamps 3. Medicaid 4. Medicare

ANS: 3 The Poor Law guaranteed medical care for poor, blind, and "lame" individuals, similar to Medicaid.

In 1909, Yssabella Waters published her survey Visiting Nurses in the United States. This document highlighted the fact that: 1.Nurses were trained by Boards of Education. 2.Trained nurses adequately covered less densely populated areas. 3.Visiting nurses services were concentrated in the northeastern quadrant of the nation. 4.Nurses were curing diseases such as tuberculosis and typhoid fever that greatly enhanced their credibility.

ANS: 3 This report emphasized the fact that visiting nurse services were concentrated in the northeast, which underscored the need for rural health.

Nurses have worked in the community to improve the health care status of individuals, families, populations, and vulnerable groups. Part of the appeal of this type of nursing is: 1. Working with wealthy contributors who provide the funds 2. Locating the source of disease and curing patients 3. The autonomy of practice and independence in problem solving and decision making 4.Caring for soldiers on the battlefield

ANS: 3 Community health nurses have a long history of autonomous practice, problem solving, and decision making. The other three answers relate to acute care nursing or gathering funding. Community health care nurses did engage in these activities in the nineteenth century, but overall, they have been best known for autonomy of practice.

When intervening with a married couple experiencing relationship discord, which nursing action reflects an intervention at the secondary level of prevention? 1. Teaching assertiveness skills in order to meet assessed needs 2. Supplying the couple with guidelines related to marital seminar leadership 3. Teaching the couple about various methods of birth control 4. Counseling the couple related to open and honest communication skills

ANS: 4 Page: 726-727 Feedback 1 Teaching assertiveness skills in order to meet assessed needs is tertiary prevention. 2 Supplying the couple with guidelines related to marital seminar leadership is primary prevention. 3 Teaching the couple about various methods of birth control is primary prevention. 4 Counseling the couple related to open and honest communication skills is a reflection of a nursing intervention at the secondary level of prevention. Secondary prevention aims at minimizing symptoms and is accomplished through early identification of problems and prompt initiation of effective treatment.

An instructor is teaching nursing students about the difference between partial and inpatient hospitalization. In what way does partial hospitalization differ from traditional inpatient hospitalization? 1. Partial hospitalization does not provide medication administration and monitoring. 2. Partial hospitalization does not use an interdisciplinary team. 3. Partial hospitalization does not offer a comprehensive treatment plan. 4. Partial hospitalization does not provide supervision 24 hours a day.

ANS: 4 Page: 740-741 Feedback 1 Partial hospitalization provides medication administration and monitoring. 2 Partial hospitalization uses an interdisciplinary team. 3 Partial hospitalization offers a comprehensive treatment plan. 4 The instructor should explain that partial hospitalization does not provide supervision 24 hours a day. Partial hospitalization programs generally offer a comprehensive treatment plan formulated by an interdisciplinary team, including medication administration. They have proved to be an effective method of preventing hospitalization.

When attempting to provide health-care services to the homeless, what should be a realistic concern for a nurse? 1. Most individuals that are homeless reject help. 2. Most individuals that are homeless are suspicious of anyone who offers help. 3. Most individuals that are homeless are proud and will often refuse charity. 4. Most individuals that are homeless relocate frequently.

ANS: 4 Page: 745-750 Feedback 1 It is inaccurate to state that most homeless reject help. 2 It is inaccurate to state that most homeless are suspicious of those who offer help. 3 It is inaccurate to state that most homeless refuse charity. 4 A realistic concern in the provision of health-care services to the homeless is that individuals who are homeless relocate frequently. Frequent relocation confounds service delivery and interferes with providers' efforts to ensure appropriate care.

An intervention by the United States federal government to protect the health of its citizens was the establishment of the Marine Hospital Service, presently known as the Public Health Service. Its purpose was to: 1. Set policy on quarantine legislation for immigrants 2. Establish hospital-based programs to care for the sick at home 3. Establish and promote environmental interventions such as adequate housing and sanitation for urban cities 4. Provide health care for merchant seamen to protect seacoast ports and cities from epidemics

ANS: 4 Providing health care to seamen was an early effort by the federal government to improve public health. Its purpose was to secure its maritime trade and seacoast cities.

5. A community-oriented nurse has identified obesity as a problem in the middle school. The next step in a population-focused practice is to make information available about the health of the middle school students. This describes the core public health function of: a. assessment. b. assurance. c. policy development. d. research.

ANS: A Assessment is a core function of public health and refers to systematically collecting data concerning the population, monitoring the population's health status, and making information available on the health of the community. In a community-oriented approach, a nurse would apply both nursing and public health theory. In this case, assessment would be the first step from the perspective of both theories. Because the practice is population focused and community oriented, it would involve the assessment of the community subpopulation of middle school children and the impact of obesity on their overall health status.

5. A case manager employed by a health maintenance organization is charged with discontinuing home health services for a disabled homebound elderly client in an effort to reduce costs. The case manager reviews the client's record and determines that home health services are still indicated for this client. The ethical dilemma faced by the case manager is: a. beneficence. b. justice. c. nonmaleficence. d. veracity.

ANS: A Beneficence—"doing good"—becomes an ethical dilemma when the insurer's goal of containing costs supersedes the case manager's duty to improve health and relieve suffering. This ethical tenet of nursing practice to act in the best interest of the client also involves fidelity to the nurse-client trust relationship. Duty to clients to secure benefits on their behalf and to limit unnecessary expenditures can create dilemmas when the goals are not uniform. Advocacy on this client's behalf with accurate and updated information based on sound assessment, evidence-based practice, and liability risk management could produce a favorable outcome for all parties. The incorrect action would be to do nothing.

7. In case management, it is unlikely that any single professional has the expertise, knowledge, or skills required to achieve success. The synergy produced by all involved parties (client, providers, payers, family/significant others, and community organizations) can result in successful outcomes. These statements relate to the sequential process of: a. collaboration. b. communication. c. cooperation. d. negotiation.

ANS: A Collaboration is achieved through a developmental process carried out in a sequence, yet it is reciprocal among those involved. Teamwork and collaboration require extensive skill sets to achieve successful outcomes. No single professional has the expertise required for all aspects. Synergy among all parties involved is required. Although communication is essential to collaboration, it is not sufficient to create or maintain collaboration.

8. A community-oriented nurse conducts home visits to new parents to assess the health status of the infant, the parent-child relationship, the parents' knowledge regarding the care of the infant, and the need for health department and social services referrals to support the needs of the new parents and the infant. This can best be described as an example of: a. clinical community health practice. b. community-based practice. c. population-focused practice. d. public health nursing.

ANS: A Community-oriented nurses who provide direct care services to individuals, families, or groups are engaging in a clinically oriented practice even when they apply concepts of population-focused, community-oriented strategies along with their direct care clinical strategies.

4. A nurse providing a tertiary prevention intervention to a population of women who are HIV positive will most likely: a. educate about self-care and the women's rights as employees. b. establish a partnership with a community to initiate a community health center. c. help identify new cases and ensure that clients receive proper treatment. d. teach how to lobby state legislators.

ANS: A Helping clients understand their rights to protect from on-the-job discrimination is part of the nurse-advocate role. Tertiary prevention includes educating women with a chronic disease such as HIV about self-care strategies and health-promotion activities to minimize risky behaviors and poor health outcomes. Enhancing levels of self-esteem and empowerment can prevent feelings of powerlessness and hopelessness, which contribute to vulnerability.

6. The nurse engaging in a formative program evaluation would most likely: a. conduct medical record audits for quality assurance. b. make a home visit before a client is discharged from the program. c. participate in a new client evaluation. d. write a policy for risk management.

ANS: A Process evaluation, also referred to as formative evaluation, occurs during program implementation and makes it possible to make midterm corrections to ensure the achievement of program goals. Process evaluation is an ongoing function of examining, documenting, and analyzing the progress of a program. This type of program monitoring can be used to justify continuing the program, because it will address the efficiency and effectiveness of the program in meeting its goals. Quality assurance programs are examples of program evaluation in health care delivery.

3. A community health nurse is serving as a case manager for premature infants receiving home health care. The case manager arranges for an in-home apnea monitor and daily home visits by a registered nurse who specializes in high-risk pediatrics. The case manager is demonstrating which of the following case manager roles? a. Consultant and coordinator b. Mentor and liaison c. Monitor and reporter d. Standardization monitor and negotiator

ANS: A The case manager is responsible for fulfilling a variety of roles. The nurse is acting as a consultant when working with suppliers to arrange for the apnea monitor. The nurse is acting as a coordinator when arranging for needed heath care services such as daily home visits by a specially trained pediatric registered nurse.

7. Local officials have requested a program evaluation of a comprehensive teen sex education program offered in the local schools in preparation for annual budget-planning discussions. The public health nurse (PHN) determines that the teen pregnancy rate has gradually declined over the years that the program has been in place. The nurse has also identified the evaluation plan that was articulated during the program planning phase. The best tool for demonstrating the efficiency of this program is: a. cost-benefit analysis. b. cost-efficiency analysis. c. relevance assessment. d. school records audit.

ANS: A The efficiency of a program can be evaluated through a formative or summative evaluation. The evaluator may be able to determine whether a given program provides better benefits at a lower cost than a similar program, or whether the benefits to the clients justify the costs of the program (efficiency). Public health programs are usually recognized as having a net positive impact, and appropriate sex education can reduce the incidence of teen pregnancy by altering teen sexual behaviors. To perform a cost-benefit study, the evaluator must decide which costs and which benefits are to be included, how the costs and benefits are to be valued, and what constraints are to be considered (e.g., legal, ethical, social, and/or economic). Health programs involving politically sensitive issues such as teen sex education can benefit from the articulation of measures to determine costs, benefits, and constraints in the program planning phase.

6. A community health nurse directly contacts a mammography clinic to arrange an appointment for a migrant worker with limited English language skills. The nurse communicates with the client through an interpreter to ensure that the appointment is scheduled to meet her needs and that the client understands the procedure to be performed. The role played by the nurse in this encounter with a member of a vulnerable population can best be described as: a. advocacy. b. empowerment. c. partnership. d. social justice.

ANS: A The nurse functions as an advocate when referring clients to other agencies and ensuring that the clients' preferences are accommodated.

6. The state public health agency has received multiple complaints regarding the availability of elder transportation services to a specific county senior center. The state agency assigns a public health nurse to work with the community to evaluate its program for elder transportation services to publicly sponsored eldercare programs. The public health core function applied is: a. assurance. b. policy development. c. primary prevention. d. public transportation.

ANS: A The public health core function of assurance focuses on the responsibility of public health agencies to be sure that activities are appropriately carried out to meet public health goals and plans. This involves making sure that essential community-oriented health services are available and accessible, especially to vulnerable populations who would otherwise not receive necessary services. Assurance also includes assisting communities to implement and evaluate plans and practices.

What CHN action could potentially increase accessibility to health care services for clients experiencing mental illness and housing instability? a. Apply for a grant to fund a mobile clinic to take health care to the clients. b. Distribute flyers to homeless persons that detail the location of various health care services. c. Refer homeless clients to temporary housing facilities. d. Solicit donations for food and clothing to be distributed to the homeless.

ANS: A Accessibility refers to the ability of clients to access needed health care services. While all of the options listed lead to opportunities for achieving better health outcomes, either directly or indirectly, only a mobile health clinic can potentially improve accessibility to health care. Neighbourhood clinics, mobile vans, and home visits can bring health care to people unable to access health care units. Coordinating health care services from a central location often improves client compliance because it reduces the stress of getting to multiple places.

24. Which of the following best explains why the home health nurse would essentially repeat the same information given to the client by the nurse in the hospital? a. Clients find it difficult to learn in the midst of the stress of the acute care setting and often don't remember what the hospital nurse taught. b. Hearing something from a different person, using different words and examples, can help ensure that learning is retained. c. Home health nurses often don't know what hospital-based nurses have already taught. d. Learning depends on receiving information more than once.

ANS: A Although all of the responses are potentially true, the bottom line is that clients may find it difficult to learn while they are hospitalized. Consequently, home care nurses should communicate clearly with discharge planners about the therapeutic plan and medication regimens, as well as what clients have been taught about self-care and symptoms that should be reported to the physician. The nurse should share this same information again, unless the client clearly demonstrates having the knowledge and skills being reviewed.

A nurse schedules an appointment with a physician who has a practice in the community to learn more about the communitys beliefs regarding childhood immunizations. This is known as: a. Informant interview b. Participant observation c. Secondary analysis d. Windshield survey

ANS: A An informant interview is a method of community data collection that involves directed conversation with selected community members.

A nurse is teaching a postpartum mother how to breastfeed her infant. The nurse notes that the mother is alert and agrees that breastfeeding is important to her and beneficial to her baby. The nurse outlines the expectations of breastfeeding for the mother and the baby. Considering the events of instruction, what should the nurse do next? a. Ask the mother about her previous experience with breastfeeding. b. Demonstrate how to position the baby for breastfeeding. c. Show the mother a video about breastfeeding. d. Have the mother demonstrate breastfeeding.

ANS: A Asking the mother about her previous experience with breastfeeding identifies her educational needs. Using the TEACH mnemonic, the first thing the nurse should do is Tune in: listen before starting to teach

While screening for diabetes at a community clinic, a CHN found out that a new client had type 2 diabetes. The CHN then provided counselling, referred the client to an endocrinologist for initial assessment and treatment, helped with arrangements for financial assistance, arranged transportation, and booked a follow-up appointment. What role does this type of service exemplify? a. Case management b. Client advocacy c. Holistic care d. Wrap-around services

ANS: A Case management involves linking clients with services and providing direct community health nursing services, including teaching, counselling, screening, and immunizing. Linking health services is accomplished by making appropriate referrals and by following up with clients to ensure that the desired outcomes from the referral were achieved.

What action should a CHN take when using the case management approach with vulnerable populations? a. Be willing to enter into a long-term relationship with families. b. Direct and control the client's care because the CHN knows what is most needed. c. Encourage families to become self-sufficient and less dependent on nursing personnel for advice and referrals. d. Rotate assignments periodically, to prevent attachment and codependency.

ANS: A Case management is a process that enhances continuity and appropriateness of care. When using case management with vulnerable populations, the CHN can offer the greatest benefit when she or he is willing to develop long-term relationships with the families served. Long-term relationships create trust and ensure continuity of care. Care involves a partnership between the CHN and the client. CHNs who direct and control the client's care cannot establish a trusting relationship and may inadvertently foster a cycle of dependency and lack of personal health control measures.

After identifying the boundaries of a community, what is the next step the nurse should take in completing a community assessment? a. Gather relevant existing data and generating missing data. b. Plan interventions that benefit the entire community. c. Formulate nursing diagnoses. d. Evaluate the interventions that were used.

ANS: A Community assessment includes gathering relevant existing data and generating missing data.

Which format would be most appropriate for teaching a group of nursing students who are learning to give injections? a. Demonstration b. Health fair c. Lecture d. Nonnative language session

ANS: A Demonstration also includes return demonstration. Giving injections can best be learned by seeing the behavior being done.

A public health nurse collaborates with a physicians clinic to increase the number of infants and toddlers who receive lead screening. According to the integrative model, what client system and focus of care are being addressed? a. Aggregate level health promotion b. Aggregate level illness prevention c. Family level illness care d. Individual level health promotion

ANS: A Each succeeding level of the client system is more complex. In this situation, the public health nurse is working with an aggregate of infants and toddlers. The focus of care being addressed is health promotion, focusing on positive measures such as education for healthy living and promotion of favorable environmental conditions as well as periodic examinations, including well-child developmental assessment and health education.

What is the purpose of the color-coded wedges on the Intervention Wheel? a. The interventions are grouped together in related wedges. b. The wedges consist of referral information for each wedge. c. The element of health teaching is the predominant feature of each wedge. d. Coalition building must be implemented with each wedge.

ANS: A Each wedge consists of related interventions. The other answers describe some of the individual wedges.

A correctly written objective would be: a. Each member of the Jones family will give an insulin injection to Billy with accurate dosage 100% of the time for 10 consecutive trials. b. Ms. Smith will perform a blood sugar test on herself with an accurate blood sugar reading. c. The community will take their children to receive immunizations within 1 month of the immunization due date. d. Fifty percent of the eligible women seen in the clinic will return for their scheduled mammogram appointment.

ANS: A Objectives are specific, short-term criteria that need to be met as steps toward achieving the long-term goal. They are written as statements of an intended outcome or expected change in behaviors and should be defined in measurable terms.

23. The mother of a high school student newly diagnosed with a condition that will require special health care services is concerned that the student will be required to be home-schooled away from the friends he has developed. Which of the following would be the most appropriate response by the school nurse? a. "Federal legislation requires that the school make provisions for those with various challenges, so your child will be able to remain in school as long as he is able." b. "I realize that this will be a difficult adjustment, but home-schooling has improved over recent decades and the Internet will allow your child to connect with friends." c. "Whether your child can remain in school will depend on state funding for those with disabilities. You might want to contact your congressman on this issue." d. "Your child may remain in school as long as he can manage the course requirements and doesn't flunk out."

ANS: A Federal legislation specifies that children cannot be excluded from schools because of a disability. The school must provide health services that each child needs. Legislation further requires the school district's committee on the disabled to develop individualized education plans (IEPs) for children.

Which statement about Florence Nightingale's ideas about ethics is correct? a. Nursing is a call to service, and the moral character of persons entering nursing is important. b. Ethical principles are based on the values of the individual nurse. c. Society will dictate the ethical principles to which nurses must adhere. d. Ethics are very important in times of war, such as in the Crimean War, when she set up public health centers.

ANS: A Florence Nightingale saw nursing as a call to service and viewed the moral character of persons entering nursing as important.

24. A group of nursing students are scheduled to present a program on healthy hearts to various community groups, with a daycare center being the first location. What of the following advice should be given to them by their instructor? a. Base the program on the audience's development and maturity. b. Bring (borrow if necessary) a model of the heart to help explain its functioning. c. Focus on entertaining the learners. d. Have lots of handouts to reinforce the lesson.

ANS: A For younger learners, it is important to keep the lesson to no more than 10 minutes in length, to use plenty of examples, pictures, and stuffed animals in the talk, and to remember the developmental stage of the children when teaching them.

What is the purpose of providing education across the three levels of prevention? Education: a. Enables clients to attain optimal health b. Identifies and treats health problems early to eliminate disability c. Enables populations to break into individuals d. Teaches people about Healthy People 2010

ANS: A Health education enables clients to attain optimal health, prevent health problems, and identify and treat health problems early to minimize disability. Elimination of a disability may not be possible.

When completing a community assessment, the nurse will: a. Identify community needs and clarify problems. b. Determine the weaknesses of a community. c. Perform the core functions of public health nursing. d. Assess individual needs within a community.

ANS: A In a community assessment, one of the core functions is a logical, systematic approach to identifying community needs, clarifying problems, and identifying community strengths and resources.

9. A nurse would like to implement a primary prevention effort to decrease the leading cause of death among children and teenagers. Which of the following actions would the nurse most likely take? a. Educate students about injury prevention measures b. Provide free condoms to sexually active students c. Screen for signs and symptoms of cancer d. Invite a guest speaker to talk about living with HIV

ANS: A Injuries are the leading cause of death in children and teenagers; therefore, prevention measures should focus on injury prevention. Because the question asks for primary prevention efforts, the intervention must occur before injury. Common interventions by the school nurse include educational programs reminding children to use their seatbelts or bicycle helmets to prevent injuries. Other classes can be on crossing the street, water safety, and fire safety.

When a nurse focuses intervention strategies on the structural dimension of community health, they are directed toward: a. Health services b. Primary prevention c. Health promotion d. Secondary prevention

ANS: A Intervention strategies include service use patterns, treatment data from various health agencies, and provider/client ratios.

A public health nurse collaborates with local community leaders to develop a local campaign educating about the importance of wearing a seat belt. What client system of the integrative model does this describe? a. Community b. Aggregate c. Family d. Individual

ANS: A Interventions to initiate or maintain healthy lifestyles must be multifocal. At the broadest level of care (community), nurses work with community leaders, other community residents, and health professionals to plan programs to promote optimal health for the community and its people.

Examples of the benefits of distributive justice are: a. Basic needs, material and social goods, liberties, rights, and entitlements b. Taxes, military service, location of incinerators or power plants c. Entitlement to equal rights and equal treatment d. The right to private property and personal assets

ANS: A Justice requires that the distribution of benefits and burdens on a society be fair or equal. the third option refers to egalitarianism, and the last option refers to libertarianism. Taxes, military service, and location of incinerators or power plants are not benefits associated with justice.

20. Which of the following comments was most likely stated by a home health nurse? a. "Every time I see a client, it costs $80." b. "Health care costs are killing our economy." c. "I can't believe how much clients are charged for things." d. "Medicaid costs are such a large portion of our state's budget."

ANS: A Nurses in many settings are not directly exposed to the financial aspects of health care, although as citizens they should be aware of the overall effects of high health care costs. In home health, nurses must be cost-conscious so that they can accurately explain to clients what Medicare will or will not cover and discuss other financial concerns. In addition, home care nurses must be knowledgeable about which medical supplies are covered.

A public health nurse is collecting data about the biological aspects of community health. What data will the nurse collect? a. Race/ethnicity b. Traditional morbidity and mortality rates c. Policy making and social change d. Homicide rates

ANS: B Biological community health is often measured by traditional morbidity and mortality rates, which can be found in The Morbidity and Mortality Weekly Report.

A diabetic client reports to the nurse that she has been learning more about controlling her blood sugars by reading information found on the Internet. Which statement by the nurse would be most appropriate? a. Looking at the date the content was posted on the website is important. b. The Internet is an unreliable source of information and should not be used. c. The best sources of information are found on pharmaceutical websites. d. Your physician will provide you better information than the Internet.

ANS: A One of the ways to assess the reliability and validity of Internet sources is to look at its currency, including the dates when the content was posted and updated.

An example of a measurable outcome health status indicator at the individual-level of practice that would be used by a public health nurse is: a. A 50-year-old woman receives annual mammograms b. School absences in a community decline c. Teachers have increased awareness of health problems d. Those in poverty utilize the free mammogram program

ANS: A Outcome health status indicators are used to measure the impact of the interventions on population health. In this case, a 50-year-old woman receiving an annual mammogram will have an impact on the population health when considering those who are receiving the screening. The other examples do not look at population health as an outcome or are not occurring at the individual-level.

Which characteristic is an indicator of community health process? a. Participation and community action b. Live birth rate c. Racial distribution d. Socioeconomic levels

ANS: A Participation and community action are indicators of community health process.

19. The board of directors is examining various submitted reports concerning its home health agency. Which of the following reports represents an example of benchmarking? a. A report by the administrator regarding how the home health agency's performance compares with that of other local and national home health agencies b. A report by the chief financial officer regarding a cost-benefit analysis related to technological advances c. A report by the chief nursing officer regarding client outcomes d. A report by the medical director regarding implementation of evidence-based practice into standards of care

ANS: A Performance improvement programs are based on measurable data, including benchmarking, which means comparing oneself with national standards and guidelines and with other agencies.

Why is it important for nurses to understand the premises of environmental health? a. Nurses should be able to assess risks and advocate for policies that support healthy environments. b. Toxicologists often consult nurses about environmental pollutants. c. Pollutant exposures such as lead are reported by nurses to the Environmental Protection Agency. d. Many Americans live in areas that do not meet current national air quality standards.

ANS: A Potential risks to health are concerns for professional nurses. It is the responsibility of the nurse to understand as much as possible about these risks: how to assess them, how to eliminate/reduce them, how to communicate and educate about them, and how to advocate for policies that support healthy environments.

A nurse is completing an exposure history using the mnemonic I PREPARE. What data would a nurse collect when asking questions about the first P? a. Present work b. Potential exposures c. Personal protective equipment use d. Problems with health

ANS: A Present work is the first P.

At a district board meeting, the CHN requests funding for an after-school recreation program that promotes healthy, fun activities in an effort to decrease substance abuse. Which level of prevention does this exemplify? a. Primary prevention b. Secondary prevention c. Tertiary prevention d. Primary and secondary prevention

ANS: A Primary prevention includes the promotion of healthy lifestyles and resiliency factors. The harm reduction approach to the issue of substance abuse focuses on health promotion and disease prevention.

7. The school health nurse has enlisted the assistance of high school role models in the areas of sports and scholarship to provide an antidrug presentation to their peers. Which of the following levels of prevention is being implemented? a. Primary b. Secondary c. Tertiary d. Both primary and secondary

ANS: A Primary prevention interventions by the school nurse include educating children and adolescents about the effects of drugs. In preventing use, students are taught by the school nurse to stay away from drugs such as marijuana, cocaine, crack, heroin, and alcohol.

What is the best intervention a community health nurse (CHN) can initiate to increase lasting resilience among new immigrants? a. Directing clients to English-as-a-second-language courses b. Giving immigrant clients money to help them get settled c. Identifying areas in the city where housing is less expensive d. Soliciting donations for food, clothing, and other needs

ANS: A Resilience refers to the ability of the client to successfully cope when faced with a threat or hardship. Individuals with low resilience are more inclined to have feelings of hopelessness and may choose suicide as a method to resolve these feelings. Support needs to be provided to those with decreased resilience in order to enhance problem-solving skills and give a greater sense of personal autonomy. When the CHN places emphasis on client strengths and assets rather than client deficits and susceptibility, resilience is more likely to increase.

A nurse who is involved in identifying individuals with unrecognized health risk factors or asymptomatic disease is using which intervention? a. Screening b. Referral and follow-up c. Surveillance d. Health teaching

ANS: A Screening involves identifying individuals with unrecognized health risk factors or asymptomatic disease.

A CHN is concerned about caregiver stress in the children of older clients with health concerns. What secondary prevention strategy can the CHN implement to limit caregiver stress? a. Asking caregivers how they are coping with their role b. Encouraging caregivers to periodically take a few hours away from their duties c. Establishing support groups for caregivers of older adult parents d. Referring some caregiving responsibilities to home health nurses (HHNs) or professional caregivers

ANS: A Secondary prevention includes screening that allows for early recognition of health concerns so that prompt interventions can prevent long-term disabilities. By asking caregivers how they are coping (thus screening), the CHN can identify problems early so that interventions can be made to limit the extent of stress, thus paving the way for improving the situation. The other options are either primary or tertiary activities, depending on whether the intervention takes place before or after development of caregiver stress.

15. A school nurse suggests to teachers that they have a session on coping strategies and stress management techniques. The nurse also sets up a peer counseling program. Which of the following is the school nurse most likely trying to prevent? a. Adolescent suicides b. Bullying c. Obesity d. Teenagers engaging in violence

ANS: A Suicide is the third leading cause of death in teenagers. To reduce the incidence of suicide in teenagers, the nurse can emphasize coping strategies and stress management techniques and organize a peer assistance program to help teenagers cope with school stresses.

Which statement about the Intervention Wheel is true? a. It provides a graphic illustration of population-based public health practice. b. It describes in detail the components of public health nursing. c. It demonstrates the practice of community health nurses for policy and lawmakers. d. It is a framework used by all health departments in the United States.

ANS: A The Intervention Wheel provides a graphic illustration that gives public health nurses a means to describe the full scope and breadth of their practice.

16. A nurse orienting to the home health role states, "I don't understand why we have to collaborate with so many other disciplines this conflicts with the concept of nurses providing all direct care to the client in the home?" Which of the following would be the best response? a. "The nurse still provides direct care for the client; however, interdisciplinary collaboration is necessary to prevent fragmentation of care." b. "Direct nursing care is a concept applied to care in tertiary facilities, such as hospitals, where materials are centrally located in one facility. In home health, this is not possible." c. "Even though home health nursing does not provide as much direct care as other nursing specialties in the community, each discipline contributes to client needs from its special knowledge base." d. "Yes, it does create conflict, but Medicare mandates interdisciplinary collaboration."

ANS: A The responsibilities and functions of other health professions in home care are dictated by Medicare regulations, professional organizations, and state licensing boards. Many of these services can be provided on a consulting basis. The plan of care should be implemented and reinforced by all involved disciplines. Therefore, interdisciplinary collaboration is required in the home health setting. Such collaboration does not conflict with the concept of providing direct care services. Direct care refers to the actual physical aspects of nursing care—anything requiring physical contact and face-to-face interactions. In home care, direct care activities include performing a physical assessment on the client, changing a dressing on a wound, giving medication by injection, inserting an indwelling catheter, or providing intravenous therapy. Direct care also involves teaching clients and family caregivers how to perform a certain procedure or task.

During a class on newborn care given at a local health department, a nursing student asks the participants to practice with baby dolls. Which step of promoting effective education is the student using? a. Eliciting performance b. Assessing performance c. Enhancing retention and transfer of knowledge d. Gaining attention

ANS: A The step of eliciting performance includes encouraging the learners to demonstrate what they have learned.

A 5-feet, 6-inch, 25-year-old woman who weighs 120 pounds runs 5 miles a day because it improves her mood and energy level. Which term best describes this example? a. Health promotion b. Illness prevention c. Health maintenance d. Health protection

ANS: A The woman is exercising to improve and promote her health. Health promotion is directed toward achieving a greater level of health.

A nurse providing care using the idea of servicing citizens, not customers is applying the: a. Ethical tenets of policy development b. Basic concepts of the feminist theory c. Underlying premise of virtue ethics d. Components of distributive justice

ANS: A There are three tenets of both policy and ethics. The approach is based on the voice of the community as the foundation on which policy is developed.

According to Leininger and Watson, the moral ideal of nursing is: a. Caring b. Advocacy c. Responsibility d. Accountability

ANS: A This conceptualization occurred as a response to the technological advances in health care science and the desire of nurses to differentiate nursing practice from medical practice.

20. A pregnant teen asks the school nurse to provide information on abortion and a list of health care providers who offer such services. If the school nurse has very strong personal beliefs against abortion, which of the following actions should be taken by the nurse? a. Call in another nurse to care for this client. b. Explain, from the nurse's perspective, all the reasons that abortion should be made illegal. c. Offer the student a combination of oral contraceptives to induce spontaneous abortion. d. Provide information on alternatives to abortion and give the client information on adoption agencies.

ANS: A This creates an ethical dilemma for the nurse. If the nurse feels so strongly that he or she cannot work with the situation, another school nurse should be called for help or the student should be referred to other health providers who can provide the care the student needs.

22. Which of the following is a primary requirement for a client to be eligible for home health nursing and Medicare reimbursement of services? a. Must be homebound b. Must be living in a medically underserved area c. Must be indigent d. Must be insured or eligible for Medicare or Medicaid

ANS: A To receive home health services, clients must be homebound. Although home health care is less expensive than hospitalization, it remains much more costly than a traditional visit to a clinic; therefore, those who are not homebound would be expected to receive care at a clinic.

The basic science applied to understanding the health effects associated with chemical exposures is: a. Toxicology b. Pharmacology c. Chemistry d. Environmental epidemiology

ANS: A Toxicology is the study of the health effects associated with chemical exposures.

Environmental health is important to nurses because chemical, biological, and radiological materials are: a. A major cause of global warming b. Often found in the air, water, and products we use c. Frequently linked to the development of chronic illnesses d. Products that nurses work with on a daily basis

ANS: B Chemical, biological, and radiological pollutants are often found in the air we breathe, the water we drink, and the products we use.

26. Which of the following practices in the home is most crucial? a. Using good handwashing procedures b. Obtaining a puncture-resistant container for family to use for needles c. Putting all contaminated material directly into a trash bag d. Washing all surfaces with disinfectant

ANS: A Universal precautions mean that all blood and body fluids are treated as potentially infectious, especially because many infections are subclinical. The nurse would use extreme care to prevent injuries when handling needles, scalpels, and razors and discard needles and syringes in puncture-resistant containers, use protective coverings if contact with blood and body fluids is expected, put materials contaminated with body fluids in double polyethylene garbage bags, and tell the family to be sure to use detergent in warm water to clean kitchen counters, dishes, and laundry and household disinfectant when cleaning the bathrooms. However, the single most important practice in preventing infections is careful hand washing before and after client care, touching food, or using the bathroom.

What CHN intervention would best serve a vulnerable population? a. Addressing multiple health concerns, including preventive education, when clients present for treatment of an illness b. Establishing a system of networks so that clients may be referred to different services such as preventive care, acute illness care, and chronic treatment c. Providing acute care services that focus on the client's main health concern and setting up appointments at discharge for other concerns d. Referring clients to specialists to address specific health concerns

ANS: A When working with vulnerable populations, it is a good idea to arrange to have as many services as possible available in a single location and at convenient times. Comprehensive services, a "one-stop shopping" approach to care delivery, are helpful for populations experiencing multiple social, economic, and health-related stresses. This becomes especially important if clients have problems accessing health care services.

The cornerstones of public health nursing practice (select all that apply): a. Focus on the health of the entire population b. Reflect community priorities and needs c. Promote health through strategies driven by epidemiological evidence d. Are grounded in an ethic of collaboration

ANS: A, B, C Cornerstones of public health nursing practice focus on the health of the entire population, reflect community priorities and needs, promote health strategies driven by epidemiological evidence, and are grounded in social justice. Nursing is grounded in an ethic of caring

A public health nurse working with a family living in poverty recognizes that they are more likely to be exposed to environmental hazards because they have (select all that apply): a. Limited funds to pay for health care b. Poor nutrition c. Homes located closer to hazardous waste sites d. Less education

ANS: A, B, C Families living in poverty are more likely to experience environmental justice issues such as disproportionate environmental exposures. Sub-standard housing, living closer to hazardous waste sites, working in more hazardous jobs, poorer nutrition, and less access to quality health care all contribute to this issue. Although limited education is related to poverty, it is not discussed as causing an increase in environmental exposure.

1. Which of the following activities are included in the Centers for Disease Control and Prevention's school health program? (Select all that apply.) a. Ensuring a healthy school environment b. Assisting teachers with education related to health c. Encouraging nutritious school meals d. Giving immunizations to students, staff, teachers, and their families

ANS: A, B, C The federal government, through the coordination of the Centers for Disease Control and Prevention, developed a plan that school health programs should follow, including health education, physical education, health services, nutrition services, counseling, psychological and social services, healthy school environment, health promotion for staff, and family/community involvement. Unfortunately schools cannot afford to give immunizations to everyone who might want such a benefit. Education in areas other than health is the responsibility of the teachers, not the nurse.

The ethical tenets that underlie the core function of assessment are (select all that apply): a. Competency: the persons assigned to develop community knowledge are prepared to collect data on groups and populations b. Moral character: the persons selected to develop, assess, and disseminate community knowledge possess integrity c. Service to others over self: a necessary condition of what is good or right policy d. Do no harm: disseminating appropriate information about groups and populations is morally necessary and sufficient

ANS: A, B, D Service to others over self is an ethical tenet of policy development. Competency, moral character, and do no harm are the ethical tenets of assessment.

5. A school nurse describes to teachers the characteristics of an adolescent who may be thinking about drastic violence. Which of the following behaviors would suggest the adolescent was having such problems? (Select all that apply.) a. Being a gang member b. Damaging property c. Leaving the scene if another student is being bullied or hurt d. Mood swings

ANS: A, B, D Six characteristics that can help identify a student who may be thinking about drastic violence are:

10. Which of the following are the major sources of information for program evaluation? (Select all that apply.) a. Community indices b. Media reports c. Program clients d. Program providers e. Program records

ANS: A, C, E Both quantitative and qualitative methods may be used to conduct an evaluation. However, the strongest evaluation designs combine both qualitative and quantitative methods. Major sources of information for program evaluation are the program clients (especially user satisfaction information), program records (especially clinical records), and community indices (epidemiologic data).

A public health nurse is working with a migrant farm worker who has experienced an exposure to a pesticide. When researching pesticides, the nurse looks at the family of the chemical. What similarities are found among chemicals that have been placed in the same family? a. Route of entry into the body b. Actions and associated risks c. Effects that they have on the body d. Potency and toxicity

ANS: B Chemicals are grouped so its possible to understand the actions and risks associated with each group.

A nurse believes everyone is entitled to equal rights and equal treatment in society when applying: a. Distributive or social justice b. Egalitarianism c. Libertarian view of justice d. Communitarianism

ANS: B Egalitarianism is defined as the view that everyone is entitled to equal rights and equal treatment in society.

3. A new student's parents had not yet submitted an immunization record, although the nurse had sent a reminder home with the student twice. Which of the following actions should be taken by the nurse to keep the child in school? (Select all that apply.) a. Call the parents or mail another reminder. b. Report the problem to the teacher and the principal. c. Send the child home with a note saying the child cannot return until the immunization record is received. d. Suggest to the parents that if they don't have health care insurance, they may qualify for programs that provide immunizations free.

ANS: A, D There are many problems with children not being immunized or having incomplete vaccination records, especially in families who have moved many times or who may not have a regular physician. The parents may have no idea whether the child has received the required shots. Families may also be without health care insurance to pay for the immunizations, or they may have insurance that does not pay for preventive care. In these cases, they may lack the resources to pay for the immunizations themselves. Therefore, the nurse's role is to be sure parents are aware of the problem, to help them obtain the records if they have been misplaced, and to suggest ways to obtain the injections without charge, even if there is no obvious evidence that lack of funds is the problem. Telling the teacher or principal won't resolve the problem and it is illegal, immoral, and unprofessional to enter false information in a student's record.

7. Health education is often used as a strategy in working with vulnerable populations. The benefits of health education can be greatly affected by the individual's or group's: a. cycle of dependency. b. health literacy. c. level of income. d. race and ethnicity.

ANS: B A new concern for public and community health nurses is whether the populations with whom they work have adequate health literacy to benefit from health education. Health literacy is a measure of the client's ability to read, comprehend, and act on medical instructions. It may be necessary to collaborate with an educator, an interpreter, or an expert in health communications to design messages that vulnerable individuals and groups can understand and use.

1. The population group that is likely to be the MOST vulnerable is: a. children with a family history of sickle cell disease and hypertension. b. homeless pregnant teens in a substance abuse program. c. nNative Americans at risk for diabetes. d. overweight children.

ANS: B A vulnerable population group is a subgroup of the population that is more likely to develop health problems as a result of exposure to risk and to have worse outcomes from these health problems than the rest of the population. That is, the interaction among many variables creates a more powerful combination of factors that predispose the person to illness. Vulnerable populations often experience multiple cumulative risks, and they are particularly sensitive to the effects of those risks. Examples of vulnerable populations of concern to nurses are persons who are poor and homeless, people with special needs, pregnant teens, migrant workers and immigrants, individuals with mental health problems, people who abuse addictive substances, persons who have been incarcerated, persons with communicable diseases and those who are risk, and persons who are human immunodeficiency virus (HIV) positive or have hepatitis B virus or sexually transmitted diseases (STDs).

8. The community planning board is attempting to determine if the clients and health care providers affected by a recent mental health outreach initiative are satisfied that the program interventions have accomplished the program objectives and that clients have benefited from this program. The analysis model that is best designed to provide an estimate of costs to achieve an outcome is: a. cost-benefit analysis. b. cost-effectiveness analysis. c. cost-efficiency analysis. d. Multi-Attribute Utility Technique.

ANS: B An evaluation of program effectiveness may help determine whether both providers and clients are satisfied with program activities, as well as whether the program met its stated objectives. A cost-effectiveness analysis is a subset of a cost-benefit analysis and is designed to provide an estimate of the costs to achieve a given outcome. Such an analysis can answer several questions: Did the program meet its objectives? Were the clients and providers satisfied with the effects of the interventions? Are things better as a result of the interventions? In cost-benefit analysis, both the cost and outcomes are quantitative. In cost-effectiveness analysis, the outcomes are both qualitative (satisfaction) and quantitative (cost).

9. The insurer's risk manager has informed an independently contracted case manager that a liability risk related to experimental treatment and technology was identified in a recent recommendation made by the case manager. Which of the following is the best example of action by a case manager that might be associated with potential liability risk related to experimental treatment and technology? a. Inappropriately recommending that treatment be curtailed when treatment was actually needed b. Failing to apply the contractual definition of "experimental" treatment found in the client's insurance policy c. Substituting the case manager's clinical judgment for that of the insurer's medical director d. Upcoding intensity of care or intervention requirements

ANS: B For case managers, liability concerns exist when three conditions are met: (1) the provider had a duty to treat (i.e., provide reasonable care) (2) a breach of contract occurred through an act of omission and (3) the act of omission caused injury or damage to the client. Five general areas of risk are liability for managing care, negligent referrals, use of experimental treatment or technology, breach of confidentiality/security, and fraud and abuse. With regard to the use of experimental treatment and technology, the case manager is obligated to apply the contractual definition of "experimental" treatment found in the client's insurance policy (certificate of coverage), because this a legally binding document between the insurer and the insured.

9. An unemployed individual with acquired immune deficiency syndrome (AIDS) develops recurrent opportunistic infections that require repeated visits to the health clinic and the purchase of various medications to combat the infections and treat their associated side effects. This best demonstrates how the stress caused by poor health status can be related to: a. barriers to access. b. cascade effects. c. cumulative risk. d. socially based inequity.

ANS: B Poor health status creates stress. Vulnerable populations cope with multiple stressors, and this creates a cascade effect, with chronic stress likely to result. Chronic stress can lead to feelings of hopelessness. Hopelessness results from an overwhelming sense of powerlessness and social isolation that contributes to a continuing cycle of vulnerability. Each factor, such as lack of employment, lack of insurance or underinsurance, the disease process, transportation challenges, limited resources, and complications of treatment can predispose individuals to poor health status. The outcomes of vulnerability reinforce the predisposing factors, which leads to more negative outcomes. This cycle of vulnerability must be broken in order for vulnerable populations to change their health status.

9. Nurses should consider opportunities for population-focused practice that result from the rapid transformation of health care delivery from a medical model to a health promotion/disease prevention model. An example of such opportunity is: a. operator of a nurse practitioner-run urgent care center in a major retail location. b. director of clinical services spanning inpatient and community-based settings that provide a wide range of services to the populations seen by the system. c. clinical director of a home health agency. d. school nurse position in the local high school.

ANS: B The new focus on populations, coupled with the integration of acute, chronic, and primary care occurring in some health care systems, is likely to create new roles for individuals, including nurses, who will span inpatient and community-based settings and focus on providing a wide range of services to the populations served by the system. Such a role might be director of client care services for the health care system, who has administrative responsibility for a large program area. There will be a demand for individuals who can design programs of preventive and clinical services to be offered to targeted subpopulations and for those who can implement such programs.

4. When planning a new community health center, a nurse will integrate knowledge of the nursing process and program management. The nurse's initial and most critical step for funding purposes would be: a. finding the lay leaders in the community. b. identifying the target population's health problems and needs. c. outlining the major causes of mortality in the community. d. prioritizing the community's problems.

ANS: B The program management process is parallel to the nursing process and consists of a rational decision making system designed to help nurses know when to make a decision to develop a program (assessment and problem identification)

2. A community health nurse is conducting a community assessment as part of a program planning initiative and is seeking a tool that is low cost, allows clients to participate in identification of need, and would stimulate community support for the program. The nurse would most likely use which of the following? a. Community forum b. Focus group c. Indicators approach d. Survey

ANS: B There are several types of needs assessment tools, including community forums, focus groups, key informants, indicators approach, survey of existing agencies, and general surveys. Both community forums and focus groups are low-cost tools. The focus group provides clients with the opportunity to participate in identification of needs and can help increase community support for a program. The focus group method has several disadvantages, such as being time consuming to carry out and tending to focus on irrelevant or political "hot button" issues. Leading focus groups requires strong skills in group process to maintain the focus of the group.

When using Anderson and McFarlanes model to complete a community health assessment, a nurse would collect information about the _____ the community. a. Resources available to promote health in b. Interactions among subsystems in c. Physical environment of d. Demographics of

ANS: B A community health assessment must include information about the subsystems and interactions of the total community with the systems external to it.

When public health nurses conduct an assessment of a communitys health, they a. Define one problem that will be the focus for a year. b. Assess a social network of interacting individuals usually in a defined territory. c. Minimize the effects of health risks and hazards. d. Intervene at the population-level by changing laws and regulations.

ANS: B A community is defined as a social network of interacting individuals, usually concentrated in a defined territory. The community assessment generally results in a lengthy list of community problems and issues. Intervention and minimization do not take place during the assessment phase.

A group of nursing students forms a student nursing association to provide support while they are enrolled in a nursing program. This is known as a community of: a. Place b. Special interest c. Problem ecology d. Function

ANS: B A community of special interest brings members together with common concerns and interests.

A CHN suspects that a new client may have a substance use disorder. When getting the health history of the client, the CHN needs to keep in mind that the client may not admit to drug use. What is a primary symptom of substance use disorder? a. Confusion b. Denial c. Forgetfulness d. Mental status changes

ANS: B A primary symptom of substance use disorder involves a refusal to acknowledge that a substance use problem exists.

The community leaders in a lesser-developed country decide not to tell the citizens of a small village about a chemical spill at a major industrial facility that could produce harmful effects. Which principle are they violating? a. Policy b. Advocacy c. Caring d. Virtue

ANS: B Advocacy requires that the community be properly informed, and this was violated in the above scenario.

16. A disaster has occurred in the community. Which of the following actions should be taken by the school nurse? a. Continue activities as much as possible as if nothing had happened b. Continue to assess for shock and stress c. Help teachers discuss the disaster with their class d. Maintain school routines and activities

ANS: B After a disaster, the school nurse has many responsibilities—for instance, continuing to assess the school community for the presence of shock and stress, encouraging parents to minimize how much their children view the disaster coverage on TV, providing grief counseling, continuing to communicate with the children, parents, and school personnel, and following up with assessment of children for anxiety, depression, regression, and posttraumatic stress disorder.

17. Which of the following is the leading cause of children being absent from school because of a chronic illness? a. Allergies b. Asthma c. Diabetes d. Upper respiratory infections

ANS: B Asthma is the leading cause of children being absent from school because of a chronic illness. URIs are an acute problem, not a chronic one.

10. The school nurse has arranged for volunteers to help check each child's hearing and vision. Any child that the volunteers feel did not "pass" will be sent to the nurse for follow-up. The nurse will then send a note to the parents that a physician should be seen. Which of the following levels of prevention is being implemented? a. Primary b. Secondary c. Tertiary d. Both primary and secondary

ANS: B Because secondary prevention involves caring for children when they need health care, this is the largest responsibility for the school nurse. This includes caring for ill or injured students and school employees. It also involves screening and assessing children and referral to appropriate health agencies or providers.

I yell at them to stop being stupid, and if they don't, I tell them that the boogeyman will steal them away at night if they don't obey." c. "I use 'time out' when my child acts out or is naughty. Sometimes, my child doesn't cope well with this, but I am persistent." d. "When my child misbehaves, I distract him and try to focus his attention on other things. If he throws a tantrum, I just pick him up and leave the store or show or wherever we may be."

ANS: B Emotional abuse involves extreme debasement of feelings and may result in the child feeling inadequate, inept, uncared for, and worthless. A parent is emotionally abusing the children by yelling at them ("to stop being stupid") and frightening them ("boogeyman will steal them away at night").

An orderly process that considers ethical principles, client values, and professional obligations is: a. Accountability b. Ethical decision making c. Moral principles d. Code for Nursing Practice

ANS: B Ethical decision making is defined as an orderly process that considers ethical principles, client values, and professional obligations.

To effectively complete the evaluation phase, the nurse must have: a. Demonstrated the ability to improve the health of the participants b. Developed measureable objectives and goals before implementation c. Encouraged full participation by community members d. Improved the health of the population through the program

ANS: B Evaluation begins in the planning phase, when goals and measurable objectives are established and goal-attaining activities are identified. After implementing the intervention, only the meeting of objectives and effects of the intervening activities have to be assessed.

A nurse in the 1960s would have referred to which code of ethics to guide ethical decision making? a. Nightingale Pledge b. Code for Professional Nurses c. Code of Ethics for Nurses with Interpretive Statements d. International Council of Nurses (ICN) Code of Ethics for Nurses

ANS: B Florence Nightingale lived in the 1800s. The Code for Professional Nurses was adopted in 1950, the Code of Ethics for Nurses with Interpretive Statements was adopted in 2001, and the International Council of Nurses (ICN) Code of Ethics for Nurses was adopted in 2000.

Which of the following is an example of a goal? a. Mr. Williams will look at his stoma without disgust each time his ostomy bag comes off. b. Mr. Williams will be able to independently take care of his ostomy bag within three months. c. Mr. Williams will gather all ostomy supplies correctly each time his ostomy bag needs to be changed. d. Mr. Williams will successfully describe to the nurse how to care for his ostomy when he is asked.

ANS: B Goals are broad, long-term expected outcomes. The correct answer describes something that will happen over a long period of time. The other choices describe objectives.

A CHN who works at a clinic for homeless persons wants to institute a more efficient treatment for chronic wounds. What CHN action offers the best way to improve outcomes for these clients? a. Administer antibiotics to all homeless persons with chronic, nonhealing wounds. b. Facilitate daily access to a room with soap, water, and bandages. c. Provide free bandaging supplies to clients at each clinic visit. d. Regularly monitor the wound condition of clients.

ANS: B Health problems faced by homeless people often are related directly to poor access to preventive health care services. The CHN can implement tertiary prevention by designating a wound room in which clients can carry out wound care activities taught during clinic visits. Only infected wounds would need treatment with antibiotics, not all wounds. Free bandaging only addresses one part of the problem, and monitoring does not improve outcomes

Nurses who use Healthy People 2020 as a guide for education: a. Focus on avoiding cigarette smoking and using alcohol in moderation b. Educate clients using primary and secondary levels of prevention c. Use Blooms taxonomy when planning educational objectives d. Design health fairs aimed at individuals

ANS: B Healthy People 2020 focuses on primary and secondary prevention. Understanding the three learning domains is crucial in providing effective health care.

1. Which of the following statements best describes the most essential difference between home health care and acute client care? a. Acute client care is individualized for the client and family. b. Home health care is provided in the client's environment. c. Reimbursement for home health care varies from that of care provided in institutions. d. The focus of acute client care is on community health.

ANS: B Home health differs from other areas of health care in that health care providers practice in the client's environment. All nurses give individualized care to clients.

2. A nurse has just received word that Medicaid will reimburse for care provided to a homeless man with schizophrenia who is afraid to come to the clinic to receive health care. Which of the following best describes this type of nursing? a. Community-oriented nursing b. Home health nursing c. Hospice nursing d. Private duty nursing

ANS: B Home health nursing is provided in the client's environment, regardless of where that may be. "Home" may be a house, apartment, trailer, boarding house, shelter, car, makeshift shelter under a bridge, or cardboard box.

8. Which of the following is a potential advantage of hospital-based home health agencies over other types of home care agencies? a. Administration and management benefits are gained from the expertise of two boards of experts. b. Continuity of care is enhanced. c. Health promotion and illness prevention concerns take precedence over acute care and rehabilitation. d. They are eligible for tax exemptions through affiliated agencies.

ANS: B Hospital-based agencies emerged in response to the recognized need for continuity of care from the acute care setting.

Which behavior would the nurse anticipate when working with an individual with low literacy? a. Asking for additional clarification of materials b. Requesting to read the information later c. Having a high level of motivation d. Being overly dependent on others

ANS: B Individuals with a limited literacy may have a limited vocabulary and general knowledge and do not ask for clarification. They may focus on details and deal in literal or concrete concepts versus abstract concepts. They may select responses on a survey without necessarily understanding them and may be unable to understand math. They may have a low motivation to engage in learning or may drop subtle clues that they cannot read by stating they will look at information later or take it home.

After evaluating learning needs, what is the next step the nurse should take in developing an educational program? a. Consider any potential barriers to learning. b. Establish goals and objectives for the program. c. Select appropriate materials for the program. d. Assess the dynamics of the group.

ANS: B Instructional objectives need to be evaluated before a teaching program is designed.

Public health interventions are implemented with: a. Legislators, policy makers, and community leaders b. Individuals and families, communities, and systems c. Children, adolescents, and adults d. Health departments, public health agencies, and visiting nurses associations

ANS: B It is important to know that public health nurses work with individuals and families, communities, and systems. The other answers may have true parts, but the second option lists the overall groups where public health nurses are intervening.

How can a community health nurse apply the Ethical Principles for Effective Advocacy? Select all that apply. a. Act in the health care providers best interest. b. Keep the client (group, community) properly informed. c. Maintain client confidentiality. d. Carry out instructions with diligence and competence.

ANS: B Keep the client (group, community) properly informed, maintain client confidentiality, and carry out instructions with diligence and competence are ethical principles for effective advocacy

Which ethical principle requires doing no harm? a. Respect for autonomy b. Non-maleficence c. Beneficence d. Distributive justice

ANS: B Non-maleficence refers to doing no harm.

however, I have decided that I am going to talk to him about birth control, just in case." b. "I have found that being very strict and checking on my daughter whenever she is out are the best ways to prevent trouble." c. "I plan to sit down with my daughter and have an honest talk about sexuality and potential risks." d. "I will start spending more time with my teens when I get home from work."

ANS: B Parenting style is a contributing factor to adolescent pregnancy. Parents who are extremely demanding and controlling or neglectful, and who have low expectations, are the least successful in instilling good values in their children. Children of parents who are neglectful are the most sexually active, followed by children of parents who are very strict. Furthermore, parents who discuss birth control, sexuality, and pregnancy with their children can positively influence delaying initiation of sexual activity and use of effective birth control. Parents who do not talk about sexuality with their teens may find them more at risk for sexual permissiveness and unwanted pregnancies.

A community that has residents who are very involved in its activities and encourage activities that promote the health of the community displays which condition of community competence? a. Commitment b. Participation c. Articulateness d. Effective communication

ANS: B Participation is an essential condition of community competence defined by active, community-oriented involvement.

A leader controls members through rewards and often keeps members in the dark about the goals and rationale behind prescribed actions. What type of leadership does this describe? a. Democratic b. Patriarchal c. Socialist d. Communication structure

ANS: B Patriarchal or paternal style is authoritative, winning respect and dependence through parent-like devotion.

Which is considered a nonpoint source of pollution? a. Hazardous waste site b. Animal waste from wildlife c. Chlorine poured down a well d. Stagnant water

ANS: B Point source means a single place from which the pollutant is released into the environment, whereas nonpoint source implies a more diffuse source of pollution.

5. In the agency, one of the nurses spent all available time visiting a group of persons with mental health problems who were trying to remain functional in the community. Which of the following types of assignments did the nurse most likely have? a. Home-based primary care b. Population-focused home care c. Proprietary home care d. Transitional care

ANS: B Population-focused home care is directed toward the needs of specific groups of people, including those with high-risk health needs such as mental health problems, cardiovascular disease, or diabetes, families with infants or young children, and older adults. Such care commonly includes structured regular visits with assessment protocols, focused health education, counseling, and health-related support and coaching.

What is a primary cause of vulnerability? a. Breakdown of family structures b. Poverty c. Prejudice d. Social isolation

ANS: B Poverty is a primary cause of vulnerability. The lack of financial resources may cause some people to not seek preventive health services. This leaves them vulnerable and with increased risk of experiencing the effects of preventable illnesses.

What CHN action would best ensure long-term positive health outcomes in pregnant teens from low-income groups and their children? a. Help teen mothers learn about body changes during pregnancy. b. Develop programs that enable teen mothers to complete their education. c. Offer courses in proper care of babies. d. Monitor pregnant teens for early detection of problems in pregnancy.

ANS: B Programs that enable the teen mother to complete her education increase her chances for a better future and improved health care over the long term, across the lifespan. Issues to discuss include education and career plans, family finances and qualifications for outside assistance, and personal values about pregnancy and parenting at this time in her life.

A nurse collects data about seat belt usage by interviewing key informants and observing behaviors in the community. What type of data is being collected? a. Quantitative b. Qualitative c. Focus-groups d. Survey

ANS: B Qualitative data is collected through interviews and observation.

Which statement fits the Liberal Democratic Theory of John Rawls? a. Rejection of any idea that societies, states, or collectives of any form can be the bearers of rights or can owe duties. b. Inequalities result from birth, natural endowment, and historic circumstances. c. Everyone has a right to private property. d. Government should be limited.

ANS: B Rawls acknowledges that inequities are inevitable in society, but he tries to justify them by establishing a system in which everyone benefits, especially the least advantaged. This is an attempt to address the inequalities that result from birth, natural endowments, and historic circumstances. The other choices relate to libertarianism.

6. Which of the following best describes services that are offered at a school-based health center? a. Employee care at a discounted cost at the school b. Care to others in the community c. Sex education, birth control, family planning, and care throughout pregnancy d. Referral and networking with other health care services in the community

ANS: B School-based health centers give care not only to students but also to other persons in the community. They may provide social services, daycare, job training, and educational counseling in addition to the medical and nursing care, mental health counseling, and dental care seen in smaller school-based centers.

Which level of prevention is a CHN practising when she offers homeless clients yearly tuberculosis (TB) screening and free treatment for those who test positive? a. Primary prevention b. Secondary prevention c. Tertiary prevention d. Secondary and tertiary prevention

ANS: B Screening homeless persons for TB and providing medications to those who test positive are examples of secondary prevention. The TB screening identifies the disease in its early stages. Medications work to prevent further development of the disease.

CHNs at a clinic for homeless people are concerned that clients rarely return for follow-up after their TB skin (Mantoux) tests. What policy would be the most appropriate one for addressing this situation? a. Call all homeless clients 48 hours after testing to remind them to return to the clinic for follow-up. b. Have the homeless persons read the test result themselves and then mail in the results on a postage-paid card coded to protect privacy. c. Readminister the test if the client returns later than scheduled for follow-up. d. Routinely refer all homeless clients for chest X-rays.

ANS: B Secondary preventive activities are aimed at reducing the prevalence or pathological nature of a condition. They involve early diagnosis, prompt treatment, and the limitation of disability. CHNs can work with homeless and near-homeless aggregates to provide education about existing services and strategies for influencing public policy that will provide more comprehensive services for homeless and near-homeless persons. If necessary, CHNs should develop a method for homeless individuals to read the reaction to the TB skin test themselves and send the results to the facility where the skin test was administered.

If a nurse wanted more information on indoor air quality, which website would be most helpful? a. The National Institutes of Health (NIH) b. The American Lung Association c. Right to Know d. The Occupational Safety and Health Administration (OSHA)

ANS: B Sources of information about air quality include the Environmental Protection Agency (EPA) and the American Lung Association.

An occupational health nurse works with an employer to develop a workplace wellness program for its employees. This intervention is occurring at the _____-level of practice. a. Individual/family b. Systems c. Community d. Government

ANS: B Systems-level of practice consists of changing laws, policies, and practices that influence population-based issues.

13. An upset mother calls the school nurse and says, "How dare you say my child has lice? My child is clean and I keep a clean house! You've obviously made an error." Which of the following would be the best response by the nurse? a. "I'm sorry you're upset, but your child cannot return to school until this problem is addressed." b. "Most lice are found in clean hair. Children often share combs. Let me tell you how to fix the problem." c. "You may have been traveling. Lice are often found in motels." d. "I'm sure you're correct one of my volunteers probably made an error. I'll recheck."

ANS: B The nurse must reassure the mother that no insult was intended; in fact, lice are most often found on middle-class children with clean hair. Lice travel easily when children share items such as combs or other property in school. Lice are not life-threatening, and the necessary shampoo and other items to treat lice are widely available over the counter.

To analyze the problem of increased adolescent alcohol use in the community, a nurse brings together several groups of people to address the issue. These groups come together at the same time to work on identifying potential causes of the problem, but do not interact during the process. Which model is the nurse applying? a. Delphi technique b. Program planning model c. Community-as-partner model d. Community empowerment model

ANS: B The program planning model is known as the nominal group process. It makes the most of the contributions by various groups with diverse interests, skills, and knowledge. These groups work in the presence of one another but do not interact.

3. A school health nurse is requested by the board of education to assist in choosing new playground equipment for an elementary school that meets safety standards. Which of the following best describes the nurse's role in this scenario? a. Case manager b. Consultant c. Counselor d. Health educator

ANS: B The school nurse is the person best able to provide health information to school administrators, teachers, and parent-teacher groups. As a consultant, the school nurse can provide professional information about proposed changes in the school environment and their effect on the health of the children. The nurse also can recommend changes in the school's policies or ask community organizations to help make the children's schools healthier places.

8. Which of the following best describes the primary reason that school health nurses spend so much time on educational programs that teach children the importance of water and fire safety, using a seatbelt in the car, and wearing a helmet when biking or skateboarding? a. Because children won't know if someone doesn't tell them b. Because injuries are the leading cause of death in children and most injuries are preventable c. Because it is a dangerous world and someone has to warn children about the dangers d. Because teaching is easy and more fun than passing out bandages and documenting care

ANS: B The school nurse, as the trusted person at school, is able to quickly give information to help prevent injuries from occurring, since most injuries are preventable. Injuries are the leading cause of death in children and teenagers.

A nurse provides counseling to an obese client about the importance of good nutrition and regular exercise with the intention of helping the client avoid future chronic diseases associated with obesity. What client system and focus of care are being applied in this situation? a. Family level health promotion b. Individual level illness prevention c. Aggregate level illness prevention d. Individual level health promotion

ANS: B The simplest level of the client system is the individual. The focus of care is illness prevention, directed at disease or disability prevention.

When would it be appropriate for a nurse to use a Geographic Information System (GIS)? a. Recording client data collected at a foot clinic b. Determining neighborhoods that have an increased incidence of lead poisoning c. Evaluating effectiveness of a farm safety program d. Scheduling health promotion programs in the community

ANS: B The use of a Geographic Information System (GIS) allows the public health nurse to apply the principles of epidemiology into practice. GIS allows nurses to code data so that it is related spatially to a place on earth and is helpful in determining concentrated areas for incidence of disease and illness.

During a group counselling session for perpetrators of intimate partner violence, which client statement indicates a lack of insight into his violent behaviour? a. "I have been taking out my frustrations about work on my girlfriend." b. "I love my girlfriend and didn't want to hurt her it was an accident." c. "It might be a good idea for me to temporarily leave the house when I feel I am getting angry." d. "When I drink alcohol, I become more abusive toward my girlfriend."

ANS: B Violence is defined as those nonaccidental acts that result in physical or psychological injury. Although the client may now be feeling remorse, at the time that the violent act was committed against the girlfriend, his intent was to inflict harm.

it was an accident." c. "It might be a good idea for me to temporarily leave the house when I feel I am getting angry." d. "When I drink alcohol, I become more abusive toward my girlfriend."

ANS: B Violence is defined as those nonaccidental acts that result in physical or psychological injury. Although the client may now be feeling remorse, at the time that the violent act was committed against the girlfriend, his intent was to inflict harm.

An example of an ethical dilemma is: a. Whether or not to set up a community health center in a rural area b. Allocating resources in a natural disaster c. Deciding to withdraw care on a hospice patient d. Applying the principles of Florence Nightingale in Bangladesh

ANS: B When resources are scarce, a dilemma may exist as to how to allocate them.

Which method of data collection is being used as a nurse gathers information about the condition of homes, size of lots, neighborhood hangouts, road conditions, and modes of transportation? a. Participant observation b. Windshield survey c. Survey d. Informant interviews

ANS: B Windshield surveys are the motorized equivalent of simple observation.

Which data would be useful for a nurse to collect when assessing safety in a community? Select all that apply. a. Number of billboards in the area b. Interviews with health care providers who are familiar with the community c. Observation of community members d. Nurses own observations

ANS: B, C, D Other nurses, social workers, and health care providers, community members, and the nurses own observations are reliable sources of information about the safety of an area

Which approach(s) can a nurse use when assessing environmental health risks? Select all that apply. a. Ask legislators to provide a list of environmental pollutants in the area. b. Develop a list of exposures associated with urban, rural, or suburban settings. c. Assess the risk by medium such as air, water, soil, or food. d. Divide the environment into functional locations: home, school, workplace, and community.

ANS: B, C, D The 2nd, 3rd, and 4th options are ways a nurse can assess the environment.

1. Public health nursing specialists are interested in which of the following topic(s)? (Select all that apply.) a. Educational materials for individuals with HIV/AIDS b. Evaluation of an outreach program for at-risk pregnant teenagers c. Community subpopulations with high rates of type 2 diabetes d. New technologies to monitor diabetes e. Prevalence of hypertension among various age, race, and gender groups

ANS: B, C, E Public health specialists often define problems at the population or aggregate level as opposed to the individual level. At the population level, public health specialists are usually concerned with more than one subpopulation and frequently with the health of the entire community.

1. Vulnerable population groups are those that, in comparison with the population as a whole, have which of the following characteristics? (Select all that apply.) a. Better access to health care services but poor health outcomes b. Greater likelihood of exposure to risk factors c. Multiple risk factors but equal health outcomes d. Worse health outcomes

ANS: B, D Vulnerable populations are defined as those at greater risk for poor health status and health care access. In health care, risk is an epidemiologic term indicating that some people have a higher probability of illness than others. The natural history of disease model explains how certain aspects of physiology and the environment make it more likely that a certain individual will develop a particular health problem. However, not everyone who is at risk develops health problems. Some individuals are more likely to develop the health problems for which they are at risk. A vulnerable population group is a subgroup of the population that is more likely to develop health problems as a result of exposure to risk or to have worse outcomes from these health problems than the rest of the population.

6. A child has multiple disabilities, and caring for the child has been both expensive and time consuming for the school. Once the child turns 16, which of the following actions should be taken by the nurse? (Select all that apply.) a. As an adult, the child is no longer eligible for school services without charge. b. The school must continue to provide needed appropriate education for the child. c. The school can exclude the child from any extra special activities. d. The school should prepare an updated individualized education plan.

ANS: B, D Educational services must be offered by the schools for all disabled children from birth through age 22 years. Children cannot be excluded from activities because of a disability. The school must always develop an individualized education plan for each child and update it at appropriate intervals. Turning 16 does not make a child an adult.

2. Which of the following best explains why the federal government is beginning to fund school-based health centers? (Select all that apply.) a. These centers help young children avoid becoming addicted to drugs while still in elementary school. b. Attendance and learning are higher in schools with health clinics. c. These centers help keep children in school longer by distributing birth control and thus avoiding pregnancies. d. Many children have no other source of health care services.

ANS: B, D The U.S. government began funding school-based health centers essentially because many school children may not receive health care services otherwise. These are family-centered, community-based clinics run within the schools. Certainly, avoiding pregnancy and drug addiction are among the goals of school-based health centers, but these are not reasons the government began funding them.

6. A community health nurse involved in care management would most likely: a. develop, conduct, and evaluate health teaching programs in primary care. b. manage the staff at a free clinic. c. monitor the health status, resources, and outcomes for an aggregate. d. provide immunizations to migrant workers.

ANS: C Care management is a continuing process in which a case manager establishes systems and monitors the health status, resources, and outcomes for an aggregate—a targeted segment of the population or group.

4. The role and goals of the community health nursing practice can best be described as: a. community-based interventions aimed at promoting, preserving, and maintaining the health of populations residing in institutional facilities such as nursing homes. b. education of nurses and other staff working in community-based and community-oriented settings to improve the overall effectiveness of their programs to meet client needs. c. population-level strategies aimed at promoting, preserving, and maintaining the health of populations through the delivery of personal health care services to individuals, families, and groups in an effort to improve the health of the community as a whole. d. activities targeted at improving the health status of clients served by community-based health service agencies such as hospice and home health agencies.

ANS: C Community health nursing practice is the synthesis of nursing theory and public health theory applied to promoting, preserving, and maintaining the health of populations through the delivery of personal health care services to individuals, families, and groups. The focus of community health nursing practice is the health of individuals, families, and groups and the effect of their health status on the health of the community as a whole (individual to families to groups to community flow). This is different from public health nursing, which is the synthesis of nursing theory and public health theory applied to promoting and preserving the health of populations. The focus of public health nursing practice is the community as a whole and the effect that the community's health status, including health care resources, has on the health of individuals, families, and groups (community to groups to families to individual flow). Both community health and public health nursing are considered to be community-oriented practices involving free-living (noninstitutionalized) clients. Community-based nursing practice is setting specific, and care is provided to clients where they live (home health or hospice nursing, community-based clinic), work (occupational health nursing), and/or attend school (school nursing). The emphasis of community-based nursing practice is acute and chronic care (illness care) and the provision of comprehensive, coordinated, and continuous services, usually within a specialty area.

1. Advantages of community health program planning include ensuring that available resources are used to meet the needs of the population and: a. applying for grants. b. identifying clients and soliciting board members' support. c. identifying resources, activities, and needs. d. increasing the visibility of the program.

ANS: C Community health program planning is population focused and puts the well-being of the public above private interests. Systematic planning for meeting the needs of populations in a community has benefits for clients, nurses, employing agencies, and the community. It ensures that available resources are used to address the actual needs of people in the community, focuses attention on what the organization and health provider are attempting to do for clients, identifies resources and activities that are needed to meet the objectives of client services, reduces role ambiguity by giving responsibility to specific providers to meet program objectives, reduces uncertainty within the program environment, increases the ability of the provider and agency to cope with the external environment and anticipate events, allows for quality decision making, and provides better control over the actual program results. Identifying clients, soliciting board support, applying for grants, and promoting the actual program are steps in the implementation phase of program management.

9. The nurse program manager is determining the direct client care costs as well as the cost of indirect nursing activities for home visits for a home health agency. Analysis of this information along with nursing workload information and client needs can best provide an agency evaluation measure for: a. program decision making. b. cost-effectiveness. c. cost-efficiency. d. perceived value.

ANS: C Cost-efficiency analysis determines the actual cost of performing a number of program services, both direct and indirect, by addressing the productivity of the workforce in achieving specific objectives. The relationship between direct and indirect activities, workforce, caseloads, client needs, and actual costs determines the productivity of the program.

5. After performing an assessment of a client seeking treatment for hypertension at the local free clinic, the nurse informs the client that the family's children may qualify for enrollment in the state children's health insurance program. The nurse provides the enrollment forms and reviews them with the client, emphasizing how to apply for the benefits. This best exemplifies which principle for intervening with vulnerable populations? a. Carrying out primary prevention b. Setting family-centered, culturally sensitive goals c. Trying to minimize the "hassle factor" d. Using the MAP-IT approach

ANS: C Nurses empower clients by helping them acquire the skills needed to engage in healthy living and to be effective health care consumers. Vulnerable individuals and families may need to go to multiple agencies to find the services for which they qualify, because agencies tend to be specialized instead of comprehensive in their service approach. More agencies are needed that provide comprehensive services with nonrestrictive eligibility requirements. Outreach and case finding are important roles for the nurse in reducing health disparities. One of the principles of intervening with vulnerable populations is to try and minimize the "hassle factor."

3. Vulnerability is multidimensional, and one of the primary contributors to vulnerability is: a. gender. b. race and ethnicity. c. resource limitations. d. urban or rural residency.

ANS: C Resource limitations are strongly related to health. Lack of adequate social, educational, and economic resources make people more vulnerable and more likely to experience health disparities, and poverty is a primary cause of vulnerability. A correlation has been found between individual indicators of socioeconomic status (e.g., income, education, and occupational status) and a range of health indicators (e.g., morbidity and mortality resulting from various health problems). Not only do individual-level socioeconomic characteristics seem to matter, but population-level characteristics such as income inequality also make a difference. Resource limitations affect the individual's ability to show resilience in the face of problems and crises. Resource limitations may also place individuals and families at risk because of substandard housing, impoverished neighborhoods, and hazardous environments. Although race has been correlated with poor health outcomes, poverty seems to be a key contributing factor for minority populations. Poverty is more likely to affect women and children than other groups.

2. A registered nurse is seeking a position as a public health nurse. In reviewing the job description, the nurse would expect to find a description of a position that focused on functions such as: a. monitoring pregnant teenagers for symptoms of complications of pregnancy. b. offering free hypertension screening and treatment referral at local health fairs to low-income, uninsured, community members. c. partnering with local seasonal farmworkers to design a program aimed at preventing illness and injury, and advocating for this population with local political and community leaders. d. preventing injury among a population of elderly residents in an assisted living facility and treating residents' chronic illnesses.

ANS: C The scope of practice of public health nurses is population focused and community oriented, with a primary emphasis on population-level interventions that target strategies for health promotion and disease prevention. In addition, public health nursing is concerned with the health of all members of a population or community, particularly vulnerable populations, and uses political processes as a major intervention strategy.

5. A nurse who is the program director for a new antismoking campaign is developing a written program plan that will include the program's goals, priorities, objectives, budget, and timelines. Before implementation of the program, the written program plan should also address which of the following? a. Cost-benefit analysis b. Perspectives on the program c. Plan for the evaluation process d. Process evaluation

ANS: C When the planning process begins, the plan for evaluating the program should also begin. Everyone who will be involved in implementing a program should be given the opportunity to play a role in planning for program evaluation. Assessment of need is one component of evaluation. Once needs have been established and the program is designed, the nurse must continue plans for program evaluation, such as ongoing process evaluation (formative evaluation) and summative evaluation.

When writing a community-level diagnosis, Among refers to the: a. Specific problem or health risk faced by the community b. Characteristics of the community c. Community client experiencing the risk or problem d. Likelihood that the problem will be solved

ANS: C Among identifies the specific community client with whom the nurse will be working to address the identified problem.

17. A nurse completes a self-assessment of performance as part of the home health nurse's annual evaluation. Which of the following terms best describes this activity? a.Collaboration b. Quality of care c. Performance appraisal d. Resource utilization

ANS: C As part of a performance appraisal, the home health nurse evaluates his or her own nursing practice in relation to professional practice standards, scientific evidence, and relevant statutes, rules, and regulations.

A nurse fulfills the environmental health competency of assessment and referral when: a. Advocating for public policy changes b. Understanding policy framework and major pieces of legislation c. Completing an environmental health history d. Describing the scientific principles about environmental health

ANS: C Assessment is always an important element of the nursing process. The third option is an example of the assessment phase of the nursing process.

A CHN is orienting a new recruit to a health clinic that primarily serves vulnerable populations. Which statement by the CHN indicates a need for additional information? a. "If a client who does not speak English comes in, you must obtain an interpreter right away." b. "We try to take care of as many problems as possible in one visit, so when you check the client in, ask about additional concerns." c. "You will like working with Filipino immigrants because they have close-knit family structures." d. "You will need to assist the client by scheduling any referral or follow-up appointments."

ANS: C Assumptions are not helpful. Each person and family should be assessed individually. No two people or groups are alike. Both good and bad stereotyping can create problems. For example, even though Filipino families are generally close knit, by assuming that all Filipino families are this way, clinic care providers will likely miss recognizing such issues as family violence.

A public health department makes sure that the essential community-oriented health services are available in the community. Which of the following core public health functions is being implemented? a.Policy development b.Assessment c.Assurance d.Scientific knowledge-based care

ANS: C Assurance focuses on the responsibility of public health agencies to ensure certain activities have been appropriately carried out to meet public health goals and plans. Policy development seeks to build constituencies that can help bring about change in public policy. Assessment includes activities that involve collecting, analyzing, and disseminating information on both the health status and the health-related aspects of a community or a specific population. Public health is based on scientific knowledge, but is not a core function. DIF:Cognitive level: ApplyingREF:p. 14 TOP: Nursing process: Assessment MSC: NCLEX: Health Promotion and Maintenance

Public health administrators in a community provide a health department to serve an indigent population of immigrants providing translators on certain days of the week. This is an example of: a. Policy b. Quality c. Assurance d. Libertarian philosophy

ANS: C Assurance refers to the role of public health in making sure that essential community-oriented health services are available, which may include providing essential personal health services for those who would otherwise not receive them.

13. A home health nurse is caring for a client who has right-sided paresis secondary to a stroke. Which of the following would be the best approach for the nurse to take? a. Arrange for private duty nurses to assist the client with daily needs. b. Assist the client with activities of daily living. c. Teach the client to participate in self-care activities. d. Teach the family how to care for the client.

ANS: C Because home health care is often intermittent, and because a reliance on others is not always possible over the long term, a primary objective for the nurse is to facilitate self-care so that clients may remain in their home. This allows clients to have some control over their life and can help prevent hopelessness and a loss of self-esteem. Although assistance may be needed, such assistance should come after helping clients to help themselves.

What action can a nurse take on an individual level to reduce pollution in the environment? a. Provide a tax incentive to factories that do not pollute. b. Make laws related to allowed levels of pollution in the area. c. Choose a less-polluting car. d. Move to an area with less pollution.

ANS: C Citizens can reduce air pollution by doing their part, which can include choosing less-polluting cars.

Which group would be likely to demonstrate the most cohesion? a. A group with several leaders b. A group with diverse attitudes and values c. A group with a common goal d. A group with efficient members

ANS: C Cohesion is the attraction between individual members and between each member and the group. Group effectiveness improves as members work together toward group goals while still satisfying the needs of individual members.

A social marketing campaign urging community members to avoid driving motorized vehicles after consuming alcohol is implemented in a local community. This intervention is occurring the _____-level of practice. a. Individual/family b. Systems c. Community d. Government

ANS: C Community-level interventions care carried out with the community as a whole.

A nurse surveys the school nurses in a community to determine their roles in the schools because this data is not available. The nurse is engaging in the process of data: a. Collection b. Gathering c. Generation d. Interpretation

ANS: C Data generation is the process of developing data that do not already exist by interacting with community members or groups. This data includes information about a communitys knowledge and beliefs, values and sentiments, goals and perceived needs, norms, problem-solving processes, power, leadership, and influence structures.

Epidemiology: a. Is a science that studies the poisonous effects of chemicals b. Explains the association between learning disabilities and exposure to lead-based paint at the cellular level c. Helps nurses understand the strength of the association between exposure and health effects d. Is a method for tracking the prevalence of a disease

ANS: C Epidemiology studies the incidence and prevalence of disease, helping nurses understand the strength of the association between exposure and health effects.

Which data source provides information about the function of the community? a. Maps b. Census data c. State departments, business and labor, local library d. Civic groups

ANS: C Function relates to production, distribution, and consumption of goods and services.

A nurse is implementing an educational program about the importance of being physically active. Which step would the nurse complete first? a. Provide learning guidance b. Present the stimulus c. Gain the learners attention d. Ask learners to recall prior learning

ANS: C Gaining the learners attention must happen first before learning can take place.

What definition accurately reflects the meaning of the term health inequities? a. Health inequities are the accumulation of multiple factors that lead to poor health. b. Health inequities occur when people are more inclined to become ill and usually do not seek appropriate care. c. Health inequities are unfair differences in health that could be avoided with reasonable action. d. Health inequities are wide variations in health status and services among certain population groups.

ANS: C Health inequities refers to differences in health that could be avoided if reasonable action was taken, and therefore these differences are considered to be unfair and socially unjust.

A nursing student develops a teaching plan about hand washing to present to a group of elementary school children at the local school. Which public health intervention is being implemented? a. Collaboration b. Surveillance c. Health teaching d. Screening

ANS: C Health teaching communicates facts, ideas, and skills that change knowledge, attitudes, values, beliefs, and practices of individuals, families, systems, and/or communities.

One advantage of health-risk appraisal instruments is that they: a. Are suitable for all age-groups b. Accurately reflect an individuals ability to initiate changes in lifestyle c. Provide support to nurses in counseling individuals about self-care behaviors d. Emphasize environmental factors

ANS: C Health-risk appraisal instruments provide support to nurses in counseling and educating individuals in self-care behaviors.

12. A home health nurse asks a client with arthritis to attend a demonstration in which an assistive device is used to put on shoes. Which phase of the nursing process is the nurse's current focus? a. Outcome identification b. Planning c. Implementation d. Evaluation

ANS: C Implementation is the phase in which the home health nurse implements the interventions identified in the plan of care. If the client is able to use the device, the process can progress to evaluation, in which the nurse will assess the usefulness of the device when incorporated into the client's activities of daily living.

2. Which of the following statements best explains why many school nurses are not able to ensure that all children receive needed health care in the schools? a. There is a shortage of baccalaureate-prepared nurses with national school health nurse certification. b. Most nurses prefer to be employed in hospitals giving direct care. c. Most school districts are unable to afford a nurse in every school. d. School districts and taxpayers see no need for nurses in schools.

ANS: C In Healthy People 2020, objective ECBP-5 states that there should be one nurse for every 750 children in each school (U.S. Department of Health and Human Services, 2010). Most schools have not achieved this objective. In 2006, approximately 40% of the nation's schools met that standard. The new objective is that 44.7% of the country's elementary, middle, junior high, and senior high schools have this many nurses by 2020 (U.S. Department of Health and Human Services, 2010). Having fewer nurses in the schools means that the nurses are expected to perform many different functions. It is therefore possible that they are unable to provide the amount of comprehensive care that the students need (Croghan, 2009).

What CHN action can help prevent depression in older adults who are at high risk for it? a. Encourage them to move to a nursing home where they will have the company of others in the same age group. b. Monitor for signs and symptoms of depression. c. Organize a health promotion program for older adults at the local centre. d. Encourage older adult clients to focus on their strengths rather than their weaknesses.

ANS: C It is important for CHNs to recognize that older adults who are depressed usually have a clinically different presentation from that of clients in other age groups. Older adults who are depressed tend to present with many bodily complaints, such as chronic pain, nausea and vomiting, and insomnia, and usually do not express feelings of sadness, guilt, or worthlessness. The depression rate among older adults is half that among younger people, but the presence of a physical or chronic illness increases rates of depression. Activities to improve the mental health status of older adults include public education programs, prevention approaches, and the provision of mental health services in primary care. Depression rates for older adults in nursing homes range from 15% to 25%, and thus this would not be a good place to recommend for social networking. Encouraging older clients to focus on their strengths rather than their weaknesses can sound insensitive if (1) the weaknesses are, indeed, profound, and (2) the necessary tools for coping are not provided along with the advice. Monitoring for signs and symptoms of depression only monitors health issues, rather than preventing them.

Which question would a nurse ask during the first phase of a risk assessment? a. Has the chemical been released into the environment? b. How much and by which route of entry can the chemical enter the body? c. Is the chemical known to be associated with a negative health effect? d. What is the prediction for potential harm?

ANS: C The first phase is determining if a chemical is known to be associated with negative health effects (in animals or humans).

10. Which of the following best describes why it is more challenging for most nurses to meet the needs of a dying child and his or her family than to meet the needs of a dying adult? a. Children don't understand what it means to die. b. A child's death is harder for anyone to accept. c. Society does not expect death to occur in children. d. Families are not prepared to deal with death.

ANS: C It is true that families are rarely prepared to deal with death. It is also true that children have only a limited understanding of dying. However, the needs of the dying child and family are unique because society does not expect death to occur to the young or to have the child die before the parent.

Which intervention can be implemented by a nurse when working with a group focused on improving the health of the community? a. Make decisions for the group to move the process along. b. Invite wealthy individuals to participate in the group process. c. Maintain members through recognition and encouragement. d. Teach topics that are of national importance.

ANS: C Maintaining members is an important intervention to facilitate group process.

19. A school nurse has developed a special class for pregnant teens to teach them everything from anticipated body changes to methods for managing common pregnancy-associated problems. The classes also allow the nurse to be in close frequent contact with the students to monitor their health status. Which of the following levels of prevention is being used by the nurse? a. Primary prevention b. Secondary prevention c. Tertiary prevention d. Both primary and secondary prevention

ANS: C Many teenage girls who are pregnant attend school; therefore, the school nurse may provide ongoing care to the mother. Although this may appear to be secondary prevention, it is tertiary prevention because adolescent pregnancies are considered to be high risk.

Which nursing action demonstrates advocacy? a. Offering a smoking cessation program b. Screening for hypertension c. Lobbying for health care reform d. Conducting home visits

ANS: C Nurses should participate in implementing new directions for health care and help envision these new directions. Nurses can be an important voice in advocating for access to consistent, effective, efficient health care for all.

When using the integrative model for community health promotion, the nurse would apply the dimension of client system by: a. Concentrating on individual health behaviors b. Studying client-centered initiatives c. Providing multidimensional nursing care among various levels of clients d. Focusing on health promotion, illness prevention, and illness care

ANS: C Nursing care is targeted toward several levels of clients in the client system.

Which nursing diagnosis would a public health nurse use when addressing the problem of obesity at the community-level of practice? a. Alteration in nutrition: More than body requirements b. Need for increased knowledge of proper nutrition c. Families at risk for obesity because of inactivity d. Overweight child related to poor dietary habits

ANS: C Nursing diagnoses must be modified to meet the needs of population-focused care in public health nursing practice. Families at risk for obesity because of inactivity is the only nursing diagnosis that addresses a community group that is in need of further intervention.

7. Which of the following types of home health agencies emphasizes health promotion and illness prevention? a. Combination agencies b. Hospital-based agencies c. Official agencies d. Proprietary agencies

ANS: C Official or public agencies include those agencies operated by the state, county, city, or other local government units, such as health departments. Nurses employed in these settings provide well-child clinics, immunizations, health education programs, and home visits for preventive health care. Consequently, they include a focus on health promotion and illness prevention as well as giving direct care.

There are two medically indigent clients in the clinic who have come to get their monthly supply of free insulin. There is only enough for one client. Which action does the nurse take first? a. Identify all options. b. Make a decision. c. Gather additional information. d. Act and assess decisions made.

ANS: C One must have all information before looking at options and making a decision.

A nurse is addressing the problem of air pollution in the community. The first step in the process of controlling the pollution would be: a. Setting standards b. Monitoring c. Permitting d. Compliance

ANS: C Permitting is a process by which the government places limits on the amount of pollution emitted into the air or water.

Which action by a case manager would be classified as primary prevention? a. Advocating for the client whose values conflict with those of the medical service provider b. Collaborating between nursing and occupational health personnel c. Educating a group regarding community services that are available if they are ever needed d. Resolving conflict between a primary care clinic and a tertiary care facility

ANS: C Primary prevention involves the use of the information exchange process to increase the client's understanding of the health care system. Remember that primary prevention occurs at a point before an illness or a problem occurs. In all of the other options, the client's health concern already exists and interventions have been employed.

When a nurse is evaluating the components of an educational program, the nurse is completing a(n): a. Education evaluation b. Educational product c. Process evaluation d. Ongoing evaluation

ANS: C Process evaluation means looking at the components of an educational program.

A public health nurse utilizes the nursing process at all levels of practice by: a. Including specific goals for community health nurses b. Developing an accurate nursing diagnosis c. Analyzing the needs of the community, system, and individuals and families d. Utilizing primary, secondary, and tertiary prevention

ANS: C Public health nurses must customize the nursing process to consider the community, systems, and individual/family levels of practice.

A group member who has taken on the role of the gatekeeper will: a. Seek and accept the authority or direction of others. b. Guide and direct the group activity. c. Control outsiders access to the group. d. Focus the movement toward the main work of the group.

ANS: C The gatekeeper controls outsiders access to the group.

According to Laffrey, Loveland-Cherry, and Winkler, the health perspective views: a. Health as the absence of disease b. Human lifestyle based on ones prescribed regime c. Humans as complex and interconnected with the environment d. Community education as the role of the registered nurse

ANS: C The health paradigm includes the view that humans are complex and interconnected with the environment.

1. According to the American Academy of Pediatrics, which of the following activities are expectations for a school health nurse? a. Ensuring that children with health problems are accepted by their peers b. Driving children home if parents can't pick them up c. Giving emergency care in the school or during school events d. Giving medications as needed if children are ill

ANS: C School nursing responsibilities include making sure that children get the health care they need, including emergency care in the school, keeping track of the state-required vaccinations that children have received, carrying out the required screening of the children based on state law, and ensuring that children with health problems are able to learn in the classroom. The nurse cannot convince children to accept other children as peers, although certainly efforts should be made. HIPAA would not allow individual examples of health problems to be shared, other than providing group statistics.

Since the Intervention Wheel was first published in 1998, it has: a. Guided national policy b. Been used as a tool in deciding licensure issues for State Boards of Nursing c. Been incorporated into the public health curricula of many nursing programs d. Gained wide acceptance internationally

ANS: C Since being published, the Intervention Wheel has been incorporated into the public/community health coursework of numerous undergraduate and graduate curricula.

. When implementing interventions at the systems-level of practice, the public health nurse would: a. Involve the entire community in solving the health problem. b. Identify health problems in the community. c. Change laws, policies, and practices that influence population-based issues. d. Provide outreach services to populations at risk.

ANS: C Systems-level practice consists of changing laws, policies, and practices that influence population-based issues.

A client who abuses intravenous (IV) drugs admits to the CHN that he has no desire to change this behaviour, so the CHN counsels him on the importance of sterilizing his needles to prevent infection and transmission of blood-borne diseases. Which level of prevention does this action represent? a. Primary prevention b. Secondary prevention c. Tertiary prevention d. Primary and tertiary prevention

ANS: C Tertiary prevention is implemented in people who already have a health concern or problem (in this instance, someone who is already misusing substances) to prevent complications (harm reduction). In this instance, because the client does not plan to stop the drug abuse, the CHN is trying to prevent complications arising from use of contaminated needles. Emphasis is placed on reducing the transmission of blood-borne diseases through contaminated needles. Giving up the addiction is the best solution but is unrealistic for many addicts. Using the harm reduction model, the CHN should provide education on cleaning needles with bleach between uses and about needle exchange programs to decrease the spread of blood-borne pathogens.

Which one of the following is an example of tertiary prevention by a CHN? a. Administering the Mantoux (skin) test to identify persons with TB b. Assessing for signs and symptoms of active TB c. Directly observing clients with active TB as they take their antituberculosis medications d. Interpreting TB skin test results

ANS: C Tertiary prevention is implemented when CHNs provide directly observed therapy (DOT) to those with active TB.

11. Which of the following would be the best way for the school nurse to fulfill his or her responsibilities in an emergency situation? a. Tell all staff to call 911 if the nurse is not in the building. b. Arrange to always be available, even if only by phone. c. Create and share an emergency plan with all teachers and staff. d. Wear a pager so that the nurse can come as soon as humanly possible.

ANS: C The American Health Association recommends that the school nurse create an emergency plan with at least two different staff members identified and responsible for implementing the plan if the nurse is not in the building at the time of the emergency. The plan would include when to call 911 and how to get a child to the hospital via ambulance if needed.

Which environmental law sets basic structure for regulating pollutants to United States waters? a. Safe Drinking Water Act b. Toxic Substance Control Act c. Clean Water Act d. Pollution Prevention Act

ANS: C The Clean Water Act sets basic structure for regulating pollutants to United States waters.

The Framingham Heart Study was successful in: a. Providing information on the effectiveness of risk-reduction interventions b. Documenting the relationship between social variables and heart disease c. Identifying factors contributing to the development of coronary heart disease d. Demonstrating the effects of mass media in modifying high-risk behavior

ANS: C The Framingham Heart Study identified factors contributing to the development of coronary heart disease and high blood pressure.

The cognitive domain includes: a. Changes in attitudes and the development of values b. The performance of skills c. Memory, recognition, understanding, reasoning, and problem solving d. Memorization of one set of skills before moving on to the next

ANS: C The cognitive domain includes memory, recognition, understanding, reasoning, and problem solving.

4. Which of the following events led to the emphasis on home care nurses caring for acutely ill clients and the increased demands for extensive documentation? a. Advances in medical technology and pharmacology b. Increased number of lawsuits for substandard care c. Introduction of Medicare d. Social Security Act of the 1930s

ANS: C The combination of preventive services and illness care followed the introduction of Medicare in 1966. The Medicare program emphasized care for more acutely ill people rather than illness prevention and health promotion. The 1997 federal Balanced Budget Act, which implemented the prospective payment system in home care, increased pressure to care for clients with acute illnesses that were likely to improve.

When implementing a community health education program, which educational principle will the nurse use as a guide? a. Refer to trustworthy sources b. Use an active voice c. Create the best learning environment d. Accentuate the positive

ANS: C The environment must be conducive to learning for educational programs to be effective.

Which example contains the components necessary to form an epidemiologic triangle? a. Pesticides, water, food b. Lead, mercury, soil c. Trichloroethylene, water, infants d. Children under 12, elderly, temperature

ANS: C The epidemiologic triangle consists of an agent (chemical), host (community consisting of several variants), and environment (air, water, soil,

What information is most important to keep in mind when caring for a pregnant teen? a. All teen pregnancies are considered high risk. b. Limited knowledge can lead to pregnancy complications. c. Pregnant teens are less likely to focus on proper prenatal nutrition. d. Pregnant teens who are poor are more likely to have poorer health outcomes.

ANS: C The nutritional needs of a pregnant teenager are especially important. The CHN needs to assess the pregnant teenager's current eating pattern and provide creative guidance to address the issue of the demands of pregnancy on a normally changing teenager's body and a teenager's usual nutritional habits of fast foods and snacking.

What are the critical attributes in the definition of community? a. Families, groups, and health organizations b. Health needs, geographical boundaries, and target population c. People, place, and functions d. Populations and health resources

ANS: C The people are community members or residents

9. Which of the following is the primary focus of hospice care? a. Curing or controlling the client's chronic disease b. Decreasing the waste of acute care (hospital) resources c. Providing palliative care to maintain comfort until death d. Teaching the client and family how to care for themselves

ANS: C The primary goal of hospice care is to help maintain the client's dignity and comfort. Alleviating pain, encouraging the client, family, and friends to communicate with each other about essential sensitive issues, and coordinating care to ensure a comfortable, peaceful death all contribute to palliative care.

22. The principal of a school was upset over a rumor that one of the children had engaged in a violent activity that injured a younger sibling. The principal asked the nurse who the children were so that the involved teachers could both support the injured child and guard other children from the violent child. Which of the following would be the best response by the nurse? a. "I'll get the names to you and the involved teachers immediately." b. "Let me get the parents' consent, and then I'll get you the names." c. "Rumors are often inaccurate let me follow up and see what happened and what needs to be done." d. "Why don't we coordinate a school-wide program on preventing accidents instead?"

ANS: C The school nurse is responsible for maintaining school health office policies, including privacy and safety of health records. The nurse must follow the HIPAA privacy rules while also ensuring the safety of children at school. When a rumor is involved, it is always wise to check its accuracy. If a child was indeed hurt, the nurse needs to make sure both children involved receive (or have already received) appropriate care. The question about having a school-wide program may be appropriate, but this behavior was apparently purposeful, not accidental.

A client reports that the narcotic she took for pain on a regular basis made her feel bad and that when she tried an alternative analgesic, she experienced withdrawal symptoms. What is this client suffering from? a. Drug abuse b. Drug addiction c. Drug dependence d. Substance abuse

ANS: C The terms drug dependence and drug addiction often are used interchangeably, but they are not synonymous. Drug dependence is a state of neuroadaptation caused by the chronic, regular use of a drug. People who are dependent on drugs must continue using them to prevent withdrawal symptoms. Drug addiction, in contrast, is a pattern of abuse characterized by an overwhelming preoccupation with the use (compulsive use) of a drug and securing its supply, and a high tendency to relapse if the drug is removed.

A woman has attended a weight reduction program for a year and lost 75 pounds. She now attends weekly meetings to keep the weight off. Which term best describes this example? a. Health promotion b. Illness prevention c. Health maintenance d. Health protection

ANS: C The womans weight is being maintained through attending the weight reduction program. Health maintenance focuses on keeping a current state of health.

Employees working with hazardous chemicals have the right to know about the chemicals they are working with through the creation of the: a. Material Safety Data Sheet b. Consumer Confidence Report c. Hazard Communication Standard d. Environmental Protection Agency

ANS: C This standard requires employers to maintain a list of all hazardous chemicals that are used on site.

11. When meeting with a client and family, the home health nurse says, "We have discussed your health problems and limitations. Now tell me what level of health and function you hope to achieve." In which of the following phases of the nursing process is the nurse engaging? a. Assessment b. Diagnosis c. Outcome identification d. Planning

ANS: C When discussing health problems and limitations, the nurse was gathering a history as part of the assessment phase. This allowed formulation of a nursing diagnosis. By steering the conversation toward goals, the nurse is now able to work with the client to mutually identify outcomes.

1. Which of the following aspects of a home health agency would most likely be examined during the accreditation process? (Select all that apply.) a. Cost of each service rendered b. Credentials of each employee c. Organizational structure d. Outcomes of care

ANS: C, D Both The Joint Commission (TJC) and the Community Health Accreditation Program (CHAP) of the National League for Nursing (NLN) look at the organizational structure through which care is delivered, the process of care through home visits, and the outcomes of client care, focusing on improved health status. Performance improvement must be ongoing in the agency. It is assumed the credentials of each employee were confirmed before employment.

2. A family is concerned about the medical bills of their father, age 63, who is unemployed and has almost no savings. Which of the following statements by the nurse accurately explain how Medicaid and Medicare would work in this family's situation? (Select all that apply.) a. "All your father's medical bills will be paid by whichever program is appropriate." b. "Choose any physician and just show them your Medicare or Medicaid card." c. "Your father must be homebound to qualify for Medicare assistance but less so for Medicaid." d. "Medicare is a federally funded program, but Medicaid is administered by your state."

ANS: C, D Many physicians will not accept a client on Medicare or Medicaid because of the low reimbursement rates. A deduction from Social Security is made for Medicare premiums, and clients are still responsible for deductibles and copays, so it is misleading to tell the family that all their father's medical bills will be paid. Medicare, for those age 65 and over or disabled, is a federal insurance program administered by the Social Security Administration, whereas Medicaid, based on a client's lack of financial resources, is a federal and state assistance program administered by the state. Medicare will only pay for home health care by skilled professionals while the client is homebound, whereas Medicaid does not necessarily require homebound status and may reimburse for home health aides and other nonskilled supportive services.

3. A public health nurse leader is encountering barriers when trying to shift the public health agency's efforts to a population-focused practice. The reasons peers are not supportive of the proposed shift to a population focus are most likely related to: a. agency colleagues' push for nurses to focus on population initiatives. b. costs associated with staff training and revision of documents. c. lack of support from the agency's funding sources. d. opinions that nursing should focus on the provision of direct client care and services.

ANS: D Barriers to implementing population-focused care include lack of understanding of the public health nurse role and its relationship to other roles in nursing, such as direct care and services

1. A home health case manager is charged with identifying opportunities for health promotion and illness prevention. The fulfillment of this charge would best be demonstrated when the case manager: a. collaborates with a local chaplain to ensure that the spiritual needs of cancer clients are addressed. b. refers a new diabetic client to a nutrition counselor for dietary teaching. c. teaches a school nurse how to care for a client who will be returning to school and will require new asthma treatments. d. tracks the immunization status of clients and facilitates access to immunization when needed.

ANS: D Case management involves assessment—in this case, screening—and arrangement for delivery of services. Primary prevention includes case management to identify at-risk clients and arrange for services to prevent disease. Through nurse management activities, general community deficiencies in the quality or quantity of health services are often discovered. Case management activities with individual clients and families will reveal the broader picture of health services in and health status of the community. Community assessment, policy development, and assurance activities that frame the core functions of public health actions are often the logical next step in the nurse case manager's practice to intervene at the community level to make changes. Therefore, the core components of case management and the nursing process are complementary. The nursing process function of assessment is complementary to the case management process function of case finding and includes such activities as applying screening tools according to program goals and objectives.

1. In 1988, the Institute of Medicine (IOM) published a report on the future of public health and its mission that defined public health as: a. what public-private partnerships do to treat vulnerable populations. b. what the government does to ensure that vital programs are in place. c. what the U.S. Public Health Service does to prevent disease, promote health, and deliver services. d. what society does collectively to ensure the conditions in which people can be healthy.

ANS: D In 1988, the IOM's report stated that public health is "what we, as a society, do collectively to assure the conditions in which people can be healthy." Consequently, the mission of public health is "to generate organized community effort to address the public's interest in health by applying scientific and technical knowledge to prevent disease and promote health." This clearly places the emphasis on the desire of the population and community to ensure access to services that foster the health status of the overall community through the equitable distribution of resources addressed to community problems that affect health.

8. Members of an extended family are in conflict over the treatment plan for the family's 90-year-old matriarch, who has developed indications of advanced dementia. The case manager is using conflict resolution strategies to allow the parties involved to develop trust, credibility, and distance from the issue at hand, as well as to retain personal dignity. The nurse is demonstrating a knowledge of the strategic process of: a. advocating. b. clarifying. c. cooperating. d. negotiating.

ANS: D Techniques for conflict management involve a range of active communication skills. These skills are directed toward learning all parties' needs and desires, detecting their areas of agreement and disagreement, determining their abilities to collaborate, and assisting in discovering alternatives and activities for reaching the goal of mutual benefit with limited loss. Negotiating is a strategic process used to move conflicting parties toward an outcome and has several stages that allow the parties to develop trust, credibility, and distance from the issue, while retaining personal dignity. All of these factors are important when attempting to move parties toward the creation of new solutions or options through relearning, brainstorming, reflective thinking, and problem-purpose-expansion techniques (systematic problem solving approaches).

4. A community health nurse is the case manager for a homebound client recovering from a hip replacement. The nurse works with the client and his family to prioritize needs and services, and to address these care needs. These activities represent which step in the nursing process? a. Assessment and planning/outcome b. Diagnosis and planning c. Implementation d. Planning/outcome

ANS: D The case management process function of problem prioritizing and planning to address care needs is complementary to the planning/outcome step of the nursing process.

A mother confides to the CHN that her live-in boyfriend pushed her 2-year-old child because he was crying too much. She begs the CHN not to tell anyone because her boyfriend has agreed to take anger management classes. What should the CHN do? a. Abide by the mother's wishes because this information was provided in confidence. b. Arrange for the earliest available counselling for the boyfriend. c. Advise the mother to take the child away from the boyfriend and find alternative housing right away. d. Report the incident to the child protection agency.

ANS: D All individuals in Canada who suspect child abuse are required to report it to the proper child protection agencies as mandated by law. First Nations peoples also have access to specific child protection agencies. All of the other options put the child's welfare at risk.

A CHN suspects that an elementary school student is being physically abused. Which action would be the most appropriate one for the CHN to take? a. Ask the student about the abuse. b. Document findings in the student's school record. c. Discuss the suspicions of abuse with the student's teachers or the family's spiritual leader. d. Notify legal authorities.

ANS: D All individuals in Canada who suspect child abuse are required to report it to the proper child protection agencies, as mandated by law. First Nations peoples have their own child protection agencies.

14. Which of the following do community-based nurses typically use to organize, sort, and document pertinent client data? a. NANDA b. NIC NOC c. Nursing Diagnosis Taxonomy d. The Omaha System

ANS: D Although all four are recognized nursing taxonomies, in community health, especially home nursing, the Omaha System is typically used. This system was developed by a visiting nurses association in Omaha and based on home nursing documentation needs. The Omaha System is most relevant to home health nursing and is most typically used by home health nurses.

3. A hospital nurse wants to know why home health nurses often take more time with assessment than nurses do in the hospital. Which of the following is the best explanation for this? a. Home care assessment includes not only the client but the supplies and equipment the family may have available for use. b. The home environment is less organized and equipped for the nurse's use. c. Family members must also be assessed for possible problems that could interfere with the primary client's recovery. d. To be effective, the home health nurse must earn the family's trust and work in partnership with them.

ANS: D Although all the responses contain some truth, the primary reason that assessment may take longer for the home health nurse is because when working in a client's home, the nurse is a guest. To be effective, the nurse must earn the trust of the family and establish a partnership with client and family.

A collection of individuals who have in common one or more personal or environmental characteristics is the definition of a(n): a. Community b. Group c. Family d. Aggregate

ANS: D An aggregate is a collection of individuals who have one or more personal or environmental characteristics in common.

An acceptable level of emissions or a maximum contaminant level allowed is an example of which environmental protection strategy? a. Controlling pollution b. Waste minimization c. Land use planning d. Environmental standard

ANS: D An example of an environmental standard is an acceptable level of emissions or a maximum contaminant level allowed.

A nurse prepares for a presentation to a group of adults using strategies appropriate for adult learning. This concept is called: a. Authoritarianism b. Learning style c. Pedagogy d. Andragogy

ANS: D Andragogy is a term and model developed by Malcolm Knowles to describe learning strategies for adult learners.

The educational process of selecting appropriate educational methods is similar to which of the following steps of the nursing process? a. Assessment b. Evaluation c. Implementation d. Planning

ANS: D Assessment would be identifying educational needs, evaluation would be evaluating educational process, and implementation would be implementing the educational plan. Planning is similar to selecting appropriate educational methods.

23. Which of the following clients over 65 years of age meets the criteria for Medicare reimbursement for home health nursing? a. The client who needs assistance with bathing and meal preparation b. The client who needs assistance with house cleaning and meal preparation c. The client who needs sitter services because she wanders from home and becomes lost d. The client whose family members need to learn how to care for his wound

ANS: D Because the nursing service must be considered "skilled," custodial services alone (e.g., sitter services and assistance with ADLs) are not sufficient cause for Medicare reimbursement.

A nurse is teaching a client about how to complete a dressing change of a wound. The nurse knows that what conditions must be met before learning will occur? a. Must be able to memorize the instructions, relay this information to a partner, and demonstrate the dressing change b. Must master the dressing change at the time it is taught, repeat the demonstration for the nurse, and teach another person c. Must be able to speak the language of the nurse, have time to practice the dressing change, and master the dressing change in a short time d. Must have the necessary ability, a sensory image of how to carry out the dressing change, and an opportunity to practice the dressing change

ANS: D Before psychomotor learning occurs, the learner must have the necessary ability, a sensory image of how to carry out the skill, and an opportunity to practice the skill.

When nurses apply the knowledge and processes of ethics to the examination of ethical problems in health care, they are using: a. Values b. Morality c. Ethics d. Bioethics

ANS: D Bioethics applies the knowledge and processes of ethics to the examination of ethical problems in health care.

What intervention would be the most appropriate for the nurse to use when trying to develop community partnerships? a. Involve only the community residents. b. Use nurses as the source of information and leadership. c. Rely on the power of local officials. d. Include a variety of disciplines.

ANS: D Community partnerships occur when community residents and health workers come from a variety of disciplines.

Which is an example of a public health nurse conducting a community assessment? a. Visiting an elderly person at home to assess and evaluate safety and fall risk b. Developing diagnoses to identify nursing interventions at a health clinic c. Evaluating services at an immunization clinic where a translator provides services d. Compiling recent data from the county health department on child abuse cases

ANS: D Compiling recent data from the county health department on child abuse cases is an example of community assessment, assessing needs and data within a community.

A nurse utilizing a disease-oriented approach would: a. Promote a greater level of positive health. b. Teach about common disease processes. c. Provide acute care management of chronic disease. d. Direct care toward disease prevention.

ANS: D Disease prevention, risk appraisal, and disease management are all elements of the disease-oriented approach.

Which statement best describes health risk appraisals? a. Data about health practices are collected from families. b. Primary prevention strategies are implemented by using the collected data. c. Identified risks can be easily modified. d. Individual health practices are compared with data from epidemiologic studies.

ANS: D During a health risk appraisal, individuals supply information about their health practices, demographic characteristics, and personal and family medical history for comparison with data from epidemiologic studies. These comparisons are used to predict individuals risk of morbidity and mortality and to suggest areas in which disease risks may be reduced. Health risks appraisals use secondary prevention by screening to prevent or detect disease in its earliest stages.

A potential barrier that a nurse may experience when in the educator role is: a. Working with clients with a low literacy level b. Memorizing the information that is to be taught c. Having a limited vocabulary d. Lacking knowledge about how to gain participation

ANS: D Educators may lack knowledge about how to gain participation. Participation can be fostered by asking open-ended questions, inviting participation, and planning small-group activities whereby a person responds based on the group rather than presenting his own information.

John Snow is called the "father of epidemiology" because of his work with: a. Cholera b. Malaria c. Polio d. Germ theory

a. Cholera

What action is the best example of enabling in a family with an alcoholic father? a. The father asks the CHN to explain why his continued drinking is dangerous. b. The son threatens to leave the home because he finds his father's behaviour embarrassing. c. The teenage daughter turns to a favourite teacher for support. d. The wife tells her husband's boss that her husband is sick when he is actually inebriated.

ANS: D Enabling is the act of shielding or preventing the addict from experiencing the consequences of his or her addiction.

An inspection of a facility after a permit is obtained for the purpose of observing whether the plans submitted in the permit application are being implemented as approved is an example of which environmental protection strategy? a. Controlling pollution b. Waste minimization c. Land use planning d. Environmental monitoring

ANS: D Environmental monitoring would be an inspection of a facility after a permit is obtained to observe whether the plans submitted in the permit application are being implemented as approved.

Which statement about feminist ethics is correct? a. Feminists include only women in their worldview. b. Persons who ascribe to feminist ethics are passive and wish to pursue their ideals through the legislative process. c. Feminists believe that men should not be nurses. d. Womens thinking and moral experiences are important and should be taken into account.

ANS: D Feminist theory ascribes to the idea that womens thinking and moral experiences are important and should be considered.

It is important for a nurse to recognize group norms because they: a. Allow for creativity and variety among group members b. Influence members perception of community c. Are helpful in evaluating the effectiveness of the group d. Maintain the group through various supports to members

ANS: D Group norms serve to maintain the group through various supports to members.

Public health nurses use a common set of interventions to: a. Describe the proper order of implementation. b. Emphasize surveillance as the main focus of public health practice. c. Guide practice and generate agency protocols. d. Improve the health status of communities, systems, individuals, and families.

ANS: D Interventions are actions taken on behalf of communities, systems, individuals, and families to improve or protect health status.

What is the cause of the greatest overall costs to the community for providing health care to people who are homeless? a. The need for increased preventive services to address the health conditions of the homeless population b. The need for more frequent clinic visits by homeless clients for multiple health problems c. The spread of contagious diseases by people who are homeless to those they pass on the street d. The fact that most of the care for people who are homeless takes place in hospital emergency departments

ANS: D People who are homeless encounter the same problems accessing health care (e.g., lack of money, lack of housing, lack of transportation) that others do in impoverished conditions. Therefore, health care of people who are homeless tends to be crisis oriented and sought in emergency departments. Low-income Canadians have the highest mortality rates and the highest rates of hospitalizations and emergency visits.

Which statement is discussed in the Code of Ethics for Nurses with Interpretive Statements? a. The profession of nursing is responsible for making political statements and supporting nurse-friendly candidates for office. b. The nurses primary focus is on acute bedside nursing, followed by community health care to promote seamless care. c. The nurse owes duty primarily to the physician to strive to protect health, safety, and the rights of the patient. d. The profession of nursing is responsible for articulating nursing values, for maintaining the integrity of the profession, and for shaping social policy.

ANS: D Provision 9 of the Code of Ethics for Nurses with Interpretive Statements discusses the need for the nursing profession to address national and global health concerns as well as be involved with shaping policies through political action.

Which statement is true about the origins of the Intervention Wheel? a. A panel of nurses from Iowa, Minnesota, North Dakota, South Dakota, and Wisconsin developed and refined the Intervention Wheel. b. It was conceived by a group of international nurses from Norway, Kazakhstan, and Japan. c. It was a result of a qualitative analysis carried out by the State Boards of Nursing. d. It resulted from a grounded theory process carried out by public health consultants at the Minnesota Department of Health.

ANS: D Public health consultants with the Minnesota Department of Health carried out a grounded theory process in response to uncertainty about the contributions of public health nursing to population health-level improvement, resulting in the identification of the Intervention Wheel components.

5. Which of the following best explains why school nurses are involved in helping teachers with the task of teaching children how to practice problem solving, communication, and other life skills? a. Teacher shortages have required nurses to be increasingly involved in teaching life skills. b. Because so many nurses want to be employed in schools, this responsibility was assumed to increase employment opportunities. c. States are requiring nurses to screen and to teach life skills. d. Nurses have been enlisted in this role to help reduce risk factors for future health problems in school children.

ANS: D School health nurses were originally involved in this capacity in 1987 after the CDC began funding schools for HIV-prevention education programs. This program was so successful that it was expanded to include programs to teach children prevention of other chronic illnesses caused in part by risk factors such as poor diet, lack of exercise, and smoking. The schools are actively involved in helping the children practice problem solving, communication, and other life skills so that they can reduce their risk factors for health problems.

What designation is given to groups at high risk of having poor health outcomes? a. Cumulative risk groups b. Health disparity groups c. Resilient populations d. Vulnerable populations

ANS: D Specific populations who are more vulnerable—that is, at-risk populations who are more susceptible to poor health because of socioenvironmental factors—are often referred to as vulnerable populations.

When applying the integrative model for community health promotion, a nurse assesses a neighborhood for accessibility and adequacy of care to provide treatment for medical conditions. What client system does this describe? a. Individual b. Family c. Aggregate d. Community

ANS: D The community focus includes the nurse assessing the community for accessibility and adequacy of care to provide treatment for medical conditions.

A nurse plans on implementing a community-wide influenza immunization program. Which factor should the nurse consider when implementing this program? a. The number of community members who have already received the immunization b. The existence of formal groups in the community c. Public policy that mandates influenza immunization for certain populations d. The communitys readiness to participate in the program

ANS: D The factors that influence implementation in the community are the nurses chosen roles, the type of health problem selected as the focus for intervention, the communitys readiness to take part in problem solving, and characteristics of the social change process.

25. A family member asks a home health nurse to explain the concept of hospice care. Which of the following would the nurse need to include as the fundamental underlying philosophy of hospice? a. Enabling the client to die at home b. Ensuring that the client's living will is upheld c. Placing experts in the position of power of attorney d. Providing comfort measures before death

ANS: D The hospice philosophy of care means providing comfort measures to an individual before death. Death may occur in the individual's home, in a hospital setting, or in an uncontrolled setting such as the community.

21. Which of the following best describes one of the outcomes of the incentives and pressures for cost control and improved health outcomes? a. Expansion in alternative health care agencies b. Improvements in client teaching materials c. Public pressure to improve health professionals' education d. Development and increased use of telehealth technology

ANS: D The incentives and pressures for cost control and improved health outcomes have increased the development and use of telehealth technology in home care. Simultaneously, technologies have been simplified and their reliability increased, facilitating their safe use in the home.

Public health nurses utilize registries to identify children with delayed or missing immunizations. They subsequently follow up with families by phone calls or home visits. This is an example of ____-level of practice. a. Systems b. Community c. Policy d. Individual/family

ANS: D The individual/family-level of practice focuses on interventions that involve working with individuals, either singly, or in groups, and with families.

A nurse is trying to increase participation in a free colorectal screening program for middle-aged adults who lack health insurance. Which implementation mechanism would be most effective? a. Small interacting groups b. Health policy c. Lay advisors d. Mass media

ANS: D The mass media (newspapers, television, and radio) represent an impersonal and formal type of communication and are useful in providing information quickly to a large number of people.

Which of the following domains would be used to teach a new diabetic how to give an insulin injection? a. Developmental b. Cognitive c. Affective d. Psychomotor

ANS: D The psychomotor domain includes the performance of tasks that require some degree of neuromuscular coordination and emphasizes motor skills.

Which characteristic is an indicator of community health structure? a. Infant mortality rate b. Effective communication c. Crime rate d. Emergency room utilization

ANS: D The structure of the community is defined in terms of services and resources.

6. A new client has just been released from the hospital after intensive treatment for multiple injuries following a motorcycle accident. Which of the following types of care will he most likely receive? a. Home-based primary care b. Population-focused home care c. Proprietary home care d. Transitional care

ANS: D Transitional care programs in the home are designed for populations who have complex or high-risk health problems and are making a transition from one level of care to another. Assessment, planning, teaching, making referrals, and following up on referrals foster independence and self-care. Besides intensive teaching about self-care, telephone calls help ensure that the client understands and is able to implement instruction.

14. A school nurse listens as one student talks about another student being upset because his father frequently spanks him with a leather belt that leaves big marks on the student's back. But the student begs the nurse not to tell anyone because he promised the friend that the information would never be shared. Which of the following actions should be taken by the nurse? a. Ask the student if abuse has occurred. b. Call in the named student and ask him to remove his shirt. c. Discuss the conversation with the student's parents. d. Notify the legal authorities.

ANS: D When the nurse identifies a child who may be abused or who receives information from someone else that a child may have been abused, the nurse must contact the appropriate legal authorities and the school's principal. Asking the student about the abuse will not always elicit a truthful answer, because children will protect their parents. A confidential file should be made about the incident; however, the nurse should let the government authorities, usually the state or county child protection department, look into the suspected case. In all cases, the child should be protected from harm, and those who have no right to know that child abuse or neglect is suspected should not be given any information.

1. A nurse performing home hospice case management notes the increasing number of hospice clients who lack caregivers in the home environment. The nurse identifies the potential need for a hospice house facility to meet the needs of these clients. The case management process frequently reveals larger picture issues such as which of the following?(Select all that apply.) a. Community cost concerns b. Community conflict resolution skills c. Community satisfaction d. Community weaknesses in quality of services e. Community weaknesses in quantity of services

ANS: D, E Case management activities with individual clients and families very often reveal the larger picture of health services and health status of a community. General community weaknesses in the quality and quantity of services often are discovered. The nurse can then intervene at the community level to initiate changes. In this case, the nurse identified a deficiency in the quality of hospice services to individuals who did not have a caregiver in the home environment and the parallel concern of the lack of hospice-related facilities to meet the needs of these clients to achieve a quality dying process.

DELETE

ANS: deinstitutionalization Page: 723 Feedback The deinstitutionalization movement closed state mental hospitals and caused the discharge of individuals with mental illness. Congress passed the Mental Retardation Facilities and Community Mental Health Centers Construction Act (often called the Community Mental Health Centers Act) in 1963. This act called for the construction of comprehensive community health centers, the cost of which would be shared by federal and state governments. Unfortunately, many state governments did not have the capability to match the federal funds required for the establishment of these mental health centers.

In the late 1800s, local health departments were formed in urban areas for what purpose? 1.To target environmental hazards associated with crowded living conditions and dirty streets and to regulate public baths, slaughterhouses, and pigsties 2.To facilitate interdisciplinary efforts and promote the "practical application of nursing" 3.To provide immunizations to all citizens 4.To provide public health education for nurses who had finished basic "training school" education

ANS:1 Local health departments were formed to handle environmental issues in cities.

During America's Industrial Revolution, the number of jobs for women rapidly increased. Nightingale's successes became known across the United States, and the first nursing schools opened. Which of the following occurred related to public health nursing? 1.Community-oriented nursing began with organizations formed to meet urban health care needs. 2.Nurses were instrumental in the construction of sewers and public water systems. 3.Nurses trained in hospitals worked long hours caring for patients with communicable diseases in a humane way. 4.Nurses closed down almshouses and orphanages.

ANS:1 The visiting nurse became the key to health care in urban areas. Although nurses were involved in interventions to improve sanitation and nutrition, they were not instrumental in their construction. Nurses worked in hospitals, but that was acute care. Almshouses and orphanages were not closed down at this time.

Lina Rogers became the first school nurse. Early school nursing focused on: 1.Investigating causes of absenteeism such as malnourishment and lack of shoes or clothing 2.Teaching school as well as being a nurse 3.Starting the first school of public health 4.Providing medical treatment to enable children to return to school

ANS:1 Early school nursing focused on investigating causes of absenteeism, not providing medical treatment. That was the responsibility of physicians.

Florence Nightingale's contributions to public health included: 1.Caring for the sick, poor, and neglected in institutions and at home 2.Using a population-based approach that led to improvements in environmental conditions 3.Writing the Elizabethan Poor Law to guarantee medical care for all 4.Founding of the district nursing association to provide health care to needy people

ANS:2 During the Crimean War, Nightingale progressively improved the soldiers' health by adopting a population-based approach that used simple epidemiological measures and greatly decreased mortality.

Lillian Wald invented the term public health nursing. Which of the following classes might a person of her day attend? 1.Taking and recording blood pressures accurately 2.Safe and sanitary baby and child care 3.Environmental pollutants and their effects on lung disease 4.Time management: balancing factory work and the home

ANS:2 Lillian Wald provided health care that included educating the community on health care matters.

The National Organization for Public Health Nursing was formed in 1912. Lillian Wald was its first president. The mission was to improve the educational and services standards of the public health nurse and promote public understanding. Which of the following is a contribution of this organization? 1.Requiring that public health nurses have a baccalaureate degree in nursing 2.Standardization of public health nursing education 3.Development of nursing cooperatives 4.Opening of the Henry Street Settlement

ANS:2 The National Organization for Public Health Nursing sought to standardize public health nursing. The Henry Street Settlement was already in existence. The baccalaureate degree in nursing was not developed yet.

Threats to health from communicable diseases, the environment, chronic illness, and the aging process have changed over time. The newer threats to health in the United States that community health nurses are currently faced with include: 1.Diphtheria, cholera, and typhoid fever 2.HIV, AIDS, and bioterrorism 3.Avian flu, tuberculosis, and radiation 4.Polluted water and air

ANS:2 The newer threats to health that public health nurses are involved in are HIV, AIDS, and bioterrorism. Choice #1 lists threats of the past. Avian flu may be a threat, but radiation and tuberculosis are ongoing. Polluted water and air are generally not seen in the United States.

A nurse planned a presentation about the latest trends in disaster planning for the senior nursing students at the local college. However, when the nurse began to share the information, the students were talking to one another and essentially ignoring the nurse. Which of the following actions should be taken by the nurse? a. Ask the students why they are being so rude. b. Explain why the information is crucial to their current clinical practice. c. Nothing; let the instructor of the course handle the problem. d. Tell a joke to get the students' attention.

B

A community health nurse obtains data about the community from primary sources, including which of the following? A) World Health Organization B) Community members C) State health department D) Other community health nurses

B Feedback: Primary sources of information include community members, including formal leaders, informal leaders, and community members, and can frequently offer the most accurate insights and comprehensive information. Information gathered by talking to people provides primary data, because the data are obtained directly from the community. The World Health Organization is an international source of information. State health department is a state source of data. Other community health nurses as well as health team members, client records, community health (vital) statistics, census bureau data, reference books, and research reports are secondary sources of information.

The community health nurse is assessing the health of a community by reviewing the dimension of process. Which of the following is true of this dimension? A) Process of a community refers to its services and resources. B) Process reflects the community's ability to function effectively. C) It usually consists of morbidity and mortality data. D) It refers to the physical, emotional and social determinants of health.

B Feedback: Process reflects the community's ability to function effectively. Structure of a community refers to its services and resources. Status/people dimension usually refers to the morbidity and mortality data and the physical, emotional, and social determinants of health.

When applying community development theory, which of the following would most likely be considered the agent of change? A) Clients B) Nurse C) Families D) Community leaders

B Feedback: When applying community development theory, the agent of change is most often the community health nurse who is considered a partner rather than an authority figure responsible for the community's health. Clients, families, and community leaders would be the care recipients.

A nurse is working with a lesbian client. Which statement by the client would indicate a need for further teaching? "Because I am in a homosexual relationship, I ... a. "... should notify my partner(s) if I test positive for any sexually transmitted disease." b. "... do not need to be screened for STDs." c. "... will get a mammogram every two years." d. "... will need to be screened for cervical cancer."

B Correct: This patient does need screening, especially if the relationship is not monogamous.

Which one of the following statements about the benefits of universal design for all persons is the most accurate? A) It is the right thing to do to allow access to the built environment for all persons including persons with disabilities. B)A healthier population may be achieved with attention to the environmental barriers that impede healthy lifestyles for all persons, including those with chronic or disabling conditions. C)The built environment refers to the naturally occurring features of the planet. D)Access implies universal design.

B Feedback: A healthier population may be achieved with attention to the environmental barriers that impede healthy lifestyles for all persons, including those with chronic or disabling conditions. Universal design is the right thing to do for persons with disabilities but is not the most accurate or best reason to ensure universal design. The built environment refers to the physical parts of where we live and work (e.g., homes, buildings, streets, open spaces, and infrastructure). Universal design incorporates access, but access does not necessarily imply universal design.

A community health nurse is reviewing the effects of the Americans with Disabilities Act of 1990. Which of the following would the nurse identify as a major result of this legislation? A)Employers with five or more employees must abide by the American with Disabilities Act. B)Architectural barriers must be removed from pathways so that they are accessible by wheelchair. C)The disabled are eligible to receive telephone services in their home at no cost to them. D) Individuals with disabilities who wish to vote must be provided transportation to polling places to vote.

B Feedback: A major accomplishment of this law was the removal of architectural barriers

A local community health center is providing classes to childbearing families about the effects of alcohol use during pregnancy. During one of the classes, a community health nurse describes fetal alcohol spectrum disorder (FASD). Which of the following would the nurse include as a common finding in the newborn? A) Large for gestational age B) Facial abnormalities C) Hypoactive behavior D) Enhanced intellectual development

B Feedback: A newborn with FASD typically exhibits facial abnormalities, is small for gestational age, demonstrates hyperactivity and intellectual impairment.

As one aspect of ensuring adequate brain development in a child, a community health nurse would encourage the intake of which during the first 2 years of life? A) Vitamins B) Fats C) Protein D) Carbohydrates

B Feedback: During the first 2 years, when rapid myelination is taking place, 50% of total calories should come from fat, but after age 2, the choice of 1% or 2% milk—should be the norm. A well-balanced diet including an adequate intake of vitamins, protein, and carbohydrates is needed

A community health nurse is planning to apply the International Classification of Functioning, Disability, and Health to clinical care in the community. The nurse could use this document as a basis for which of the following? A)Curriculum design B)Needs assessment C)Quality of life measures D)Policy design

B Feedback: In clinical care, the ICF document would be helpful with a needs assessment. Using the ICF document for curriculum design would reflect education

A community health nurse is developing a plan to address the nutritional needs of infants, toddlers, and preschoolers. Which of the following would the nurse need to incorporate as a major issue associated with nutritional problems? A) Life in the culture of poverty B) Overfeeding of an infant C) Cultural food preferences D) Rapid growth spurts

B Feedback: Overfeeding an infant can lead to childhood obesity and becomes a risk factor for heart disease, hypertension, and diabetes. Many parents overfeed infants and toddlers and allow preschool-aged children to make inappropriate food choices, which can lead to a lifetime of nutritional problems. Poverty may limit food choices and selection of inappropriate foods

Which of the following is a major difference between Healthy People 2010 and Healthy People 2020 with respect to people with disabilities? A)Healthy People 2010 emphasized the disparities that persons with disabilities experience. B)The emphasis on "secondary conditions" in Healthy People 2010 has been replaced in Healthy People 2020 with a concern for health disparities for people with disabilities. C)People with disabilities do not experience health disparities any more than people in the general population. D)Healthy People 2020 continues to emphasize secondary conditions.

B Feedback: The emphasis on "secondary conditions" in Healthy People 2010 has been replaced in Healthy People 2020 with a concern for health disparities for people with disabilities. Key to addressing the barriers is for people with disabilities to "(1) be included in public health activities

Which one of the following is a principle that guides and enhances family nursing practice? A) The nurse should expect that the family will be normal and will not change. B) The nurse should start where the family is at the present time and not the ideal level of functioning. C) The nurse should focus on each family member individually. D) The nurse should evaluate the family based on consistency with traditional family patterns.

B Feedback: When working with families, community health nurses begin at the present, not the ideal level of functioning. This is accomplished after the nurse views the family collectively, not individually. The nurse should expect that what is normal for one family is not necessarily normal for another and that families are constantly changing. The nurse should recognize the validity of family structure variations.

A community health nurse is making a home visit to a family. The family has a son with a disability who requires a motorized wheelchair. The family asks for the nurse's assistance. Which response by the nurse would be most appropriate? A)"His disability should automatically cover the cost of the chair." B)"We'll need to check with the insurance company about coverage." C)"You'll probably be able to get it if you pay for half the cost." D)"You might be able to get a regular wheelchair, but a motorized one is impossible."

B Feedback: One major obstacle for families with a disabled or chronically ill member may be obtaining needed assistive devices and technology. Just because the technology exists does not mean that it can be obtained. Often the insurance carrier, whether private or governmental, sets limits on which products can be obtained or which brands are acceptable. Thus the nurse working with the family would need to check with the insurance company first and determine what if anything is covered and then determine how to go about obtaining what the son needs.

The nurse educator knows that the nursing student has grasped the concept of self- evaluation when the nursing student makes which one of the following statements? A) It is important for the family to evaluate itself. B) It is important for the nurse's growth and effectiveness as a community health nurse. C) Evaluations by others are not helpful. D) Individuals can always see their own strengths or flaws.

B Self-evaluation is important for the nurse's growth and effectiveness as a community health nurse. Self-evaluation is referring to the nurse's growth and not the family's. Sometimes, we cannot see our own strengths or flaws, and evaluations by others are helpful.

A community health nurse determines that it is time for teaching a family about health promotion activities when they say which of the following? A) "Is there a place we can go for the medicine?" B) "It's time we do something about eating right." C) "When are you coming back to visit?" D) "Our grandchildren visit every summer."

B Teaching health promotion activities should begin only after family members express an interest and recognize a need, such as the statement about eating right. Asking about where to get medicine and when the nurse is coming back do not reflect an interest or need. The statement about grandchildren reflects information about the family structure and demographics.

A nurse just finished teaching breast self-examination to a large group of women at a professional conference. During the session, she distributed literature and used culturally appropriate visual aids. However, the session was not as effective as it could have been. Which of the following was the most important thing omitted by the nurse? a. Time for audience members to ask questions and clarify the information b. Explanation of why culturally appropriate images are more acceptable c. Opportunity for the women to practice what they learned d. Use of simple language instead of printed material

C

The leader of an Alzheimer's support group surveys the members of the group to determine the best time for the group to meet. Which of the following norms is being supported through this action? a. Group norms b. Task norms c. Maintenance norms d. Reality norms

C

The nurse has just taught a client newly diagnosed with diabetes how to administer sliding-scale insulin. The most effective way to evaluate learning is to: a. provide an online test module. b. ask whether there are any questions. c. ask for a return demonstration. d. give a short paper-and-pencil quiz.

C

As part of an orientation for a group of newly hired nurses for a community agency, a community health nurse emphasizes which of the following as crucial? A) Cyclic nature B) Client focus C) Interaction D) Flexibility

C Feedback: Although the nursing process is cyclical, deliberative, flexible (adaptable), client focused, and need oriented, interaction is an essential first consideration for community health nursing. All steps of the nursing process depend on interaction.

A community health nurse involved in assessing community health needs is planning to get a group of approximately 10 similar individuals together to obtain information about opinions on first pregnancies. The nurse is using which assessment method? A) Survey B) Community forum C) Focus group D) Descriptive epidemiology

C Feedback: The nurse is using a focus group, similar to the community forum or town hall meeting in that it is designed to obtain grassroots opinion. Focus groups usually have only a small group of participants, usually 5 to 15 people and the members chosen for the group are homogeneous with respect to specific demographic variables. A survey involves a series of questions, often to provide a broad range of data. Community forum is a qualitative method to obtain community opinions the members typically represent all segments of the community involved with the issue. Descriptive epidemiologic studies examine the amount and distribution of a disease or health condition in a population by person, place, and time.

Nursing students in a community health nursing course identified toxic waste disposal to be a major problem in their community. The most cost-effective type of community assessment to determine the extent of the problem and the resources available to handle it would be a A) familiarization assessment. B) community subsystem assessment. C) problem-oriented assessment. D) comprehensive community assessment.

C Feedback: The problem-oriented assessment is commonly used when familiarization is not sufficient and a comprehensive assessment is too expensive and not needed and a subsystem assessment is too narrow to determine the extent of the problem.

A community health nurse is making a family home visit. The family consists of the mother, father, 1-year-old, and 4-year-old. The mother states that the 4-year-old who is in preschool had a cold last week and now the 1-year-old has it. She asks the nurse if she can use the over-the-counter cold medication that she gave to the 4-year-old for the 1-year-old. Which response would be most appropriate? A) "It shouldn't be a problem if you use that same medication for the 1-year-old." B) "I don't know if the medicine will work, but you can try it and see." C) "That type of medicine should not be used in children under age 2." D) "You should use a smaller dose but watch if it makes him irritable."

C Community health nurses need to emphasize that over-the-counter cough and cold medications should not be used for children under age 2. The Food and Drug Administration (FDA) deemed them unsafe and ineffective and most manufacturers took them off the market in fall of 2007 in response to emergency room visits and deaths linked to their toxic effects. Therefore, the nurse would tell the mother not to use the medicine since the child is only a 1-year-old.

A couple who are gay have adopted a child. The child's new grandfather says, "I'm worried about the effects of their relationship on my grandchild." How should the population health nurse respond? a. "The child will be adversely affected." b. "Abuse potential is higher among same sex couples." c. "Parenteral attachment to the child is the primary factor in successful parenting." d. "The child will be more likely to be gay as an adult."

C Correct: Children adopted by GLBT individuals tend to develop secure parental attachments and grow up as successfully as children in heterosexual families.

A couple who are lesbian would like to adopt a child. Which information will the population health nurse provide? a. This adoption will be prohibited by Federal law. b. This adoption will be prohibited by State law in all states. c. Most adoption agencies will accept applications from GLBT persons. d. Sexual orientation is not a significant factor in adoption.

C Correct: While this is a true statement, the percentage of agencies reporting placement of a child with a GLBT family or individual is much lower.

A community health nurse constructs an eco- map for a family based on the understanding that this tool is useful for which reason? A) Family relationships over three or more generations are depicted. B) The ecological system of a family's neighborhood is charted. C) It was originally devised to depict the complexity of the client's story. D) Directions for gathering data about neighborhoods are provided.

C Feedback: An eco-map depicts the complexity of the client's story. Lines are drawn to indicate connections to other systems with arrows signifying the direction of energy or flow of resources, and the absence of lines indicates a lack of connections. The family's neighborhood is one component, but not the central focus of the eco-map. A genogram displays family information about complex family patterns such as family relationships over three or more generations, arrows signify the direction of energy or flow of resources, and absence of lines indicates a lack of connections.

When working with the community to develop effective plans to meet the needs of people with disabilities, the community health nurse reviews the statistics involving individuals with disabilities. Which of the following would the nurse identify as accurately reflecting the number of persons with ongoing disability? A)10 million people—less than 5% of the US population B)27 million people—about 10% of the US population C)36 million people—about 12% of the US population D)150 million people—almost 50% of the US population

C Feedback: An estimated 36 million people or 12% of the population is living with some ongoing level of disability.

A community health nurse is working with a group of pregnant women in the community to reduce lifestyle risk factors associated with low-birth-weight newborns and infant mortality. Which of the following would the nurse address? A) Multiparity B) College level education C) Environmental toxins D) Single gestation pregnancy

C Feedback: Environmental toxins are lifestyle risk factors associated with low-birth-weight newborns and infant mortality. Other risk factors include primiparity, low educational level, and multiple gestation among others.

After teaching a group of pregnant women about the need for proper oral health during pregnancy, which of the following statements by the group indicates the need for additional teaching? A) "We should continue seeing the dentist like we always do." B) "Chewing sugar-free gum is okay even with the pregnancy." C) "If we need dental work, we can postpone it until after the birth." D) "Flossing and brushing are just as important now as before."

C Feedback: Postponing dental work is inappropriate. Dental health procedures have generally been found to be effective and safe for pregnant women, especially during the second trimester. Statements about the importance of regular dental health checkups and proper dental hygiene, along with referrals for dental treatment when needed, indicate effective teaching. Sugar-free gums that contain xylitol and chlorhexidine may be helpful in reducing the maternal-child transmission of caries- causing bacteria.

community health nurse to take his or her shopping to purchase some of the food items he or she needs for his or her diet. Which response by the nurse would be most appropriate? A) "I can't, I'm going in a totally different direction." B) "You need to find some other way to get to the store." C) "I'll help you find a way to get to the store." D) "Use the foods you have in the house and shop next week."

C Feedback: The nurse needs to empower the client and helping the client find a way to get to the store promotes the skill of planning so that the client can begin to manage her own needs effectively. Telling the client that the nurse is going in a different direction, telling him or her to find some other way to get to the store, and telling him or her to use the foods in the house are neither therapeutic nor do they promote empowerment.

During a home visit and assessment of an infant and new mother, the nurse determines that the infant looks healthy. The mother asks several questions and listens attentively. One question she asks is whether the nurse is going to visit her again. Which response would be most appropriate? A) "Your baby looks healthy. You should not have any further questions." B) "The agency limits the number of visits I can make; I will let you know." C) "I plan to visit again. What would you like to accomplish at the next visit." D) "I can come weekly for the next 16 weeks, and then a staff nurse will visit."

C Feedback: The response about what the client wants to accomplish together represents a mutual approach, giving power to the client and hints at accomplishing things together on subsequent visits. Noting that the infant looks good and that the mother should not have further questions is closed-ended and nurse- focused and does not promote open communication between the client and the nurse. The option about limiting visits is inaccurate; the nurse has more control over the number of visits than this response implies, unless the visit is part of a special and very limited program. Even then it would not be phrased this way. The final option about 16 weekly visits followed by a staff nurse sounds overwhelming and unrealistic. Very few clients receive this much service from public health agencies.

After teaching a group of students about leading health risks, the instructor determines that the teaching was successful when the students identify which of the following as the #1 leading health risk? A)Unsafe sex B)High blood pressure C)Underweight D)Tobacco consumption

C Feedback: The ten leading health risks are (1) underweight

When reviewing the concept of universal design with a group of community developers, the community health nurse recognizes that the emphasis is on which of the following as the underlying theme? A)Disability needs B)Adaptation C)Access D)Safety

C Feedback: The underlying theme of universal design is ensuring access. Universal design is the design of products and environments to be usable by all people, including those with disability needs as well as chronic illness. The concept incorporated issues of adaptive design and safety measures such as reinforcement in bathroom walls to allow for installation of grab bars, but the key issue is accessibility.

A group of nursing students are studying for a test on child health statistics. The students demonstrate that they are prepared for the test when they identify which of the following as a major cause of death in the 1- to 4-year-old population. A) Pneumonia B) Poverty C) Unintentional injuries and homicide D) Heart disease

C Feedback: Unintentional injuries and homicide cause most of the deaths for the 1- to 4-year-old population: motor vehicle crashes, falls, drowning, fires, and burns. Pneumonia is not a major cause of death among children. Poverty contributes to morbidity and mortality but is not a cause of death in and of itself. Heart disease is the fourth leading cause of death among young children.

After teaching a class about the International Classification of Functioning, Disability, and Health (ICF), the instructor determines that the teaching was successful when the students identify which of the following as a personal factor? A)Problems with ambulation B)Working C)Coping styles D)Amputation

C Feedback: Personal factors are the features of an individual's background, life, and living that are not part of a health condition or health status, such as gender, race, age, other health conditions, fitness, lifestyle habits, upbringing, coping styles, social background, education, profession, past and current experience, overall behavior pattern and character style, individual psychological assets, and other characteristics. Problems with ambulation would be considered an activity limitation. Working would be an activity. Amputation would be an impairment.

During a home postpartum visit, a community health nurse suspects that the woman is experiencing depression. Which of the following responses would be most appropriate? A) "You should try drinking more caffeine to help you be less tired." B) "Don't worry. Good mothers don't hurt their babies." C) "Try napping when the baby is napping." D) "You should be able to achieve perfection with housekeeping tasks while you are still on maternity leave."

C Getting adequate sleep is important, because sleep deprivation exacerbates psychiatric symptoms. Napping when the baby naps, resting when possible throughout the day, and going to bed early (albeit with the knowledge that sleep may be interrupted two or more times to feed the infant) will provide more hours of rest and sleep. Caffeine can lead to sleep disturbance, and alcohol is a depressant that has been implicated in depression. The elimination of both is a simple yet helpful suggestion. The statement about telling the woman not worry discounts her feelings. In addition, it also may increase her anxiety because she may be worried about hurting the baby. The statement about ability to achieve perfection with housekeeping tasks while on maternity leave is counterproductive as it has been recently identified perfectionism increases the risk for postpartum depression.

After teaching a group of students about various community nutritional programs, the instructor determines that the teaching has been successful when the students identify which of the following as a result of the WIC program? A) Increased expenditures for health care B) Decreased rates of large for gestational age babies C) Reduced rates of infant iron deficiency anemia D) Drop in breast-feeding rates

C In addition to supporting women and young children with nutritious foods and achieving the initial goals of decreasing the rates of preterm and LBW babies, increasing the length of pregnancy, and reducing the incidence of infant and child iron deficiency anemia, WIC also increases breast-feeding rates and improves pregnant women's nutritional status.

Ann is a psychiatric home health nurse. She has just received an order to begin regular visits to Mrs. W, a 78-year-old widow who lives alone. Mrs. W's primary care physician has diagnosed her as depressed. Which of these potential problems is a priority to evaluate during the first home visit? A. Complicated grieving B. Social isolation C. Risk for injury D. Sleep pattern disturbance

C. Risk for injury

A community health nurse avoids focusing care on illness and health problems, considering clients as which of the following? A) Aggregate B) Agent C) Community D) Total system

D Feedback: It was proposed by Mundinger and Jauron that the use of nursing diagnoses in the community could be formulated by substituting the terms, client, family, group, or aggregate for the word patient. Community health nurses look for evidence of all kinds of needs that relate to or influence a client's levels of wellness. Needs cover the whole span of the health-illness continuum and the total person, family, group, aggregate, population, or community—the total system. Not all clients are communities. They may be individuals or families. Not all clients are in the category of groups, aggregates, or populations. A client can be an individual or a family. When using the epidemiological triad, there are three components: host, agent, and environment. A client would be considered the host. An agent is a factor that causes or contributes to a health problem or condition.

Which one of the following has a negative influence on family health and individual health? The level at which a family functions significantly affects the individual's level of health. B) A healthy family fosters individual growth and sustains members during times of crisis. C) Family patterns dictate whether members participate in their own health care. D) Individuals can obstruct the family's health and families can obstruct individual family members' health.

D Feedback: A negative influence on family health and individual health is that individuals can obstruct the family's health and families can obstruct individual family members' health. The level at which a family functions significantly affects the individual's level of health; that a healthy family fosters individual growth and sustains members during times of crisis; and the influence of families that dictate whether members participate in their own health care are all positive influences.

When planning for a home visit, which of the following would be most helpful to ensure a successful home visit? A) Documentation in a timely manner B) Summarizing the main visit points C) Providing incidental teaching D) Scheduling an appropriate visit time

D Feedback: As part of planning the community health nurse would contact the family to set up an appropriate time for the visit. Documenting in a timely manner ensures successful evaluation. Providing incidental teaching and summarizing the main visit points help to ensure successful implementation.

When a community health nurse is conducting a family assessment on an assigned family, which of the following would be most appropriate? A) Use quantitative data only to maintain and preserve objectivity B) Interview one family member to avoid confusion and repeated information C) Use a checklist format, completing the tool in the family's presence D) Make several visits and accumulate data from all family members

D Feedback: Completing a family assessment takes time. It is better to keep notes from several visits with the family and observe the family as a group during some family activity. The nurse should not use obtrusive questionnaire techniques or take notes in the family's presence. Quantitative data will give a one-sided view of family data. It is best to interview all family members over time. The nurse should collect both quantitative and qualitative data, which will provide a rich family assessment.

An instructor is reviewing the principles that resulted from the 2011 reassessment of global progress on addressing disability with a class. Which of the following would the nurse include? A)The government must focus solely on seeking improvement in the lives of individuals and families living with disability. B)Government must play the only role in addressing disability. C)It is unimportant for service providers, academic institutions, the private sector, communities, and people with disabilities and their families to address disability. D)The importance of all persons, including those with disabilities to participate in their country's development.

D Feedback: In 2011, the World Health Organization and the World Bank reassessed global progress on addressing disability in light of the 2006 Convention on the Rights of Persons with Disabilities. It was determined that not only must governments seek improvements in the lives of individuals and families living with disability, but every citizen needs to participate in their country's development. Government at every level may play a most significant role, but there are important roles for service providers, academic institutions, the private sector, communities, and especially people with disabilities and their families.

When assessing a family, which of the following would lead the nurse to question the health of a family? A) Role relationships are inflexible. B) Coping is actively attempted. C) Family members communicate regularly. D) The family lacks regular links with the broader community.

D Feedback: In a healthy family, role relationships are structured effectively so that they can change with changing family needs. Active coping, regular family communication, and regular links with the broader community reflect a healthy family.

A community health nurse is providing information handouts that describe the need for annual gynecologic exams to local health clinics that provide care for individuals with disabilities. The nurse is fulfilling which role? A)Advocate B)Coordinator C)Researcher D)Educator

D Feedback: Providing information handouts is a method for teaching. Therefore, the nurse is assuming the role of an educator at the community level. In the advocate role, the nurse would assist in arranging for the population to receive the appropriate screening, ensuring that any special modifications or adaptations would be available. In the coordinator role, the nurse might arrange for screening to be performed during a routine visit made by a client to a facility or make arrangements for transporting the clients to a facility for the screening. In the researcher role, the nurse might contact other community health nurses about their experience in ensuring screening for this population to discover if they have similar needs or problems.

A community health nurse is working with a group of visually impaired individuals and is developing a program to promote self- confidence and self-respect. Which of the following organizations would be a valuable resource for the nurse? A)American Foundation for the Blind B)American Council of the Blind C)Guide Dogs for the Blind D) National Federation of the Blind

D Feedback: The nurse would contact the National Federation of the Blind whose focus is on helping blind individuals achieve self- confidence and self-respect. The American Foundation for the Blind advocates for the visually impaired through increased state and federal funding. The American Council of the Blind provides services such as information and referral, scholarship assistance, public education, and industry consultation, governmental monitoring, consultation, and advocacy. The Guide Dogs for the Blind makes guide dogs available for the visually impaired.

A community health nurse is developing a plan of care for a family who is providing round-the-clock care in their home for a daughter who is chronically ill. Which of the following would be most important for the nurse to integrate into the plan? A)Insurance limitations B)Skill teaching C)Flexible visiting D)Respite care

D Feedback: Respite care is another area of great importance for families of the disabled and the chronically ill. It can be emotionally draining to meet the daily needs of a member who cannot perform self-care. This often leads to caregiver fatigue and increased stress. It is also important to recognize the effect of the situation on noncaregivers in the family, particularly nondisabled siblings of a disabled child. Respite care offers some needed relief to the family and allows for uninterrupted attention to the nondisabled children. Whatever the source, some type of respite care is often vital to the family's health and should be a priority in the overall treatment plan of the family. Although insurance considerations, teaching, and flexible visiting would be issues to be addressed, respite care is a priority.

While providing preconception counseling to a group of women, the community health nurse emphasizes the need for adequate weight gain during pregnancy based on the understanding of which of the following? A) Maternal nutritional habits do not affect later life. B) Inadequate weight gain increases the woman's risk for cesarean birth. C) Prolonged labor can occur with too little or too much weight gain. D) Adequate weight gain is positively correlated with normal birth weight.

D Research has demonstrated a positive correlation between weight gain during pregnancy and normal birth weight in babies. Inadequate weight gain is related to LBW, premature births, and perinatal mortality. Gaining large amounts of weight can also cause problems at birth, including increased numbers of cesarean deliveries, large-for- gestational-age babies, along with the mother's retention of excessive weight. Maternal nutritional habits set gene switches that affect later life.

A community health nurse implements a teaching program for new parents about ways to reduce infant mortality. Which suggestion would the nurse include as most important? A) "Never leave the infant alone on a dressing table or couch." B) "Check the temperature of the bath water before bathing." C) "Stay within an arm's length when the baby is in the tub." D) "Put the infant to sleep on his or her back."

D Suffocation is the leading cause of death in infants. Therefore, the suggestion to put the baby to sleep on the back is a major preventive measure and the most important. Never leaving the infant alone would reduce the risk for falls, a leading case of nonfatal injuries for infants. Checking bath water temperature is appropriate to prevent burns. Staying within an arm's length while bathing is a suggestion to prevent drowning.

While making a family health visit, an older relative who is visiting has been drinking and becomes verbally abusive and increasingly loud. Which action by the community health nurse would be best? A) Continue the visit with caution B) Suggest the relative go in another room and take a nap C) Ask the sober family members to take the visitor home D) Terminate the visit, making plans for another visit

D Terminating the visit is the best choice. It appears the problem is escalating, and the nurse's safety should always come first. Continuing the visit puts the nurse and possibly the other family members at risk. Making a suggestion to go into another room and take a nap may agitate the relative and make a deteriorating situation worse. Making a suggestion for a family member to take the visitor home may agitate the relative and make a deteriorating situation worse. Additionally, this is the client's home and the suggestion should be the client's.

A nurse considers an audience's ability to read, comprehend, and act on medical instructions while preparing health education materials. Which factor is the nurse considering? a. Health literacy b. Ethnicity c. Medical training d. Disparity

a. Health literacy

The purpose of public health core functions is to: a. Clarify the role of the government in fulfilling the mission of public health. b. Ensure the safety of populations in receiving quality health care. c. Provide community-based individualized care to every person in the United States. d. Unite public and private providers of care in a comprehensive approach to providing health care.

a. Clarify the role of the government in fulfilling the mission of public health.

A public health nurse uses Assumption 2, "Public health nursing practice focuses on populations," to guide practice. Which would be considered a population of interest? a. Healthy school children b. Homeless individuals c. A person recently diagnosed with diabetes d. Teenage parents

a. Healthy school children;

The Affordable Care Act of 2010 included several elements that involve health planning. These elements include (Select all that apply.) a. Reducing premium costs for millions of working families and small businesses b. Capping out-of-pocket expenses c. Prohibiting denial of coverage because of a preexisting condition d. Covering the cost of preventive care e. Establishing regional cooperative arrangements among medical schools, research institutions, and hospitals to improve the health manpower and facilities available to the communities.

a, b, c, d, Provisions from the Affordable Care Act of 2010 included: Reducing premium costs for millions of working families and small businesses by providing hundreds of billions of dollars in tax relief—the largest middle-class tax cut for health care in history. Reducing what families will have to pay for health care by capping out-of-pocket expenses and requiring preventive care to be fully covered without any out-of-pocket expense. Keeping insurance companies honest by setting clear rules that rein in the worst insurance industry abuses. Prohibiting insurance companies from denying insurance coverage because of a person's preexisting medical conditions while giving consumers new power to appeal insurance company decisions that deny doctor-ordered treatments covered by insurance (U.S. Department of Health and Human Services, 2017). Regional Medical Programs were intended to establish regional cooperative arrangements among medical schools, research institutions, and hospitals to improve the health manpower and facilities available to the communities.

What action would the nurse take when triaging victims following a disaster? a. Allocation of treatment based on the victim's potential for survival b. Assignment of tasks to the appropriate disaster response personnel c. Determination of the level of disaster and agency involvement d. Participation in community preparedness for the disaster response

a. Allocation of treatment based on the victim's potential for survival

Which statement is true about mortality rates? Mortality rates: a. Are informative only for fatal diseases b. Provide information about existing disease in the population c. Are calculated using a population estimate at year-end d. Reveal the risk of getting a particular disease

a. Are informative only for fatal diseases

Collecting data and monitoring the health status of the population defines which of the core public health functions? a.Assessment b.Prevention c.Assurance d.Policy development

a. Assessment

What are the five "rights" of case management? a. Care, time, provider, setting, price b. Patient, medication, route, time, documentation c. Place, setting, patient, plan, outcomes d. Disease process, time, place, beneficence, advocate

a. Care, time, provider, setting, price

An example of a health disparity is: a. Childhood immunization rates b. High dropout rates c. Unemployment d. Income below poverty level

a. Childhood immunization rates

The levels of practice encompassed by the Intervention Wheel are: a. Communities, individuals and families, and systems b. Assessment, diagnosis, and evaluation c. Primary, secondary, and tertiary d. Communities, populations, and aggregates

a. Communities, individuals and families, and systems;

A mental health nurse working in the 1960s received additional education to provide care for the severely mentally ill in the community because of which legislation? a. Community Mental Health Centers Act b. Stewart B. McKinney Homeless Act c. National Health Planning and Resource Development Act d. Patient Protection and Affordable Care Act

a. Community Mental Health Centers Act

Which trends are occurring among vulnerable populations? Select all that apply. a. Community-based care and interorganizational partnerships b. Outreach and case finding c. Elimination of disparities d. Culturally and linguistically appropriate care

a. Community-based care and interorganizational partnerships b. Outreach and case finding d. Culturally and linguistically appropriate care

The nurse investigating environmental health problems caused by contaminated ground water is best described as practicing: a.Community-oriented nursing b.Community-based nursing c.Policy development d.Tertiary care

a. Community-oriented nursing

A study that uses information on current health status, personal characteristics, and potential risk factors or exposures all at once is called: a. Cross-sectional b. Ecological c. Case-control d. Cohort

a. Cross-sectional

The public health workforce should demonstrate competency in which of the following competency categories? a.Financial planning and management b.Workforce needs assessment c.Acute care services d.Curriculum development

a. Financial planning and management

When working in a disaster, which attribute would be most important for the nurse to possess? a. Flexibility b. Training c. Experience d. Special interest

a. Flexibility

The cornerstones of public health nursing practice (select all that apply): a. Focus on the health of the entire population b. Reflect community priorities and needs c. Promote health through strategies driven by epidemiological evidence d. Are grounded in an ethic of collaboration

a. Focus on the health of the entire population b. Reflect community priorities and needs c. Promote health through strategies driven by epidemiological evidence

Which is a characteristic of a vulnerable population? a. Have worse health outcomes and an increased sensitivity to risk factors than the general population b. Have a single risk factor but experience worse health outcomes than the general population c. Have multiple risk factors but equal health outcomes than the general population d. Have worse outcomes with better access to health care than the general population

a. Have worse health outcomes and an increased sensitivity to risk factors than the general population

Which statement about the Intervention Wheel is true? a. It provides a graphic illustration of population-based public health practice. b. It describes in detail the components of public health nursing. c. It demonstrates the practice of community health nurses for policy and lawmakers. d. It is a framework used by all health departments in the United States.

a. It provides a graphic illustration of population-based public health practice.

What knowledge and skills are required in order to become a competent case manager? Select all that apply. a. Knowledge of community resources and financing mechanisms b. Written and oral communication skills c. Proficient negotiation and conflict-resolving practices d. Application of evidence-based practices and outcomes measurements

a. Knowledge of community resources and financing mechanisms b. Written and oral communication skills c. Proficient negotiation and conflict-resolving practices d. Application of evidence-based practices and outcomes measurements

Why must a nurse who is using population management be able to work with integrated care delivery systems? a. Management has shifted from inpatient care to primary care providers as points of entry. b. Emphasis is on episodic illness care for individuals rather than on population management. c. Care management services and programs do not provide access and accountability, as provided by case management services. d. Assessment of the needs of the population is no longer necessary.

a. Management has shifted from inpatient care to primary care providers as points of entry.

A riverfront community builds a retaining wall to divert flood water away from the town. This is an example of: a. Mitigation b. A natural disaster c. Disaster reduction d. Disaster health

a. Mitigation

The process of moving conflicting parties toward an outcome is called: a. Negotiation b. Conflict management c. Problem-purpose expansion method d. Brainstorming

a. Negotiation

Proposed changes to the health care delivery system in the United States will: a.Provide new opportunities for public health specialists. b.Result in isolated care being provided to individuals. c.Emphasize specialty care. d.Increase the utilization of acute care services.

a. Provide new opportunities for public health specialists.

Which is an example of a bioterrorism and emergency readiness competency for a public health worker? a. Recognition of unusual events that might indicate an emergency b. Involvement in the chain of command c. Using a set of preplanned activities for every disaster d. Leaving communication to the authorities

a. Recognition of unusual events that might indicate an emergency

When a nurse examines birth and death certificates during an epidemiologic investigation, what data category is being used? a. Routinely collected data b. Data collected for other purposes but useful for epidemiologic research c. Original data collected for specific epidemiologic studies d. Surveillance data

a. Routinely collected data

A nurse who is involved in identifying individuals with unrecognized health risk factors or asymptomatic disease is using which intervention? a. Screening b. Referral and follow-up c. Surveillance d. Health teaching

a. Screening

Which populations are at greatest risk for disruption after a disaster? Select all that apply. a. Single-parent families b. Children c. Substance abusers d. Middle-class families

a. Single-parent families b. Children c. Substance abusers

Which federal program created support for older and poor Americans? a. Social Security Act b. Medicare Amendment c. Medicaid Amendment d. Hill-Burton Act

a. Social Security Act

The number and proportion of persons aged 25 or older with less than a high school education is an example of: a.Sociodemographic characteristics b.Health status c.Health risk factors d.Health care resource consumption

a. Sociodemographic characteristics

A nurse is working in a temporary shelter for victims following a natural disaster. Which condition is the nurse most likely to encounter? a. Stress b. Communicable disease c. Chronic illness d. Injuries requiring first aid

a. Stress

Which is an example of a human-made disaster? a. Structural collapse b. Communicable disease epidemics c. Mud slides d. Floods

a. Structural collapse

Which is an example of an aggregate or population? a.Students in a county school system b.Christians around the world c.A patient in the intensive care unit at the local hospital d.People who play cards together once a week at private homes

a. Students in a county school system

What is the purpose of the color-coded wedges on the Intervention Wheel? a. The interventions are grouped together in related wedges. b. The wedges consist of referral information for each wedge. c. The element of health teaching is the predominant feature of each wedge. d. Coalition building must be implemented with each wedge.

a. The interventions are grouped together in related wedges.

Which of the following are considered barriers to public health nursing? Select all that apply. a.The mindset that the only role for the nurse is at the bedside. b.The structures within which nurses work and the process of role socialization within those structures. c.Few nurses receive graduate-level preparation in the concepts and strategies of the disciplines basic to public health. d.The number of job opportunities in the area has been steadily declining.

a. The mindset that the only role for the nurse is at the bedside. b. The structures within which nurses work and the process of role socialization within those structures. c. Few nurses receive graduate-level preparation in the concepts and strategies of the disciplines basic to public health.

Implementing quality performance standards in public health is important because: a. They are used to guide improvement in the public health system. b. They rigidly control public health. c. Administrators will not monitor public health at the local level, but instead at the national level. d. They can be used as hiring guidelines for nurses.

a. They are used to guide improvement in the public health system.

Which statement about disasters is true? a. They can be natural or human-made. b. They can be relieved without assistance. c. There is always injury and death when a disaster occurs. d. The timing of a disaster does not influence the types of injuries that will occur.

a. They can be natural or human-made.

Which of the following is an example of the mission of public health according to the Institute of Medicine? a.Tracking avian flu outbreaks and doing surveillance in the United States b.Providing a flu shot for an elderly person at the health department c.Keeping track of alternative therapies in use in the United States d.Keeping snake antivenom at the Centers for Disease Control and Prevention in Atlanta

a. Tracking avian flu outbreaks and doing surveillance in the United States

Before participating in health care planning, a community health nurse must understand the concept of a. community as client. b. individual as client. c. family as client. d. environment as client.

a. community as client. It is essential that community health nurses understand and are comfortable with the concept "community as client" before participating in health care planning. When focusing on the individual or family, nurses must remember that these clients are members of a larger population group or community, and environmental factors influence them.

An example of a community health nursing practice is: a.Administrating a flu shot to a client in a physician's office b.Conducting a flu shot clinic at a community center c.Performing a client assessment in a hospital d.Providing supervision of staff in a rehabilitation center

a.Administrating a flu shot to a client in a physician's office

Nurses incorporate epidemiology into their practice and function in epidemiologic roles through (select all that apply): a. Policy making and enforcement b. Collection, reporting, analysis, and interpretation of data c. Environmental risk communication d. Documentation on patient charts and records

b. Collection, reporting, analysis, and interpretation of data c. Environmental risk communication d. Documentation on patient charts and records

Epidemiologic studies of diseases conducted by nurses during the twentieth century were influenced by the (select all that apply): a. Increasing rate of poverty b. Declining child mortality rates c. Overcrowding in major cities d. Development of new vaccinations

b. Declining child mortality rates d. Development of new vaccinations

In case management, the process of performing interdisciplinary, family, and client conferences would occur during which phase of the nursing process? a. Assessment b. Diagnosis c. Planning/outcome d. Implementation

b. Diagnosis

What emotion would a nurse anticipate when working with an older person following a disaster? a. Anger b. Fear of loss of independence c. Violence d. Regression

b. Fear of loss of independence

A case manager submits documentation that a nursing visit was completed at a client's home, but it was never performed. According to the general areas of legal risk, how would this action best be categorized? a. Liability for managing care b. Fraud/abuse c. Negligent referral d. Confidentiality/security

b. Fraud/abuse

The wide variations in health services and health status between certain population groups are called: a. Priority population groups b. Health disparities c. Disadvantaged populations d. Risk markers

b. Health disparities

The case manager who uses effective collaboration and team strategies to make arrangements for services is performing the role of: a. Broker b. Negotiator c. Liaison d. Facilitator

b. Negotiator

An example of a measurable outcome health status indicator at the individual-level of practice that would be used by a public health nurse is: a. A 50-year-old woman receives annual mammograms b. School absences in a community decline c. Teachers have increased awareness of health problems d. Those in poverty utilize the free mammogram program

b. School absences in a community decline

An example of a vulnerable group experiencing multiple risk factors is: a. Smokers who use chewing tobacco as well as cigarettes b. Substance abusers who test positive for human immunodeficiency virus (HIV) c. Persons with limited access to care because they live in a rural area d. New mothers needing information about baby and child care

b. Substance abusers who test positive for human immunodeficiency virus (HIV)

What is most likely to occur in the community during the Honeymoon Phase after a disaster? a. First responders work tirelessly to save others. b. Survivors share their stories. c. Medical personnel experience exhaustion. d. Community organizations rebuild the community.

b. Survivors share their stories.

Which model addresses the structure and processes of using the population-based tools of disease management and critical pathways to offer care for client populations? a. Client-focused b. System-focused c. Social service d. Long-term care

b. System-focused

Population-based preventive programs launched in the 1970s are responsible for increased: a.Use of tobacco b.Use of automobile safety restraints c.Incidence of hypertension d.Incidence of obesity

b. Use of automobile safety restraints

Systems theory provides a framework that includes interventions that are possible at the three levels of prevention. The level of prevention that includes early diagnosis is a. primary prevention. b. secondary prevention. c. tertiary prevention. d. supralevel prevention.

b. secondary prevention. Secondary prevention includes early diagnosis and treatment to reduce the duration and severity of disease or dysfunction. Tertiary prevention applies to irreversible disability or damage and aims to rehabilitate and restore an optimal level of functioning. Option D is not a type of prevention.

After the implementation of various policies to reduce tobacco use, the tobacco industry has begun targeting (Select all that apply.) a. elderly people. b. Hispanics. c. youths and young adults. d. African Americans. e. international markets.

c, e The tobacco industry has now started targeting youths and dramatically increased international exports. Tobacco sales among American adults is down.

A public health nurse utilizes the nursing process at all levels of practice by: a. Including specific goals for community health nurses b. Developing an accurate nursing diagnosis c. Analyzing the needs of the community, system, and individuals and families d. Utilizing primary, secondary, and tertiary prevention

c. Analyzing the needs of the community, system, and individuals and families

Which of the following statements is true with regard to a population focus in public health nursing? a.Priority is given to the highest risk population. b.Direct caregiving is limited to preventive measures, such as administration of immunizations. c.Attention is given to the population or community as a whole, regardless of whether they do or do not access the health care system. d.Only populations outside institutional settings are considered.

c. Attention is given to the population or community as a whole, regardless of whether they do or do not access the health care system.

Since the Intervention Wheel was first published in 1998, it has: a. Guided national policy b. Been used as a tool in deciding licensure issues for State Boards of Nursing c. Been incorporated into the public health curricula of many nursing programs d. Gained wide acceptance internationally

c. Been incorporated into the public health curricula of many nursing programs d. Gained wide acceptance internationally

How can nurses promote social justice? a. By contacting lawmakers about environmental health issues b. By assisting at homeless shelters c. By advocating for policies to improve social conditions d. By serving on a local coalition to prevent obesity

c. By advocating for policies to improve social conditions

When implementing interventions at the systems-level of practice, the public health nurse would: a. Involve the entire community in solving the health problem. b. Identify health problems in the community. c. Change laws, policies, and practices that influence population-based issues. d. Provide outreach services to populations at risk.

c. Change laws, policies, and practices that influence population-based issues.

A social marketing campaign urging community members to avoid driving motorized vehicles after consuming alcohol is implemented in a local community. This intervention is occurring the _____-level of practice. a. Individual/family b. Systems c. Community d. Government

c. Community

A community health nurse working as a school health nurse conducted a community assessment and determined that the focus of programs and health education for the academic year should address the problem of childhood obesity. In planning, it is important for the nurse to allow time for individual nursing assessment and education for children who are overweight or obese and to plan classroom education programs. The nurse is demonstrating an understanding of which important community health nursing concept? a. Health planning b. Aggregate health care c. Community as client d. Use of the nursing process

c. Community as client Community health nurses focus care on health needs of aggregates

Which is an example of the primary goal of public health? a.Ensuring that a newly diagnosed 40-year-old hypertensive man takes his medication b.Finding home care for a 70-year-old client recuperating from a hip replacement c.Conducting an infant car seat safety check d.Contacting a local hospice to admit a terminally ill 60-year-old woman

c. Conducting an infant car seat safety check

What is the purpose of the National Response Framework? a. Create a new branch of government that deals with bioterrorism. b. Establish a way for the Red Cross to carry out its mission. c. Develop a nationwide all-hazards approach to domestic incident management. d. Extend presidential power to act quickly upon weapons of mass destruction.

c. Develop a nationwide all-hazards approach to domestic incident management.

An example of primary prevention is: a. Pap smear b. Blood pressure screening c. Diet and exercise d. Physical therapy

c. Diet and exercise

The threat of bioterrorism has the potential to: a.Dissolve community-based programs. b.Cause the health care system to collapse. c.Divert funds from other public safety health care programs. d.Increase the need for shelters.

c. Divert funds from other public safety health care programs

Which nursing diagnosis would a public health nurse use when addressing the problem of obesity at the community-level of practice? a. Alteration in nutrition: More than body requirements b. Need for increased knowledge of proper nutrition c. Families at risk for obesity because of inactivity d. Overweight child related to poor dietary habits

c. Families at risk for obesity because of inactivity

When studying chronic disease, the multifactorial etiology of illness is considered. What does this imply? a. Genetics and molecular structure of disease is paramount. b. Single organisms that cause a disease, such as cholera, must be studied in more detail. c. Focus should be on the factors or combinations and levels of factors contributing to disease. d. The recent rise in infectious disease is the main focus.

c. Focus should be on the factors or combinations and levels of factors contributing to disease.

A nurse caring for a homeless population recognizes that those who are homeless: a. Need more nursing care than other vulnerable groups b. Have no desire to seek medical care c. Have even fewer resources than poor people who have adequate housing d. Are living in despair with no hope or resilience

c. Have even fewer resources than poor people who have adequate housing

The public health nurse analyzes data related to the number and type of United States Environmental Protection agency air quality standards that a community failed to meet. This data is an example of using which community health profile indicator? a.Sociodemographic characteristics b.Health status c.Health risk factor d.Functional status

c. Health risk factor

A nursing student develops a teaching plan about hand washing to present to a group of elementary school children at the local school. Which public health intervention is being implemented? a. Collaboration b. Surveillance c. Health teaching d. Screening

c. Health teaching

Which of the following statements about public health is accurate? a.Prevention of early deaths can be more effectively accomplished by medical treatment than by public health approaches. b.Expenditures and resources for public health have increased in recent years. c.Historically, gains in the health of populations have been related largely to changes in safety, sanitation, and personal behavior. d.Reform of the medical insurance system is the single change needed to improve the health of Americans.

c. Historically, gains in the health of populations have been related largely to changes in safety, sanitation, and personal behavior.

A community health nurse is using the Health Planning Model to improve a client's health. The nurse prepares for unexpected problems or complications in the client's care. This attention to unexpected problems takes place during which step? a. Assessment b. Planning c. Intervention d. Evaluation

c. Intervention During the intervention step of the Health Planning Model, the nurse must prepare for unforeseen problems or complications and consider alternatives to meet the client's needs. The nurse gathers information about the needs of the community during the assessment step. During the planning step, the nurse determines which problems or needs require intervention and identifies the desired outcomes or ultimate goals of the interventions. Evaluation determines success or failure of the project.

A public health staff nurse who has a clear understanding of population-focused practice: a.Is frustrated providing care to individuals b.Would rather perform population-focused interventions c.Is able to improve the effectiveness of care provided d.Is considered a public health nurse specialist

c. Is able to improve the effectiveness of care provided

Public health professionals refer to three levels of prevention as tied to specific stages in the: a. Epidemiologic triangle b. Web of causation c. Natural history of disease d. Surveillance process

c. Natural history of disease

The proportion of persons with positive test results who actually have a disease, interpreted as the probability that an individual with a positive test result has the disease, is the: a. Sensitivity b. Specificity c. Positive predictive value d. Negative predictive value

c. Positive predictive value

A nurse is developing a one-stop service to meet the needs of a vulnerable group. Which is an example of this type of service? a. Wrap-around services where mental services are linked b. Giving all immunizations on a single clinic visit c. Providing multiple services during a single clinic visit d. Providing free services to the medically indigent

c. Providing multiple services during a single clinic visit

Which is an example of a priority population group? a. People earning more than $100,000/year b. People earning less than $100,000/year c. Residents of remote rural areas of Canada d. African American physicians

c. Residents of remote rural areas of Canada

A nurse works with a group of abused women to enhance their levels of self-esteem. Which level of prevention is being performed? a. Primary level of prevention b. Secondary level of prevention c. Tertiary level of prevention d. Health promotion

c. Tertiary level of prevention

A public health nurse provides a clinic for HIV-positive citizens in the community. This is an example of: a.Primary prevention b.Secondary prevention c.Tertiary prevention d.Policy making

c. Tertiary prevention

Vocational rehabilitation of a person with a neuromuscular disease is an example of: a. Primary prevention b. Secondary prevention c. Tertiary prevention d. Health promotion

c. Tertiary prevention

The global health organization that works for children's survival, development, and protection is the a. World Health Organization (WHO). b. Centers for Disease Control (CDC). c. United Nations International Children Emergency Fund (UNICEF). d. Pan American Health Organization (PAHO).

c. United Nations International Children Emergency Fund (UNICEF). UNICEF works for children's survival, development, and protection by developing and implementing community-based programs. UNICEF achievements are well documented in child health, nutrition, education, water, sanitation, and progress for women. PAHO is an international public health agency that works to improve the health and living standards of the Americas. The WHO introduced the goal of "health for all." The CDC strives to prevent and control infectious and chronic diseases, injuries, workplace hazards, disabilities, and environmental health threats.

When would a nurse use a critical path? a. When implementing the five "rights" of case management b. When demonstrating competencies required for practicing case management c. When attempting to achieve a measurable outcome for a specific client d. When effectively managing conflict resolution

c. When attempting to achieve a measurable outcome for a specific client

Which case management activity would most likely be implemented by a bachelor's prepared nurse rather than an advanced practice nurse (APN)? a. Working with community aggregates b. Working with systems of disease c. Working with individuals d. Working with outcomes management processes

c. Working with individuals

The community health nurse understands that a client's health can be negatively or positively affected by his or her suprasystem. Which correctly describes a client's suprasystem? The suprasystem includes a. only health-related resources, such as primary care providers and insurance sources. b. only non-health-related resources, such as use of public transportation. c. all resources available for use by the client, including health-related and non-health-related resources. d. all official (governmental) agencies used by the client or family, such as the health department or social services.

c. all resources available for use by the client, including health-related and non-health-related resources. The suprasystem includes a variety of organizations and community resources. Some are health related, such as the presence or absence of hospitals, clinics, primary care providers, or health centers. Support services such as Meals on Wheels, public transportation, and recreational facilities are also important. Governmental agencies used by the client are also assessed as part of the suprasystem.

Population overcrowding can result in a. increased productivity. b. improved education. c. increased incidence of disease. d. decreased mortality.

c. increased incidence of disease. Overcrowding leads to pollution, stress, disease, and violence. Mortality rates are increased, and quality of life decreases.

Which are examples of case-managed conditions? a. Bankruptcy, financial distress, depression b. Flu, colds, frequent headaches c. Malaria, bird flu, Dengue fever d. AIDS, spinal cord injury, ventilator dependency

d. AIDS, spinal cord injury, ventilator dependency

Which nursing intervention does the case manager implement when fulfilling the role of coordinator? a. Providing information to all parties about the situations affecting the client b. Educating the client and providers in order to make informed decisions c. Supporting all parties to work toward mutual goals d. Arranging, regulating, and balancing needed health services for the client

d. Arranging, regulating, and balancing needed health services for the client

What would most likely occur during the preparedness stage of disaster management? a. Heightened inspection and increased security in the community b. Incorporation of provision of pets into local disaster plans c. Purchase of personal protective equipment for all citizens d. Assembly of disaster kits for the home, workplace, and car

d. Assembly of disaster kits for the home, workplace, and car

The nurse who conducts vision screenings on children in the school setting is practicing _____ nursing practice. a.Community-oriented b.Public health c.Community health d.Community-based

d. Community-based

Which is an example of a public health nurse conducting a community assessment? a. Visiting an elderly person at home to assess and evaluate safety and fall risk b. Developing diagnoses to identify nursing interventions at a health clinic c. Evaluating services at an immunization clinic where a translator provides services d. Compiling recent data from the county health department on child abuse cases

d. Compiling recent data from the county health department on child abuse cases

Which type of management would be most appropriate for a nurse to use when helping clients improve their health status? a. Utilization management b. Case management c. Disease management d. Demand management

d. Demand management

The factors, exposures, characteristics, and behaviors that determine patterns of disease are described using: a. Descriptive epidemiology b. Analytic epidemiology c. Distribution d. Determinants

d. Determinants

An example of community preparedness for a disaster is: a. Assembling emergency supplies b. Understanding the workplace disaster plan c. Taking a disaster training course d. Developing an evacuation plan to remove individuals from danger

d. Developing an evacuation plan to remove individuals from danger

Which organization would a nurse collaborate with when developing and coordinating an emergency response plan for a community? a. American Red Cross (ARC) b. Stress Management in Disaster Program (SMID) c. Centers for Disease Control and Prevention (CDC) d. Emergency Management Agency (EMA)

d. Emergency Management Agency (EMA)

A case manager supports a client's decision to return home after having a total hip replacement rather than go to a skilled nursing facility. Which phase of the nursing process is being used? a. Assessment b. Planning/outcome c. Implementation d. Evaluation

d. Evaluation

When the case manager contacts providers and negotiates services and price, which phase of the nursing process is used? a. Assessment b. Diagnosis c. Planning/outcome d. Implementation

d. Implementation

Public health nurses use a common set of interventions to: a. Describe the proper order of implementation. b. Emphasize surveillance as the main focus of public health practice. c. Guide practice and generate agency protocols. d. Improve the health status of communities, systems, individuals, and families.

d. Improve the health status of communities, systems, individuals, and families.

Public health nurses utilize registries to identify children with delayed or missing immunizations. They subsequently follow up with families by phone calls or home visits. This is an example of ____-level of practice. a. Systems b. Community c. Policy d. Individual/family

d. Individual/family;

d. Individual/family

d. Individual/family;A social marketing campaign urging community members to avoid driving motorized vehicles after consuming alcohol is implemented in a local community. This intervention is occurring the _____-level of practice.

Which statement is true about the origins of the Intervention Wheel? a. A panel of nurses from Iowa, Minnesota, North Dakota, South Dakota, and Wisconsin developed and refined the Intervention Wheel. b. It was conceived by a group of international nurses from Norway, Kazakhstan, and Japan. c. It was a result of a qualitative analysis carried out by the State Boards of Nursing. d. It resulted from a grounded theory process carried out by public health consultants at the Minnesota Department of Health.

d. It resulted from a grounded theory process carried out by public health consultants at the Minnesota Department of Health.

Which ethical principle is influenced when care in a managed system is provided by less-experienced providers? a. Autonomy b. Beneficence c. Confidentiality d. Justice

d. Justice

An example of secondary prevention is: a. Rehabilitation b. Avoidance of high-risk behaviors c. Immunization d. Mammogram

d. Mammogram

What legislation resulted in increased consumer involvement in the U.S. health care system and a review of the allocation of health care resources? a. Partnership for Health Program b. Hill-Burton Act c. Regional Medical Services Programs d. National Health Planning and Resources Development Act

d. National Health Planning and Resources Development Act The National Health Planning and Resources Development Act addressed the need for the provider and consumer to be involved in the planning and improving of health services as well as placing the system of private practice examination. The Partnership for Health Program provided federal grants to states to establish and administer a local agency program to enact local comprehensive health care planning. The Hill-Burton Act was passed by Congress to address the need for better hospital access. The Regional Medical Programs intended to make the latest technology for the diagnosis and treatment of heart disease, cancer, stroke, and related diseases available to community health care providers through the establishment of regional cooperative arrangements among medical schools, research institutions, and hospitals.

Public health nurses who develop and implement local public health policies through partnerships with agencies, organizations, and consumers within the community are using which core public health function? a.Assessment b.Prevention c.Assurance d.Policy development

d. Policy development

To understand the causes of health and disease, epidemiology studies: a. Individuals b. Families c. Groups d. Populations

d. Populations

People who live in developing countries are at risk for a variety of health threats as a result of all factors except a. limited sanitation facilities. b. increased chance of exposure to communicable diseases, including AIDS, tuberculosis, and hepatitis B. c. higher rates of tobacco use compared with most developed countries. d. lack of interest in health issues and health care.

d. lack of interest in health issues and health care. In developing countries, people are more likely to be exposed to communicable diseases as a result of a number of factors, including high population density, lack of accessible health care and treatment, and poor or limited sanitation. In addition, although tobacco use is decreasing in most developed countries, it is increasing in many underdeveloped ones. Although people in developed countries often lack resources and information, they are nonetheless interested in health promotion, disease prevention, and many aspects of health care delivery.

Public service announcements, dissemination of pamphlets, and focus groups are examples of measures that can be used by the community health nurse. These are also known as a. levels of intervention. b. nursing assessment. c. evaluation methods. d. nursing interventions.

d. nursing interventions. Interventions should be included from a range of strategies, including mass media (public service announcements, radio, television, billboards), general information dissemination (e.g., pamphlets, DVDs, CDs, posters), electronic information dissemination (e.g., websites, blogs, tweets, video stream), and public forums (e.g., town meetings, focus groups, discussion groups). Intervention levels apply to aggregates, communities, or individuals. Assessment includes the collection of data to determine the needs of the community or aggregate. Evaluation determines the success or failure of a project. Methods may include verbal or written feedback, surveys, or other tools that measure whether objectives were met.


Ensembles d'études connexes

Chapter 21: America and the Great War, 1914-1920

View Set

Securities Markets and Investment Companies

View Set

REVIEW: Perimeter, Circumference, & ALL Area

View Set